OB-GYNE

¡Supera tus tareas y exámenes ahora con Quizwiz!

A 35 y/o G2P1 at 10 weeks' gestational age experience severe crampy hypogastric pai and vaginal bleeding. One day PTC, she noted passage of meaty tissue, thereafter pain and bleeding subsided. On IE, cervix was smooth and closed. What is the most likely diagnosis? A. Imminent abortion B. Inevitable abortion C. Threatened abortion D. Incomplete aborition E. Complete abortion

47 E. The pain and bleeding subsided after passage of meaty tissue. On PE, the cervix was closed. These findings support the diagnosis of complete abortion.

On what day of the endometrial cycle is subnuclear vacuoles seen? A. D17 B. D18 C. D19 D. Never seen E. D17-19

48 A. D17- subnuclear vacuoles; D18 - vacuoles move to the apical portion; D19 - cells begin to secrete glycoprotein and MPS contents in the lumen

The average length of the funis A. 40 B. 48 C. 55 D. 63 E. NOTA

49 C. The average length of the funis/umbilical cord is 55 cm.

A 25 yr old female presents with vaginal spotting and RLQ pain. A TVS is done which shows empty uterus. Serum HCG was 4000mIU/ml. Which of the following is not a risk factor for her condition? A. PID B. Previous similar diagnosis C. IUD use D. Congenital malformation of the uterus E. None of the above

5 E. This is a case of ectopic pregnancy. The absence of a gestational sac inside the uterus with TVS when the HCG level is >1500mIU/mL is 100% accurate in excluding a live uterine pregnancy. An investigation must be carried out to rule out ectopic pregnancy. Source: Williams Obstetrics 23RD ed p 244

The source of HCG is the A. Syncytiotrophoblast B. Cytotrophoblast C. Decidua D. Fetus E. Nitabuch's layer

50 A.

What is the treatment of choice for a symptomatic Bartholin's duct cyst ot abscess? A. Incision and drainage B. Enucleation C. Marsupialization D. Excision E. Observation

51 C. Marsupialization is preferred due to the high recurrence rate of incision and drainage.

In a patient with a thin, gray-white frothy discharge, what is the most likely diagnosis? A. Trichominiasis B. Atrophic vaginitis C. Bacterial vaginosis D. Candidiasis E. Cervicitis

52 C.

What is the least number of mitosis per hpf with cytologic atypia that leads to a diagnosis of leiomyosarcoma? A. 1 B. 3 C. 5 D. 7 E. 9

53 D.

In radical hysterectomy, the main blood supply of the uterus is ligated close to its origin. The uterine artery is a branch of the following structure? A. Anterior division of internal iliac artery B. Posterior division of internal iliac artery C. External iliac artery D. Common iliac artery E. Posterior iliac artery

54 A. The uterine atery (inferior vesical artery in males) is anterior division branch of internal iliac artery.

The following is a risk factor for endometrial cancer A. Multiparity B. HSV infection C. Diabetes D. OCP use E. NOTA

55 C. Other Risk factors include nulliparity, obesisty, hypertension

Which of the following should be investigated in patients with Mullerian agenesis? A. Cardiac B. Urologic C. Skeletal D. CNS E. GIT

56 B. Mullerian agenesis is associated with urologic anomalies such unlateral renal agenesis.

What is the treatment for a 25 y/o with primary amenorrhea due to 45, XO? A. HRT B. Estrogen C. Progestins D. OCPs E. AOTA

57 D.

Which factor primarily dictates the age of menopause? A. Genetics B. Race C. Number of pregnancies D. Age of menarche E. OCP use

58 A. Genetics primarily dictates the age of menopause.

A 70 y/o multipara consulted because of chronic vulvar pruritus. On pelvic exam, there is a diffuse erythematous eczematoid lesion on the right labia, about 3 x 4 cm? What is the probable clinical diagnosis? A. Paget's Disease B. Melanoma C. HSV infection D. Leukoplakia E. Lichen sclerosus

59 A. The description stated is characteristic of Paget's Disease of the vulva.

Which of the following is NOT true regarding HCG? A. Peaks at 10 wks gestation B. Nadirs at 14-16 wks C. Increases by more than half every 48 hours prior to 10 weeks. D. Urine tests are based on antibody specificity to the alpha subunit of HCG. E. Also elevated in H mole, choriocarcinoma, germ cell tumors, large cell carcinoma of the lung

6 D. Alpha subunit is similar with alpha subunit of LH, FSH and TSH Beta subunits are unique: this is the one tested for in urine pregnancy tests Source: Williams Obstetrics 23RD ed p63

To rule out microinvasion, the mininum procedure to establish diagnosis should be A. Hysterectomy B. Cone biopsy C. Paps smear D. Colposcopy with biopsy E. Incision biopsy

60 B. The minimum procedure should be cone biopsy to document microinvasion of the basement membrane.

The ff scenario pertains to questions 181-183. A 28 yr old nullipara came to your birthing clinic for the first time for a regular checkup? Her last menstrual period was on March 27, 2015. The mother was asking when she expects to deliver. What is the expected date of delivery based on Naegele's rule? A. December 31, 2015 B. January 1, 2016 C. January 2, 2016 D. January 3, 2016 E. February 4, 2016

61 D. Naegele's rule: Month +9; Day +7; Year +1

The mother was asking for an exact date of fertlization/conception. Although you cannot identify the exact date you can predict that it most likely occurred during what date? A. March 13, 2015 B. March 20, 2015 C. April 3, 2015 D. April 10, 2015 E. Same date as the LMP

62 D. The conception age/fertilization age is usally approximated at 2 weeks after the menstrual age.

A CBC was performed as well, you will classify the pregnant patient as being anemic if the Hgb level is below this value? A. 10.0 g/dL B. 10.5 g/dL C. 11.0 g/dL D. 11.5 g/dL E. 12.0 g/dL

63 C. Normal hgb levels depends on the AOG of the px. 1st trimester- 11g; 2nd trimster- 10.5 g (because of hemodilution of the RBC's); 3rd tri- 11g

In doing a clinical pelvimetry, which of the following findings will indicate a clincially adequate pelvis? A. Diagonal conjugate is 10 cm B. Side walls are convergent C. Bispinous diameter is 8 cm D. Sacrum is curved E. Subpubic arch is 90 degress

64 D. A clinically adequate pelvis: Inlet- Diagonal conjugate of atleast 11cm, Engagement of the head, Positive Mueller Hillis manuever; Midpelvis- Sidewall are non convergent, Sacrum is curved, Bispinious diameter is atleast 10 cm; Outlet- Pubic arch is wide (>110 deg), Bituberous diameter is atleast 8cm

In relation to the rpeviosu question, what treatment would you recommend for this patient? A. Expectant management B. Word cathetherization C. I and D D. Marsupialization E. Excision

77 A. In this case wherein the cyst is asymptomatic and there is no sign of infection or abscess (being nontender), expectant management is advised. However for a large painful Bartholin's, a word catheter is palced to releive the obstruction. Marsupialziation is typically reserved for patients in whom the catheter has failed

An 20 year old woman presents to your office complaining of amenorhhea. She never had a mesntrual period before but she complains of mild cyclic abdominal bloating. She reports normal breast dev't at age 11. She reports she has become sexually active bbut finds her intercourse painful. Her past medical history is unremarkable. On PE you noted normal appearing axillary and pubic hair. Her breast dev't is normal. A pelvic exam shows a normal appearing external genitalia and a shortened vagina ending in a blind pouch. WHich of the ff tests would be your first step in diagnosing the patient? A. Karyotyping B. Pelvic UTZ C. Serum FSH D. Hysterosalpingogram E. DIagnostic laparoscopy

78 B. A pelvic UTZ should be the first step aimed to determine the presence or absence of the uterus, tubes and ovaries. Given the presence of normal breasts and pubic hair, an FSH would not be helpful.

On further examination you noted normal appearing bilateral ovaries as well as an absent utrerus and fallopian tubes. What is the most likely diagnosis? A. Imperforate hymen B. Transverese vaginal septum C. Mullerian agenesis D. Androgen insensitivity syndrome E. Asherman's syndrome

79 C. Mullerian agenesis known as Mayer-Rokitansky- Kuster-Hauser syndrome refers to congential absence of the uterus, oviducts and upper vagina.

A 32 year old G2P1 (1001) seeks consult at 6 weeks AOG for initial obstetric visit. Her previous child had a spina bifida. How much folate should be given to prevent NTD's in her current pregnancy? A. There is insufficient evidence that folate can prevent NTDs B. 40 micrograms/day C. 400 micrograms/day D. 4 milligrams/day E. 40 milligrams/day

8 D. 400 micrograms/day ideally started 3 months prior to conception can help prevent NTDs. For those with previous children with NTDs, a higher dose of 4 milligrams/day should be given. Source: Williams Obstetrics 23RD ed 205

Engagement is the entrance of a particular diameter of the fetal head in occiput position into the smallest diameter of the maternal pelvis. This diameter of the fetal head is the? A. Suboccipitofrontal diameter B. Suboccipitobregmatic diameter C. Submentobregmatic diameter D. Occipitofrontal diameter E. Biparietal diameter

9 E. Engagement occurs when the biparietal diameter (the widest transverse diameter of the fetal head in occiput position) reaches the level of the ischial spines. Source: Williams Obstetrics 23RD ed p378

Where can you find the widest portion of the fallopian tube? A. infundibulum B. ampulla C. isthmus D. interstitial E. Intramuscular

164 B.

What is the recommended regimen for intrapartum antimicrobial prophylaxis for perinatal Group B Streptococcal disease? A. Ampicillin, 2 g IV initial dose, then 1 g IV every 4 hours or 2 g every 6 hours until delivery B. Cefazolin, 2 g IV initial dose, then 1 g IV every 8 hours until delivery C. Clindamycin, 900 mg IV every 8 hours until delivery D. Vancomycin, 1 g IV every 12 hours until delivery E. Penicillin G, 5 million units IV initial dose, then 2.5 million units IV every 4 hours until delivery

194 E. Penicillin is the recommended drug. The other drugs listed in the choices are only alternative to penicillin.

A 46 year old asymptomatic patient is found to have an 8 week size irregular myomatous uterus. The best management would be: A. administration of GnRH analogues B. reevaluation in 6 to 12 months C. fractional dilatation and curettage D. myomectomy E. hysterectomy

544 E.

A 34 year old nulliparous sought consult due to progressive dysmenorrhea. Internal examination showed a fixed, retroverted uterus with characteristic nodularities palpated in the uterosacral ligaments and cul-de-sac. A diagnosis of endometriosis was made for which the patient opted to have medical treatment for the moment. Which of the following medications will induce a "medical oophorectomy"? A. Danazol B. Progestins C. OCP D. Leuprolide E. Clomiphene

293 D. Leuprolide acts as an agonist at pituitary GnRH receptors. By interrupting the normal pulsatile stimulation of, and thus desensitizing, the GnRH receptors, it indirectly downregulates the secretion of gonadotropins luteinizing hormone (LH) and follicle-stimulating hormone(FSH), leading to hypogonadism and thus a dramatic reduction in estradiol and testosterone levels in both sexes.

The highest risk for the development of a future Ectopic Pregnancy is which among the following? A. Previous PID B. Tubal Corrective Surgery C. Previous Ectopic Pregnancy D. IUD placement E. Previous abdominal surgery

418 B. SIMILAR TO PREVIOUS BOARD EXAM CONCEPT/PRINCIPLE

At what gestational age does surfactant production begin in the developing fetal lungs? A. 16 weeks AOG B. 18 weeks AOG C. 20 weeks AOG D. 24 weeks AOG E. 28 weeks AOG

400 D.

How many arteries do the umbilical cord posses? A. O B. 1 C. 2 D. 3

576 C. 2 arteries and 1 vein

which of the following is/are a function of human Chorionic Gonadotropin? A. Rescue and maintenance of the corpus luteum B. To promote male sexual differentiation C. Promote uterine vascular dilatation and myometrial smooth muscle relaxation D. stimulation of maternal thyroid hormone production E. all of the above

302 E. williams 22nd ed - pp72-73

Which of the following are possible complications of the above case? A. DIC B. Couveaire uterus C. Acute renal failure D. A and B are correct E. All of the above

103 E. Page 10 of OB2 Topnotch Handout. All are possible complications of abruptio placenta.

Decreased libido in menopause is improved by: A. Testosterone 200mg IM/month B. CEE 0.625 mg daily C. MPA 2.5 mg daily D. Ethinyl estradiol 0.002 mg E. None of the above

114 A. Page 13 of gyne Topnotch Handout. Decreased libido is improved by IM testosterone.

The inter-spinous diameter of the midpelvis should be at least how many centimeters for it to be adequate for vaginal birth? A. 9 B. 9.5 C. 10 D. 11 E. 12

173 B., C. Williams 23rd edition chapter of Labor and Delivery page 472, “there is a reason to suspect midpelvic contraction whenever the interspinous diameter is less than 10cm.†I can also consider B 9.5 since it appeared it anatomy topnotch day 3 handout page 16.

Implantation usually takes place how many days after fertilization? A. 5 B. 7 C. 9 D. 10 E. 12

174 B..usually takes place on the 6th or 7th day after fertilization. Williams 23rd edition page 48

Which among the following is considered a definitive sign of pregnancy? A. Positive pregnancy test B. Breast changes C. Cessation of menses D. Braxton hicks contraction E. Perception of fetal movement by a midwife

166 E.

This is the bluish discoloration of the cervix, vagina, and labia that is caused by estrogen which results in venous congestion. A. Chadwick Sign B. Hegar's Sign C. Goodel's sign D. Spalding sign E. Robert's sigN

167 A. hegar is softening of the uterus. Goodels is softening of vaginal portion of the cervix. Spalding and robert's sign are radiographic evidences of fetal death

In a multiparous mother, she may first perceive fetal movements at ______ weeeks AOG. A. 14-16 weeks B. 16-18 weeks C. 18-20 weeks D. 20-22 weeks E. 22-24 weeks

168 B. for nulliparous it is 18-20 weeks

Which of the following vaccines can be given to a pregnant patient? A. measles B. MMR C. Varicella D. Vaccinia E. Meningococcus

169 E. SIMILAR TO PREVIOUS BOARD EXAM CONCEPT/PRINCIPLE

The following are all absolute contraindications for Aerobic exercise during pregnancy EXCEPT: A. Poorly controlled seizure disorder B. Incompetent cervix C. Restrictive lung disease D. Placenta previa after 26 weeks E. Preterm labor during the current pregnancy

170 A. A is only a relative contraindication

Which among the following Anti TB drugs is contraindicated in pregnant women? A. ethambutol B. rifampicin C. ethambutol D. streptomycin E. All of the above

171 D. this is contraindicated because of ototoxicity to the fetus. SIMILAR TO PREVIOUS BOARD EXAM CONCEPT/PRINCIPLE

Which of the following will constitute the boundaries of the pelvic inlet? A. Sacral promontory B. Ala of the sacrum C. Linea terminalis D. Symphysis pubis E. All of the above

172 E. A and B are posterior boundaries. C is the lateral boundary. D is the anterior boundary

which of the following tocolytics can induce a reversible type of oligohydramnios? A. Magnesium sulfate B. ritodrine C. atosiban D. indomethacin E. Terbutaline

303 D. prostaglandin inhibitors are associated with oligohydramnios. Williams 22nd ed p 871

which of the following fetal malformation is strongly correlated with poorly controlled diabetes mellitus both preconceptionally and during pregnancy? A. Situs inversus B. Spina bifida C. Caudal regression D. Renal agenesis E. Anencephaly

304 C. Williams 22nd ed p1177

use of atenolol as an antihypertensive medication in pregnnacy is associated with which of the following perinatal moribidity? A. Fetal growth restriction B. Cardiac defects C. Hyperglycemia D. Preterm birth E. Renal defects

305 A. Williams 22nd ed p 1051

what is the type of placenta is assoicated with placental abruptio and fetal hemorrhage? A. Bipartite placenta B. Circumvallate placenta C. Succenturiate placenta D. Circummaginate placenta E. Placenta membranacea

306 B. there is an increased risk with circumvallate placentas of antepartum hemmorrhage both from placental abruption and from fetal hemorrhage. Circummarginate placenta risk are less well defined. Placenta membranacea can give rise to serious hemorrhage because of its associated placenta previa or accreta.

What type of forceps should be use in delivery of fetus with molded head ? A. Pipers B. Kielland C. Chamberlain D. Simpsons E. Tucker-Mclane

307 D. In general, Simpsons forceps are used to deliver the fetus with a molded head, as is common in nulliparous women. The tucker-Mclane instrument is often ued for the fetus with rounded head seen in most multiparas. Wlliams 22nd ed p549.

40 y/o G6P5 32-33 weeks AOG complains of vaginal bleeding and severe abdominal pain. VS BP 160/100 PR 123 RR 24 T 37.2C. PE of the abdomen:FH 36cm FH 97bpm, UC: every minute, 50 sec duration, strong. what complication is associated with the above condition?? A placenta accreta B. uterine atony C. DIC D. puerperial infection E. all of the above

308 C. abruptio placenta is associated with DIC, renal failure (ATN), hemorrhagic shock, couvelaire uteurs, and fetal maternal hemorrhage.

A 28 y/o G2P1 mother at 35 week AOG came to your clinic with the following biophysical profile scoring: NST has 3 accelerations greater 15 bpm each for more than 15 seconds, 20 seconds of fetal breathing for 30 minutes, 2 discrete body movements, 1 episode of extremity extension then subsequent flexion. AFI 1.2cm. What is your next plan of treatment? A. No fetal indication for intervention. Repeat test weekly B. Deliver C. Repeat BPP D. Observe E. none of the above

309 C. BPP=4 (NST = 2, fetal breathing = 0, fetal movements = 0, fetal tone = 2, AFI =0). For a BPP of 4, there is probable fetal asphyxia. The intervention is to repeat BPP on the same day. If the result if less than or equal to 6, deliver the baby.

Which is a true statement with regards to variability? A. The baseline HR becomes more variable as rate increases. B. Good variability is an assurance of fetal well- being. C. Normal variability: 10-30 bpm D. Baseline variability is an important index of respiratory function. E. A reduced baseline HR variability is the single most reliable sign of fetal compromise.

323 E. The baselines HR becomes more physiologically fixed as rate increases. Good variability is not necessarily reassuring. Normal variability (moderate): 6-25 bpm. Baseline variability is an important index of cardiovascular function. Williams Obstetrics 23rd edition pp.415-417

True of breast masses: A. It is easier to distinguish between benign phyllodes from fibroadenomas than benign from malignant phyllodes. B. Treatment for phyllodes tumor is radiation therapy. C. Fibroadenomas are not associated with an increased risk of breast cancer. D. Fibroadenomas are benign despite causing some skin dimpling and nipple retraction E. Phyllodes tumors are difficult to distinguish clinically from fibroadenomas but the former are usually bilateral.

337 C. Treatment for phyllodes tumor is wide local excision. Fibroadenomas don’t cause skin dimpling or nipple retraction. Phyllodes tumors are rarely bilateral. Berek & Novaks Gynecology 14th edition pp.657-659

Component/s of active management of labor is/are: A. oxytocin and cesarean delivery B. cesarean delivery C. oxytocin and amniotomy D. all of the above E. none of the above

321 C. SIMILAR TO PREVIOUS BOARD EXAM CONCEPT/PRINCIPLE. Williams Obstetrics 23rd edition p.405

The most common explanation for fetal tachycardia: A. fetal compromise B. fetal cardiac dysrhythmias C. maternal administration of atropine D. maternal fever from infection

322 D. Williams Obstetrics 23rd edition p.413

An involuntary reflex involving contraction of pelvic muscles precipitated by attempts at vaginal entry is known as: A. dyspareunia B. genital arousal disorder C. orgasmic dysfunction D. vaginismus E. sexual assault

338 D. Novaks p.333

The probability of achieving a live birth within a single cycle is termed: A. fecundability B. fertility C. fecundity D. gestation

339 C. Fecundability - the probability of achieving a pregnancy within a single cycle. Berek & Novaks Gynecology 14th edition p.1186

The fecundability of a normal couple is estimated at: A. 10-15% B. 15-20% C. 20-25% D. 25-30% E. 30-35%

340 C. Berek & Novaks Gynecology 14th edition p.1186

A 33 year old G6P5 (5005) consulted due to vaginal bleeding. She is currently on her 30th week AOG. Past OB History were all NSD. On PE, BP - 110/70; PR - 78/min; RR - 20/min; T - 36.7C; FH - 31cm; FHT - 132/min; LM3 - breech, no uterine contractions. Speculum exam showed brownish discharge per cervical os. Which of the following is contraindicated in this patient? A. Digital exam B. Speculum exam C. Transvaginal ultrasound D. All of the above

341 A. This is a possible case of placenta previa (patient is G5P5, breech presentation), therefore internal exam is contraindicated.

Which of the following is not an imminent sign of eclampsia? A. Severe headache B. Visual disturbances C. Epigastric pain D. Facial edema

350. D..

What is the anaesthesia of choice for pregnant cardiac patients? A. General anesthesia B. Epidural anesthesia C. Subarachnoid block D. Spinal anesthesia

351 B.

Conjoined twinning occurs when fertilisation occurs after the first: A. 48 hours B. 5 days C. 8 days D. 12 days

352 D.

In evaluating a 28 year old patient complaining of vaginal bleeding, what information is most important? A. Age at menarche B. History of dysmenorrhea C. Last menstrual period D. OCP use

353 C. Always rule out pregnancy first in patients who are of reproductive age

What is the usual cause of dysfunctional uterine bleeding in the postmenarcheal and premenopausal age groups? A. Hematologic disorder B. ncreased prostaglandin C. Anovulation D. Hyperprolactinemia

354 C.

A 16 year old patient consulted because of primary amenorrhea. On PE, her breasts are fully developed. Pelvic exam: normal external genitalia; a cotton tip applicator could be inserted 3 cm from the vulvar introitus. On rectal exam, there was nothing palpated. What is the most likely cause of her problem? A. Ovarian agenesis B. Transverse vaginal septum C. Mullerian agenesis D. Imperforate hymen

355 C. (+) breast (-) uterus: Mullerian agenesis or Testicular feminization.

What is the possible aetiology of the PID of a woman who is not responding to the usual antibiotic regimen for acute bacterial PID? A. Gonococcal B. Chlamydia C. Mycoplasma D. Tuberculous

360 D.

A 16-year old female presented to your clinic with primary amenorrhea. On physical examination, she has signs of normal breast development. Ultrasound evaluation failed to demonstrate the presence of any uterus and cervix. Karyotype analysis revealed that she is a 46 XX. What is your most likely diagnosis at this point? A. Androgen insensitivity B. Mayer-Rokitansky-Kuster-Hauser syndrome C. Imperforate hymen D. Premature ovarian failure E. Hypogonadotropic hypogonadism

386 B. SIMILAR TO PREVIOUS BOARD EXAM CONCEPT/PRINCIPLE.

During prenatal check up, you palpated the fundus of the mother to be just above the symphysis pubis. Approximately how many weeks is the AOG? A. 12 B. 14 C. 16 D. 18 E. 20

175 A. OB Williams 23rd edition page 79 and OB topnotch handout page 19. at 12 weeks uterus is just palpable above the symphysis pubis

This plane corresponds to the smallest circumference of the fetal head A. Biparietal diameter B. occipitomental C. occipitofrontal D. suboccipitobregmatic E. Bitemporal

176 D.

A patient came in for consult at your clinic due to primary amenorrhea associated with anosmia. Work up showed hypogonadotropic hypogonadism due to deficiency in GnRH. What do you call this disease? A. Asherman's syndrome B. Sheehan's syndrome C. Kallman syndrome D. Turner syndrome E. MRKH syndrome

177 C.

Uterine synechiae is more commonly known as: A. Asherman's syndrome B. Sheehan's syndrome C. Kallman syndrome D. Turner syndrome E. MRKH syndrome

178 A. SIMILAR TO PREVIOUS BOARD EXAM CONCEPT/PRINCIPLE

A G1P1 (1001) came in for consult due to secondary amenorrhea. Her last delivery was complicated by severe post partum bleeding due to retained placental fragments which prompted the attending obstetrician to do endometrial curettage. After that, the patient had no longer had any mentruation. Which among the following is most likely the cause of the secondary amenorrhea? A. Asherman's syndrome B. Sheehan's syndrome C. Kallman syndrome D. Turner syndrome E. MRKH syndrome

179 A. The question is asking for the MOST likely diagnosis or the primary diagnosis. Sheehan's syndrome can be one of your differentials. However, this is less likely the cause of the secondary ammenorhea as compared to Asherman's syndrome since amenorrhea is the only symptom present in the case. A patient with Sheehan's syndrome should also have other signs and symptoms pertaining to hormonal imbalance aside from amenorrhea. SIMILAR TO PREVIOUS BOARD EXAM CONCEPT/PRINCIPLE

To confirm the diagnosis of the answer in number 199, this procedure should be done. A. Transvaginal ultrasound B. Pelvic MRI C. Pelvic CT scan with contrast D. hysteroscopy E. Progesterone-estrogen challenge test

180 D. SIMILAR TO PREVIOUS BOARD EXAM CONCEPT/PRINCIPLE

A G1P0 patient at 36 weeks AOG had a biophysical score of 8 (NST, Fetal movement, Fetal muscle tone, AFV have 2 points each while Fetal breathing had 0 point). What should you tell the patient regarding the result of the test? A. Repeat BPP should be done within 24 hours B. Labor should be induced C. The results are normal and the patient can go home D. Patient should undergo emergent cesarean section E. The results are abnormal, admit the patient for tocolysis

181 C. A biophysical score of 8-10 is normal.

A Pap-smear result of a 28 year old patient revealed inflammation with atypia. What would you advise the patient? A. Colposcopy and biopsy B. Repeat pap-smear after 3 months C. Treat inflammation then repeat pap-smear D. observation E. None of the above

189 C. Inflammation affects the cytologic study so treat the inflammation first then repeat pap smear after treatment.

WHO recommends low risk pregnant patients to have at least how many pre-natal check ups? A. 7 B. 6 C. 5 D. 4 E. 8

190 D. WHO recommends at least 4 prenatal visits for uncomplicated low risk pregnant patients. 1st visit: 8-12 weeks, 2nd visit: 24-26 weeks, 3rd visit: 32 weeks, 4th visit:36-38 weeks

8 days post partum, the mother developed fever. What is the most common cause of post-partum fever at this time? A. endometritis B. Surgical site infection C. Mastitis D. DVT E. UTI

198 C. post-partum fever most likely causes: within 24 hours- atelectasis; 1 day- UTI; 2-3 days- endometritis; 4-5 days- surgical site infection; 5-6 days- DVT; 7-21 days- mastitis.

Implantation happens 1 week post-conception. What is the most common location of implantation in the uterine wall? A. posterosuperior B. anterosuperior C. posterolateral D. anteroinferior E. Inferolateral

199 A. MC site of implantation- posterosuperior wall

What laboratory examination can be done to confirm the diagnosis in #185? A. CXR B. Liver Enzymes C. CBC D. 24 hour urine protein E. Ultrasound

286 D. proteinuria would differentiate pre eclampsia with gestatinal hypertension and chronic hypertension

A 23 year old G1P0 on her 15th week of pregnancy consulted because of fever and right flank pains. T - 38.9C; BP - 110/70; PR - 85/min; RR - 20/min; FHT 137/min by Doppler. The rest of the PE was essentially normal. What is the probably diagnosis? A. Asymptomatic bacteriuria B. Cystitis C. Acute pyelonephritis D. Acute glomerulonephritis

349 C.

Fetal attitude is primarily determined by what Leopold's maneuver (LM)? A. LM1 B. LM2 C. LM3 D. LM4 E. Both B and C

292 D. LM1 presentation LM2 position LM3 engagement

After another 2 hours, the repeat IE showed the following findings: cervix 6 cm dilated, 90% effaced, ROA, st-1, (+) BOW, FHT 135 bpm. How would you know manage this case ? A. Expectant management B. Labor Induction C. Cesarean section D. Cervical ripening E. None of the above

299 C. Dx: Arrest in cervical dilatation. Perform CS

Which of the following skin changes in pregnancy is an effect of Hyperestrogenemia? A. Linea Nigra B. Chloasma C. Darkening of Areola D. Striae Gravidarum E. Spider Nevi

415 E. All hyperpigmented lesions are due to MSH, Striae gravidarum is due to the effects of corticosteroids.

A G1P0 on her 34th week AOG underwent a Biophysical Assesment which revealed the following: 1.)Reactive NST, 2.)One breathing period lasting 10 seconds, 3.) 2 discrete movements of arms, 4.) One definite extension and return to flexion, 5.) AF volume with the largest pocket at 1 cm. What would be the nest step of the OB-GYN? A. Observe and repeat per protocol B. Repeat BPF in 24 hours C. Repeat test weekly D. Do a CST E. Deliver

416 E. BPS is 4 with an abnormal AFV hence protocol states that delivery should be prompted.

A 50 year old female had amenorrhea for 12 months, hot flushes, and decreased libido. She wants to improved her libido. What will you give? A. IM testosterone B. IM estrogen C. IM progesterone D. Combined estrogen and progesterone IM E. combined oral contraceptive

468 A. IM testosterone will improved the libido.

Pelvibc diaphragm consists of the ff, except A. puboperinealis B. ileococcygeus C. coccygeus D. puborectalis E. NOTA

45 E. The pelvic disphragm is composed of the levator ani and coccygeus muscle. The levator ani is composed of the pubococcygeus/puboperinealis (pubovaginalis, puboperinealis and puboanalis), puborectalis and ileoccygeus muscle.

What is the most common symptom of ectopic pregnancy? A. Abdominal pain B. Amenorrhea C. Vaginal bleeding D. Hypotension E. Tachycardia

475 A. Abdominal pain, vaginal bleeding, and amenorrhea are the components of ectopic triad. The most common symptom is abdominal pain. Hypotension, tachycardia, along with peritoneal signs are signs of ruptured ectopic pregnancy.

Which of the following is/are considered physiologic cyst? A. Follicular cyst B. Corpus luteum cyst C. Theca lutein cyst D. All of the choices E. None of the choices

476 D.

A 70 year old female had uterine prolapse. Her cervix descends halfway to the introitus. This is grade: A. I B. II C. III D. IV E. V

477 A. Grading of uterine prolapse is as follows: when the cervix descends half way to the introitus - I; to the introitus - II; outside the introitus - III; entire uterus outside the introitus at all times -IV.

What layer of decidua is eventually lost as pregnancy progresses? A. Decidua parietalis B. Decidua capsularis C. Decidua basalis D. All of the above E. A and B only

478 B. decidua capsularis is the layer overlying the blastocyst.It is lost eventually due to obliteration of the growing fetus.

What is the most common benign solid tumor of the vulva? A. Lipoma B. Bartholin's cyst C. Urethral diverticulum D. Fibroma E. Urethral diverticulum

479 D.

In the ovarian-endometrial cycle, what is responsible in the regulation of granulosa cells proliferation and differentiation as primary follicles grow? A. Transforming Growth Factor-beta B. Growth differentiation factor 9 C. Insulin-like growth factor D. vascular endothelial growth factor E. epidermal growth factor

480 B. GDF 9- regulates proliferation and differentiation of granulosa cells; TGF-beta - regulates extracellular matrix; epidermal growth factor - stimulates stromal cell differentiation; vascular endothelial growth factor modulates angiogenesis; IGF I and II - differentiation of endometrium.

The average duration of the first stage of labor in nulliparous women is? A. 5 hours B. 8 hours C. 10 hours D. 14 hours E. 20 hours

588 B. Average duration of the first stage of labor is 8 hours and 5 hours for nulliparous and multiparous women respectively.

Which of the following characteristics would make a patient NOT amenable to single agent chemotherpay for gestational trophoblastic disease? A. 2 months since pregnancy B. Serum hCG 50, 000 mIU/mL C. Metastasis to the lung D. No prior chemotherapy E. None of the above

589 B. HCG should be below <40,000

22 year old female complains of sudden onset right lower quadrant pain accompanied by vaginal spotting and missed menses. Patient has pallor and tachycardia. Pregnancy test was positive. Pelvic exam is (+) for wiggling tenderness. What is the gold-standard in diagnosing this disorder? A. Transvaginal ultrasound B. Culdocentesis C. Laparoscopy D. Quantitative hCG determintation E. Laparotomy

590 C. Laparoscopy is the gold standard to diagnose ectopic pregnancy

What forceps is used for deep transverse arrest of the head? A. Piper B. Simpson C. Tucker MacLane D. Kielland E. Prague

591 D. Piper - aftercoming head in breeach Simpson - molded head Tucker McLane - round head (mulitparas)

Which of the following structures in the female genital tract comes from the endoderm? A. Ovaries B. Oviducts C. Uterus D. Vagina E. Vulva

600 D. Ovaries, oviducts and uterus came from the mesoderm, the vagina, urethra comes from both the endoderm and mesoderm. Endoderm from the cloaca which are part of the primitive hindgut. The vulva is form from ectoderm.

This ligament extend from the lateral portion of the uterus, arising below and anterior to origin of the oviducts, that is continuous with the broad ligament, outward and downward to the inguinal canal terminating at upper portion of labium majus: A. Broad ligament B. Cardinal ligament C. Transverse cervical ligament D. Uterosacral ligament E. Round ligament

504 E.

This segment of the uterine tube is the widest and most tortuous. This is the most common site of occurrence of ectopic prenancy: A. Intramural B. Interstitial C. Isthmus D. Ampulla E. Infundibulum

505 D. Intramural/Interstitial- within muscular wall; ectopic pregnancy here result in severe maternal morbidity Isthmus-narrowest portion; preferred portion for BTL Infundibulum - fimbriated extremity; funnel-shaped opening of the distal end of the fallopian tube

Mila, 78 year old G5P5 was recently diagnosed to have Endometrial Carcinoma. What is the definitive management? A. Progestin B. Progestin, Do hysterectomy if with bleeding C. Hysterectomy D. TAH-BSO E. Explratory laparotomy

520 D.

37 yo G3P3 consulted at the health center for cervical CA screening. She tested (+) for aceto- whitening. What should be done next? A. Colposcopy B. Annual screening until 2 consecutive normal tests than every 5 years thereafter C. pap smear D. do a punch biopsy E. Treat with antibiotics

554 A.

Ovarian artery/vein is found coursing in the ligament of: A. Infundibulopelvic ligament B. Suspensory ligament C. Ovarian ligament D. All of the above

580 D. They are all referring to infundibulopelvic ligament containing the ovarian vessels

This ligament of the uterus comes from the posterolateral wall going to the supravaginal portion of the cervix? A. Broad ligament B. Cardinal ligament C. Uterosacral ligament D. Round ligament E. Transverse cervical ligament

581 C. The uterosacrals are the landmarks used by the gynecologist to locate the cervix.

Derivatives of the Urogenital Sinus in females except? A. Lower 3/4 vagina B. Vestibule C. Bladder D. Urethra E. Ureter

668 E. The phallic part of the definitive SUG enlarges to become the vaginal vestibule that is caudally closed off externally by the urogenital membrane. It then tears in the course of the 7th week

The female equivalent of the gubernaculum testis is: A. Round ligament B. Ovarian epithelium C. Suspensory ligament of the ovary D. Epoophoron E. None of the above

601 A. Round ligament is the female equivalent of gubernaculum testis. This ligament courses from lateral uterus inserting to the labia majora.

Of the following possible complications, which should be anticipated in a multifetal pregnancy? A. Shoulder dystocia B. Postmaturity C. Multiple congenital abnormalities D. Postpartum hemorrhage E. All of the above

630 D. postpartum hemorrhage is a common complication following a multifetal pregnancy

Fetal flexion, rotation , extension and expulsion occur during which phase of labor? A. Preparatory division B. Acceleration phase C. Phase of maximum slope D. Deceleration phase E.Latent phase

631 D.

A 28 year-old primigravid consulted at ER due to painless vaginal bleeding. She is currently on her 32nd week of gestation. The least you would do is? A. Perform an internal examination B. Do an emergency ultrasound C. Check the Fetal heart tone D. Non-stress test E. No excemption. You should do all of the above

660 A. Internal exam is a NO NO!! The patient is possibly having placenta previa and IE can only be done on a double set-up. You may induce bleeding that is very hard to control.

The Paramesonephric duct if the female genital tract corresponds to which organ in the males? A. Prostate gland B. Testicular appendix C. Prostatic utricle D. Bulbourethral gland E. Urinary bladder

667 B. In males, the paramesonephric or mullerian duct regress to form the appendix of testes.

Strongest ligament supporting the uterus? A. Mesovarium B. Cardinal ligament of Mackenrodt C. Utersacral ligament D. Suspensory ligament of the Ovary E. Broad ligament

669 B. It attaches the cervix to the lateral pelvic wall by its attachment to the Obturator fascia of the Obturator internus muscle, and is continuous externally with the fibrous tissue that surrounds the pelvic blood vessels. It thus provides support to the uterus

True of pelvic conjugates? A. Obstetrical conjugate is the one which can be directly measured clinically B. True anatomic conjugate is the shortest distance between the midpoint of the promontory and inner surface of the symphysis C. Diagonal conjugate cannot be measured clinically D. The obstetrical conjugare is adequate if distance > 10 cms. E. all of the above

670 D. OC - measured by substracting 1.5 to 2 cms from the diagonal conjugate which is the one that can be measured clinically. True anatomic conjugate is the distance between between the promontory and the UPPER border of the inner surface of syphysis.

Which of the following is not commonly associated with Meig's syndrome? A. Right sided pleural effusion B. Malignant ovarian mass C. Ascites D. A and B E. None of the above

19 B. Meig's syndrome is commonly associated with fibromas (benign masses), right sided pleural effusion, and ascites. Source: Topnotch handout on Obstetrics and Gynecology

Maneuver in breech delivery for the head: A. Loveset B. Mauriceau-Smellie-Viet C. Zavanelli D. McRobert's E. Pinard

28 B. Other maneuvers for breech delivery of the head: Bracht and Prague. Loveset: delivery of nuchal arm. Pinard: delivery of the foot. Zavanelli: reversal of the cardinal movements of labor to perform CS, used as last resort

What is the characteristic liver pathologic finding in preeclampsia? A. Centrilobular necrosis B. Macrovesicular steatosis C. Bridging portal fibrosis D. Periportal necrosis E. None of the above

280 D. Periportal necrosis and sinusoidal thrombin deposition is characteristic of preeclampsia/eclampsia

A 37 y/o G1P1 (1001) consulted because of severe dysmenorrhea for 3 consecutive years. Initial consult revealed bilateral adnexal masses about 7cm each with thickened uterosacral ligament. If surgery is an option, what is the most appropriate procedure for this patient? A. TAHBSO B. bilateral oophorocystectomy C. fluid aspiration of the bilateral mass D. no surgical procedure is warranted. E.

312 B. patient is still in a reproductive age. The goal is to remove the mass and to preserve as much as functional ovarian tissue

which of the following BPP parameter is the last to be affected by hypoxia ? A. Fetal breathing B. Fetal movement C. Fetal tone D. Amniotic fluid volume E. NST

320 A. During Hypoxia, the first parameter to be affected is NST while the last is the fetal tone.

Hysteroscopy is best performed on what day of the menstrual cycle? A. 4th day B. 8th day C. 15th day D. 18th day E. 20th day

160 B. The best answer is a week after menstruation. In this case it is in the 8th day of the menstrual cycle.

Hydatidiform mole is a delicate, friable mass of thin-walled translucent, cystic grapelike structures. Which of the following is a characteristic of a complete mole? A. Can have 69 XXX or XXY B. Embryo-fetal parts are often present C. often present as missed abortion D. 50% of cases have large uterine size E. none of the above

161 D. other choices are characteristic of partial moles. Other characteristics of complete mole are: 46 XX or 46 XY; embryo fetus is absent; usually presents as molar gestation.. Partial moles are usually small for dates.

The following teratogens are correctly matched to their effect to the fetusEXCEPT: A. ACE inhibitor: fetal renal damage B. Lithium: ebstein anomaly C. Fluoroquinolone: anemia D. Isotretinoin: craniofacial malformation E. Streptomycin:ototoxicity

162 C. Fluoroquinolone can cause cartilage damage

A post partum patient suddenly had cardiac arrest which eventually lead to her demise. Biopsy to her heart showed cardiomyopathy. Which among the following types of cardiomyopathy is most likely the cause? A. dilated B. restrictive C. hypertrophic D. anaplastic E. metaplastic

163 A. dilated cardiomyopathy is associated with post partum cardiac arrest

A 38 y/o G3P2 (2002) at 32 weeks AOG, complained of minimal vaginal bleeding. She had 2 previous LTCS, the first was due to placenta previa. BP 110/70, PR 76/min, RR 18/min, no uterine contractions noted, FH 32 cm, FHR 138/min, LM3 cephalic. Ultrasound was done and showed the placenta is grade II, posteriorly implanted, edge is covering part of the internal os. What type of previa is this? : A. Total B. Partialis C. Marginalis D. Low lying E. Vasa previa

101 B. Page 9 of OB2 Topnotch Handout. Placenta previa partialis the placenta is partially covering the internal os.

A 40 y/o G6P5 (5005), was brought to the ER due to epistaxis, abdominal pain with minimal vaginal bleeding. Her BP was 200/110, PR 120/min, RR 20/min. LMP: July 31, 2014. She had irregular prenatal care at a lying in clinic. On PE her abdomen was rigid, tender uterus, FH 34cm, LM3 cephalic, FHR 100/min by doppler. Speculum: cervix: minimal bleeding; IE: cervix long, 1 cm, cephalic, floating. Leg edema ++, DTRs ++. Ultrasound showed etroplacental blood clots. What is the most likely diagnosis? A. Patient is in active labor B. Uterine rupture C. Placenta previa D. Abruptio placenta E. Vasa previa

102 D. Page 10 of OB2 Topnotch Handout. The diagnosis is abruptio placenta. Risk factors: Hypertension, advanced maternal age, increased parity. Signs and symptoms: Significant bleeding after 20 weeks gestation. Uterine hypertonus manifested by rigid abdomen and tender uterus. FHR is 100 (fetal distress).

An 18 y/o G1P1 who delivered via LTCS 7 days ago is now having fever. She was in labor for 16 hours and ROM for 20 hours. VS BP 120/70 PR 110/min 0 RR 23/min T 39.9 C. On PE her abdomen is soft, uterine fundus is 2 cm below the umbilicus, (+) direct tenderness on the hypogastric area. Her vaginal discharge is serosanguinous, foul smelling. Hgb 8 gm/dl WBC 23,000 cells/ul. What is the most likely diagnosis? A. UTI B. Pyelonephritis C. Endometritis D. Surgical site infection E. None of the above

104 C. Due to the prolonged rupture of membranes (>18 hours) the patientis now having endometritis as evidenced by the foul smelling vaginal discharge and uterine tenderness.

A 45 y/o G10P8 (8028) delivered via NSD. After 20 minutes, there was still profuse vaginal bleeding. 0 BP 100/70 PR 105/min RR 21/min T 36.8 C. If uterine atony in this patient fails to respond to medications in this patient what is the best management? A. Uterine compression B. Uterine packing C. B Lynch suturing D. Hysterectomy E. None of the above

105 D. Page 11 and 12 of OB2 Topnotch Handout. This patient is in uterine atony unresponsive to medictaions. But since she is already of advanced age with increased parity hence the best management in this case is a hysterectomy.

A 29 y/o G1P0 37 weeks AOG went to your clinic for prenatal check-up. On PE what maneuver will you do to determine the engagement of the presenting part? A. Fundal grip B. Pawlick's grip C. Zavanelli maneuver D. Rubin's maneuver E. Pinards maneuver

106 B. Page 3 of OB Pearls. Pawlick's grip is the third maneuver done in Leopolds and is used to determine the engagement of the presenting part. A is the First maneuver in Leopold's and is used to determine the fetal part lying in the fundus. Pinards is a breech decomposition maneuver, wherehe fingers are pressed in the baby's popliteal fossa causing flexion of the knee, foot is grasped and delivered as footling. Rubins maneuver- fetal shoulders are rocked from side to side by applying force on the mothers abdomen. Zavanelli is cephalic replacement into the pelvis followed by CS.

In BPP, which of the following parameter is most important and reflects chronic hypoxia? A. AFV B. Fetal breathing C. Fetal movement D. Fetal tone E. NST

107 A. Page 3-4 of OB Pearls. The most important parameter in BPP is AFV. Abnormal AFV usually connotes chronic hypoxia.

The following would warrant in-patient treatment for PID except? A. TVS showing thickened, fluid filled tubes with tubo-ovarian complex B. Appendicitis is ruled out C. Patient does not respond clinically to Ambulatory Regimen A D. Patient is concurrently pregnant E. High fever with nausea and vomiting

108 B. Page 9 of Topnotch Handout. The following are conditions for in-patient treatment: 1) Surgical emergencies cannot be excluded; 2) Pregnant; 3)Does not respond clinically to oral antimicrobial therapy; 4)Unable to follow or tolerate an outpatient oral regimen; 5)Severe illness, nausea and vomiting, high fever; 6) Tuboovarian abcess

A 23 y/o HIV positive on HAART with a CD4+ count 3 of <250 cells/mm comes for or prenatal checkup. Which of the following drugs should be discontinued in the first ttrimester? A. Lamivudine B. Zidovudine C. Nevirapine D. Efavirenz E. None of the above

109 D. Page 25 of Topnotch Handout. Efavirenz should be discontinued in the first trimester.

This bipotential structure develops into the urethra, lower 3rd of the vagina, Bartholin's and Skene's glands in the female: A. Genital folds B. Mesonephric duct C. Mullerian duct D. Genital tubercle E. Urogenital sinus

110 E. Page 4 of Gyne Topnotch Handout. The urogenital sinus develops into the male urethra, Cowper's gland and prostate gland in males while in females it develops into the urethra, lower 3rd of the vagina, Bartholin's and Skene's glands

4 y/o child with recurrent urinary tract infection and vulvar itching was referred by her pedia for an apparent "imperforate hymen". On PE there was noted labial fusion with a translucent vertical line at the midline. What is the treatment of choice? A. Surgical correction B. Clobetasol ointment C. Topical estrogen cream D. OCPs E. Antibiotics

111 C. Page 5 of gyne Topnotch Handout. This is a case of Adhesive vulvitis whih is a self limiting consequence of chronic vulvitis. Pathognomonic is the translucent vertical line at the midline. Treatment for this is Topical estrogen cream BID for 3 weeks.

A 25 y/o G1P1 female with previously regular menses consulted due to amenorrhea for 6 months. PE is normal with no signs of virilization. Pregnancy test is negative. You started her on MPA for one week after which she has no withdrawal bleeding. You decided to give CEE for 21 days followed by MPA for 7 days, after which she had withdrawal bleeding. What will you determine next to find out the cause of the amenorrhea? A. Prolactin level B. TSH level C. LH level D. FSH level E. No need for another test

112 D. Page 12 of gyne Topnotch Handout. If patient is positive in the estrogen-progeterone challenge the next step would be to determine the FSH level. If low fsh it is an HP failure, if FSh is high it is probably due to ovarian failure.

If the patient in the above case had no withdrawal bleeding after the estrogen-progesterone challenge test, which of the following could have caused her ammenorrhea? A. Hypothyroidism B. Prolactinoma C. Premature ovarian failure D. PCOS E. Asherman's syndrome

113 E. Page 12 of Gyne Topnotch Handout. If patient is negative in the estrogen-progeterone challenge the next step would be to determine if there is outflow tract obstuction

A 33 y/o female G2P2 presented at the ER with a 2-month history of progressive dyspnea, abdominal distension, and vague pelvic pressure. Physical examination showed dullness in thoracic percussion on the right, (+) fluid wave on abdominal percussion and a unilateral, movable mass on bimanual pelvic exam. Chest radiography showed pleural effusion on the right, and abdominal ultrasonography showed ascites, and a unilateral 5 cm solid mass on the left ovary.The symptoms resolved after removal of the tumor. What ovarian tumor would be the most likely cause of the patient's symptoms? A. Choriocarcinoma B. Sertoli-Leydig cell tumor C. Fibroma D. Mucinous tumor E. Clear cell tumor

115 C. SIMILAR TO PREVIOUS BOARD EXAM CONCEPT/PRINCIPLE. Page 26 of Gyne Topnotch Handout. This is a case of Meig's syndrome typically associated with a fibroma.Meigs syndrome is defined as the triad of benign ovarian tumor with ascites and pleural effusion that resolves after resection of the tumor. Ovarian fibromas constitute the majority of the benign tumors seen in Meigs syndrome. Meigs syndrome, however, is a diagnosis of exclusion, only after ovarian carcinoma is ruled out

Main objective in hormonal treatment in endometriosis: A. Ovulation induction B. Induce hypomenorrhea C. Induce amenorrhea D. Regulate menstruation E. Fertility

116 C. Page 23 of Topnotch Handout. The primary goal of medical management of endometriosis is induction of amenorrhea.

When is it recommended to begin screening with PAP smear? a. 21 years old b. 1 year after the initiation of vaginal intercourse c. Once sexually active d. A and B are correct E. All of the above

117 A. Page 19 of Topnotch Handout. Screening recommendation for PAP smear is 3 years after onset of sexual activity or 21 years old which ever occurs first.

During which phase in a woman's lifetime do oocytes complete Meiosis II: A. At birth B. Puberty C. Ovulation D. Fertilization E. Menopause

118 D. Page 15 of OB1 Topnotch Handout. At birth, what we have are now actually primary oocytes, because they have already entered Meiosis I, which is specifically arrested at Prophase I. So, at birth, it only becomes 2 M. Why? What happened to the other 4 M? They aged (underwent apoptosis/atresia). So in time, after birth, they decrease in population. When they reach puberty, when they start to have menstruation, with influence of LH surge that promotes ovulation, these primary oocytes are now called secondary oocytes because they already completed Meiosis I. They are being prepared for ovulation. At ovulation, when these oocytes are then released from the follicle, the secondary oocytes undergo Meiosis II, and then they are specifically arrested at Metaphase 2. This is important because they became diploid (2N) to haploid (1N). This is again important in meeting another haploid (1N), which is the sperm, so that during fertilization the secondary oocyte completes meiosis II forming a mature ovum, the secondary oocyte and sperm will combine and become a diploid (2N), which is now called a zygote.

The following vaccines may be given to pregnant women except: A. Rabies vaccine B. MMR C. Meningococcal vaccine D. Hepatitis B vaccine E. Pneumococcal vaccine

119 B. SIMILAR TO PREVIOUS BOARD EXAM CONCEPT/PRINCIPLE. Page 30 of OB1 Topnotch Handout. Live virus vaccines eg. Varicella-zoster, MMR, polio, and yellow fever are not recommended during pregnancy. Toxoids or killed bacteria or viruses eg rabies, influenza, pneumococcal, meningococcal and hepatitis B has not been associated with adverse fetal outcomes.

A monochorionic, diamniotic set of monozygotic twins can be expected if division occurs___: A. <4 days B. 4-8 days C. 8-13 days D. 13-15 days E. >15 days

120 B. Page 2 of OB pearls Topnotch Handout. Dichorionic/Diamniotic-Days 1-3, Monochorionic/Diamniotic-Days 4-8, Monochorionic/Monoamniotic-Days 8-13, Conjoined twins-Days 13-15.

A 26-year old female came to your clinic complaining of vaginal itching, dysuria, and frothy greenish vaginal discharge. The most likely diagnosis is: A. Bacterial vaginosis B. Candidiasis C. Trichomoniasis D. Gonococcal infection E. Chlamydia infection

121 C. This is characteristic of Trichomoniasis, caused by Trichomonas vaginalis. Other clinical manifestation include strawberry cervix and foul smelling dicharge. Gonococcal infection among women might not produce recognizable symptoms until complications (e.g. PID) have occured. SIMILAR TO PREVIOUS BOARD EXAM CONCEPT.

According ACOG, what criteria should be met before arriving at a diagnosis of Dystocia? A. Uterine contraction pattern of 200 Montevideo units or more in a 10 minute period has been present for 2 hours without cervical change. B. The latent phase has been completed with cervix dilated at 6 cms or more. C. The first stage of labor has lasted for more than 20 hours in nullipara. D. There is a complete cessation of cervical dilation. E. The uterotonic has already been given but to no avail.

122 A. According to ACOG, 2 criteria should be met before diagnosis: 1. The latent phase has been completed with cervix dilated 4 cm or more. 2. Uterine contraction pattern of 200 MV units or more in 1 10 minute period has been present for 2 hours witut cervical change. (SIMILAR TO PREVIOUS BOARD EXAM CONCEPT)

A 23-year-old woman, G2P1 (1001) whose LMP was 8 weeks ago presented to the emergency room with history of nausea and vaginal bleeding for the last 3 days. On examination, her fundus is palpated midway between the umbilicus and symphysis pubis. No adnexal tenderness elicited. What is the most likely diagnosis? A. Ectopic pregnancy B. Missed abortion C. Cervical cancer D. Dysfunctional uterine bleeding E. Hydatidiform mole

123 E. You will suspect molar gestation for symptoms of pregnancy (amenorrhea, nausea/vomiting), presence of bleeding prior to 16 weeks, and fundus larger than dates (most common finding).

Milk ejection is the result of the action of: A. Oxytocin B. Prolactin C. Estrogen D. Human placental lactogen E. Progesterone

124 A. SIMILAR TO PREVIOUS BOARD EXAM CONCEPT

A 24-year-old patient presented with nausea, vomiting, lower abdominal pain, fever, and mucopurulent vaginal discharge. Pregnancy test was negative. On pelvic examination, there is presence of left adnexal tenderness. Transvaginal ultrasound was requested. Which of the following situation warrants admission? A. Presence of tubo-ovarian abscess B. Elevated ESR and CRP C. Presence of abundant numbers of WBC on vaginal fluid D. Abnormal mucopurulent discharge E. Temperature of more than 38.3 C

125 A. Decision for in-patient treatment: surgical emergencies cannot be excluded, pregnant, does not respond clinically to oral antimicrobial, unable to follow or tolerate outpatient oral regimen, severe illness, tubo-ovarian abscess.

The in-patient antibiotic regimen for pelvic inflammatory disease include: A. Ceftriaxone 250 mg IM single dose + Metronidazole 500 mg oral BID for 14 days B. Cefoxitin 2 g IM and Probenecid + Doxycycline 100 mg oral BID for 14 days. C. Ceftriaxone 250 mg IM single dose + Doxycycline 100 mg oral BID for 14 days with or without Metronidazole D. Cefoxitin 2 g IV q6 + Doxycycline 100 mg oral BID for 14 days E. Cefotetan 2 g IM single dose + Gentamicin 3mg/kg/dose

126 D. Option C - Ambulatory regimen

A 30-year-old female comes to your clinic with 6- month history of irregular and unpredictable menses. Her pregnancy test is negative. She weighs 128 lbs, and she is 5'4". Your main impression is anovulatory bleeding. How will you manage this patient if she still desires fertility? A. Conjugated equine estrogen B. Medroxyprogesterone for 7-10 days C. Leuprolide D. Cyclic progestin E. Danazol

127 D. Progestins is the treatment of choice for anovulatory DB. For those who desires contraception: OCP. Otherwise, give cyclic progestin therapy from Day 14-25 of each cycle.

An 18-year-old female was brought by her mother to your office. Her daughter has never had a menstrual period ever. On examination, patient had breast but with short vagina. However on ultrasound, she had absent uterus. Karyotyping done showed 46 XX. This is a case of: A. Androgen insensitivity B. Turner's syndrome C. Premature ovarian failure D. Outflow obstruction E. Mayer-Rokitansky-Kuster-Hauser syndrome

128 E. MRKH or mullerian agenesis is a congenital malformation characterized by a failure of the Mullerian duct to develop resulting in complete or partial absence of uterus and cervix and variable degrees of vaginal hypoplasia of its upper portion. I

The definitive sign of pregnancy seen with transvaginal ultrasound is: A. An anechoic sac B. A thin-walled sace within the uterus C. A gestation sac containing a yolk sac D. A lemon sign E. An intradecidual sign

129 C. An anechoic sac (intradecidual sign) without a distinct chorionic ring is the earliest sign of pregnancy. A thin-walled sac within the uterus is not definitive evidence of pregnancy. A lemon sign is a cranial sign of a neural tube defect.

Which of the following is a probable sign of pregnancy? A. Perception of fetal movement by the examiner B. Changes in cervical mucus C. Softening of the cervix D. Chadwick's sign E. Cessation of menses

130 C. Option A. Definitive sign. Option B - Presumptive. Option D - Presumptive (according to William's), Option D - Presumptive.

Variable decelerations in fetal heart monitoring may indicate? A. A response to movement B. Head compression C. Uteroplacental insufficiency D. Umbilical cord compression E. No prognostic significance

131 D. acceleration- a response to movement; Early deceleration - Head compression; late deceleration: uteroplacental insufficiency; Variable deceleration- umbilical cord compression. SIMILAR TO PREVIOUS BOARD EXAM CONCEPT.

The single most important risk factor in the development of post partum pelvic infection is: A. Prolonged labor B. Multiparity C. Early rupture of membranes D. Route of delivery E. Presence of urinary tract infection

132 D.

A 35-year old G2P1 at 18 weeks AOG comes for a prenatal visit. On examination, BP 160/90, 2+ bipedal edema, FHT 144 bpm. Spot urine dipstick showed 2+ protein. What is the most likely diagnosis? A. Preeclampsia, severe B. Mild pre-eclampsia C. Chronic hypertension D. Chronic hypertension with superimposed pre- eclampsia E. Gestational hypertension

133 D. BP > 140/90 before pregnancy or before 20 weeks : chronic hypertension. The presence of new onset proteinuria in hypertensive women : superimposed preeclampsia

A 28-year old female had her pap smear which revealed HGSIL. Biopsies shows carcinoma in situ. Management for this patient is: A. Hysterectomy B. Cold knife conization C. Chemoradiation D. Colposcopy and repeat biopsy after 3 months E. Repeat pap smear after 6 months

134 B. PAP smear LSIL - observe and repeat pap smear after 3 mos. HGSIL with moderate dysplasia - ablative or excisional; HGSIL, carcinoma in situ, child brearing/desires fertility- therapeutic conization. SIMILAR QUESTION ASKED IN THE BOARDS

A 58-year old G4P4 women comes to your clinic with a 3-week history of painless vaginal bleeding. Her blood pressure is 130/90. She has a history of breast cancer and have taken Tamoxifen for 3 years. What will be your next step? A. Do an endometrial biopsy B. Advise hysterectomy C. Prescribe with cyclic progestins D. Do a pap smear and colposcopy E. Refer for TAHBSO

135 A. Prolonged estrogen exposure (Tamoxifen use) is a risk factor for endometrial cancer. The most common presentation is post menopausal bleeding. Diagnosis is done through endometrial biopsy to determine presence of atypia, which warrants hysterectomy.

A 24-year-old woman presented with absence of menses for 6 months now. Urine B-HcG is negative. She does not display any signs of virilization. Conjugated Equine Estrogen was given for 21 days followed by Medroxyprogesterone. No withdrawal bleeding was noted. You will suspect: A. Premature ovarian failure B. Outflow obstruction C. MRKH D. Hypopituitarism E. CAH

136 B. If there is no withdrawal bleeding after a Progesterone Challenge test and Estrogen- progesterone challenge test, you consider outflow obstruction. Request for HSG.

A 33-year old patient, nulligravid, comes to your clinic due to a 3 year hisotry of dull, aching pain during menstruation. On pelvic examination, you note a tender, retroverted uterus, with uterosacral nodularity. The most likely diagnosis is: A. Adenomyosis B. Endometriosis C. Endometrial polyp D. Leiomyoma E. Cervical polyp

137 B. Classic pelvic findings in endometriosis include a fixed, tender uterus with scarring and tenderness posterior to the uterus on IE, and characteristic nodularities in the uterosacral and cul-de-sac.It responds to cyclic changes in ovarian hormone production. Adenomyosis usually presents with diffusely enlarged uterus. Both, however, presents with dysmenorrhea or painful menstruation. SIMILAR TO PREVIOUS BOARD EXAM CONCEPT.

A 30-year old, non-pregnant female presented with a 5 cm solid adnexal mass on ultrasound. She is not on OCP. What is the next step in the management of her condition? A. Observe for 8-12 weeks, then repeat ultrasound. B. Pelvic laparoscopy C. TAHBSO D. HSG E. OCP

138 B. Indications for Pelvic laparoscopy/Surgery(Cystectomy): solid adnexal mass (regardless of age), adnexal mass after menopause or before puberty, cystic mass >8 cm, cystic mass of 5-8 cm persisting more than 8 weeks in a menstruating woman, cystic mass in patient on OCP

You will advise a woman using the oral contraceptive for the first time to start taking it: A. On Day 1 of menses B. On Day 5 of menses C. On Day 7 of menses D. On days 14 of menses E. Anytime as long as she is not pregnant

139 A. SIMILAR TO PREVIOUS BOARD EXAM CONCEPT

A 40-year-old non-obese female presented with a 5-month history of bleeding in between periods. Her BP is 120/70. Hysterosalphingograpy was requested. You will advise the patient to have it done A. During periovulatory period B. Before menses C. After menses D. Anytime during the cycle E. On day 21 of cycle

140 C. The hysterosalpingography procedure is best performed on days 5-12 of menstrual cycle to make certain that patient is not pregnant during the exam. SIMILAR TO PREVIOUS BOARD EXAM CONCEPT.

MI, a 28 year old G1 was admitted for NSD, while attending to the patient, you noted uterine rupture when which of the following were noted A. Sudden retraction of the presenting part B. Sudden drop in maternal BP C. Sudden onset of painful contractions D. A and B only E. All of the above

141 D. A and B are symptoms noted in a uterine rupture.

MJR, a 40 year old G8P7 was admitted due to labor pains, upon PE you noted a transverse lying fetus with the shoulder as the presenting part, what is your plan of management? A. Classical CS B. Low transverse CS C. Manipulate the fetus to convert into cephalic D. Deliver the baby vaginally E. None of the above

142 A. A neglected transverse lie is one of the indications to perform a classical Cesarean Section.

PES, a 30 year old G2P1 at 30 weeks AOG was admitted because of severe pre-eclampsia, the loading dose for the anti-convulsant of choice for seizures due severe pre-eclampsia is A. 1 - 2 gm/hr IV slow bolus B. 4 gm slow IV bolus to run for 2 hours C. 10 g IM on each butt cheek D. All of the above E. B and C

143 B. The loading dose for magnesium sulfate is 5 gm IM on each butt cheek, 4 gm slow IV bolus for 2 hours and the maintenance dose is 1-2gm/hr IV drip or 5gm IM every 6 hours

KL, a 30 year old G2P1 at 24 weeks AOG has malaria, as her OB you know that aside from quinine, another alternative drug for malaria safe to use during pregnancy is? A. Quinine B. Chloroquine C. Mefloquine D. Artemether-lumefantrine E. None of the above

144 B. Chloroquine is safe for pregnant women regardless of the trimester of pregnancy.

TC, a 30 year old female came in at the ER due to vaginal bleeding, upon further examination, you were able to conclude cervical pregnancy, which among the following is the treatment of choice A. Exploratory laparotomy B. Systemic methotrexate C. Intra-amniotic injection of methotrexate, KCl and hyperosmolar glucose D. B and C E. All of the above

145 D. Both B and C are part of the management of a cervical pregnancy, curettage is the procedure needed to completely evacuate the products of conception

MSD, a 30 year old female came at the OPD clinic because of vaginal discharge, on PE you noted a green-yellow, forthy vaginal discharge with a strawberry cervix, what is the treatment of choice? A. Clindamycin B. Itraconazole C. Fluconazoe D. Metronidazole E. Ciprofloxacin

146 D. This is a case of trichomoniasis and the treatment of choice is metronidazole

PM, a 28 year old G1P0 at 18 weeks AOG is being treated for hyperthyroidism, she asks you why you are changing her hyperthyroid medication methimazole to PTU, you explain that PTU is preferred for pregnant patients because: A. It does not cross the placenta as readily as methimazole B. It does not block the peripheral conversion of T4 to T3 C. It acts on a different pathway of thyroid hormone synthesis D. All of the above E. None of the above

147 A. PTU does not readily cross the placenta as much as methimazole does. PTU also blocks the conversion of T4 to T3 in the peripheral tissues. Both PTU and methimazole have the same site of action.

KS, a 17 year old female was brought by her mother due to amenorrhea, upon PE, you noted anosmia, what is the most likely diagnosis? A. Fitz-Hugh-Curtis syndrome B. Mayer-Rokitansky-Kuster-Hauser syndrome C. Ashermann syndrome D. Sheehan's syndrome E. Kallman's syndrome

148 E. Kallman's syndrome is the congenital absence of GnRH producing cells causing primary amenorrhea associated with anosmia. Sheehan's syndrome is hypopituitarism caused by ischemic necrosis of the pituitary gland. Ashermann's syndrome is intrauterine adhesions usually from repeated curettage.

A post-menopausal patient came in following an endometrial biopsy, results showed complex hyperplasia with atypia, as her attending OB-Gyne, what will you do? A. Observe the patient and repeat biopsy after 6 months B. Start chemotherapy C. Give progestin tablets D. Schedule the patient for hysterectomy E. Perform dilation and curettage

149 D. In a post-menopausal patient wuth complex hyperplasia with atypia, the risk of endometrial CA is 50%, the treatment therefore is to remove the uterus by performing hysterectomy

The only cure for severe pre-eclampsia A. Control the BP B. Delivery of the fetus C. Giving tocolytics D. Giving anti-epileptics E. None of the above

150 B. Delivery of the fetus is the only cure for severe pre- eclampsia, especially if the fetus is mature enough to survive outside the uterus, then delivery of the fetus will improve the mother's pre-eclampsia

A 28 year old G2P1 patient came in at the ER due to labor pains, on history, you noted she has GDM currently treated with insulin and the latest ultrasound showed that baby weighs approximately 5kg, on PE, you noted that the cervix is fully dilated and the head is at station 0, what is the maneuver that you will perform in order to push the head and the baby back into the uterus followed by a Cesarean Section A. Zavanelli Maneuver B. McRobert's maneuver C. Chavis maneuver D. Hibbard's maneuver E. None of the above

151 A. McRobert's maneuver - sharply flexing the thighs upon the patient's own abdomen which straightens the sacrum relative to the lumbar vertebrae. Chavis maneuver - shoulder horn instrument consisting of a concave blade is slipped between symphysis pubis and impacted shoulder. Hibbard's maneuver - pressure is applied to the infant's jaw and neck in the direction of the mother's rectum with strong fundal pressure applied by an assistant as anterior shoulder is freed.

You are a resident attending to a patient in labor, you incorporated 10 units of oxytocin on the present IVF, you know that you have to stop the oxytocin if: A. Contractions persist at a frequency more than 5 in a 10 minute period or 7 in a 15 minute period B. There is a persistent non-reassuring fetal heart rate pattern C. There are early decelerations associated with uterine contractions D. A and B only E. All of the above

152 D. The presence of both signs warrant stoppage of the oxytocin infusion and re assessment of the mother for any problems, option C only tells that the heart rate of the fetus decreases just before each contraction and is a sign of head compression against the birth canal

Which among the following is a virilizing tumor A. Granulosa - Theca cell tumor B. Sertoli - Leydig cell tumor C. Endodermal sinus tumor D. Choriocarcinoma E. Clear cell adenocarcinoma

153 B. Sertoli Leydig cell tumor resembles fetal testes and secrete testosterone causing virilization

MGB, a 16 year old was brought by her mother due to cessation of menses, you asked her menstrual history and noted that she started her menses at 12 years old and was subsequently regular occuring every 28-30 days but suddenly ceased for 3 months. All other physical examination findings were normal, you noted that the most common cause of secondary amenorrhea is A. Mayer-Rokitansky-Kuster-Hauser syndrome B. Ashermann's syndrome C. Sheehan's syndrome D. Pregnancy E. Turner's syndrome

154 D. Pregnancy is still the most common cause of secondary amenorrhea and should be ruled out first in any woman of the reproductive age presenting with amenorrhea

In a menstrual cycle of a normal woman, the post ovulatory phase is _________. A. Constantly 12 days B. Usually 12 days but variable C. Constantly 14 days D. Usually 14 days but variably E. Constantly at 16 days

165 C.

KCL, a 27 year old G2P2 post NSD for 2 days came in to your clinic due to dizziness, all other physical exam parameters were normal except for a BP of 160/100, you asked if she had prior BP measurements like these and the patient said she had elevated BP measurements before but she was asymptomatic, you know that to fit the diagnosis of gestational hypertension, her elevated BP must have been elevated after 20 wks AOG and up to? A. 12 weeks post partum B. 6 weeks post partum C. 8 weeks post partum D. 4 weeks post partum E. None of the above

155 A. Gestational hypertension is BP elevation from a previously normotensive patient detected AFTER 20 weeks AOG and should resolve by 12 weeks post partum (Williams Obstetrics, 24th ed.)

MR, a 32 year old G2P1 at 10 weeks AOG came in due to vaginal bleeding, upon PE, her BP was 140/90, you noted on ultrasound a Swiss cheese pattern, what is the treatment of choice A. Metronidazole B. Suction dilation and curettage C. None, observe and come back after 2 weeks D. Tocolysis E. None of the above

156 B. This is a case of incomplete H-mole as the ultrasound pattern is Swiss cheese, snow storm pattern is seen in complete H-mole. Suction D & C is the treatment of choice

MR, same patient as above asked if she could get pregnant, you said that while under treatment, she could not get pregnant for how long? A. 6 months B. 9 months C. 1 year D. 1 1/2 year E. None of the above

157 A. or C. The patient diagnosed with molar pregnancy undergoing treatment and observation should not get pregnant for 1 year if she underwent chemotherapy for GTD to prevent teratogenic effects to the fetus and 6 months if she was only put on hormonal contraception to prevent pregnancy.

MS, a 31 year old G1P1 came in to your clinic due to malodorous vaginal discharge, with a characteristic fishy odor, squamous cells surrounded by lots of bacteria without any inflammatory cells and vaginal pH >4.5, what is the treatment of choice? A. Metronidazole B. Clindamycin C. Itraconazole D. Ciprofloxacin E. Cefuroxime

158 A. This is a case of bacterial vaginosis and the microscopic finding is the description of clue cells seen in bacterial vaginosis

MR, a 32 year old G2P1 at 10 weeks AOG came in due to vaginal bleeding, upon PE, her BP was 140/90, you noted on ultrasound a Swiss cheese pattern, what is the treatment of choice A. Metronidazole B. Suction dilation and curettage C. None, observe and come back after 2 weeks D. Tocolysis E. None of the above

159 B. This is a case of incomplete H-mole as the ultrasound pattern is Swiss cheese, snow storm pattern is seen in complete H-mole. Suction D & C is the treatment of choice

A 22 year old nulligravid, amenorrheic for 8 weeks, came in due to right lower quadrant abdominal pain with vaginal spotting. Pelvic exam shows right adnexal tenderness.Pregnancy test is positive. hCG level is 3500. Ultrasound revealed no gestational sac. This condition is treated medically if the following criteria is present except A. >6 weeks AOG B. tubal mass <3.5cm C. absence of fetal heart motion D. hCG <15000 E. unruptured

182 A. Ectopic pregnancy is treated medically if the following criteria is present <6 weeks AOG tubal mass <3.5cm absence of fetal heart motion hCG <15000 unruptured

What condition most likely predispose a patient to develop ectopic pregnancy? A. Previous cervical conization B. Pelvic inflammatory disease C. Use of IUD D. Induction of ovulation E. Exposure in utero to DES

183 B. Any factor delaying transit of the ovum through the fallopian tube may predispose a patient to ectopic pregnancy. The major predisposing factor is PID.

24 year old G1P0 at 35 weeks AOG is diagnosed with urinary tract infection. Which of the following is the best choice for treatment? A. Cephalosporin B. Tetracycline C. Sulfonamide D. Chloramphenicol E. Erythromycin

184 A.

A patient with cervical cancer underwent intravenous pyelogram study which showed hydronephrosis. How do you manage the patient at this stage? A. Extrafascial hysterectomy B. Chemoradiation C. TAHBSO D. USO E. TAHBSO + chemoradiation

185 B. Cervical cancer with hydronephrosis is Stage III. Management for cervical cancer: Extrafascial hysterectomy for Stage IA to Stage IIA; Chemoradiation for highly advance carcinoma

Diagnostic work up for left lower quadrant pain in an 8 year old girl revealed an ovarian neoplasm. What is the most common ovarian tumor seen in this type of patient? A. Papillary serous epithelial B. Fibrosarcoma C. Germ cell tumor D. Brenner tumor E. Sarcoma botryoides

186 C. Germ cell tumors are the most common ovarian neoplasm in young age group

Mother brought her daughter for consult because of early onset menstruation. The girl was noted to have thelarche at 7 years old, adrenarche at 8 years old and menarche at 9 years old. What is the most common cause of this condition in girls? A. CNS tumor B. Hypothyroidism C. McCune-Albright syndrome D. Gonadal tumors E. Idiopathic

187 E. Diagnosis is precocious puberty. The most common cause is idiopathic.

A G3P2 (2012), underwent completion curettage for incomplete abortion 8 months ago, is being investigated for secondary amenorrhea. Pregnancy test is negative. There is no withdrawal bleeding in progesterone challenge test and in estrogen- progesterone challenge test. What should you request next? A. FSH level B. Endometrial biopsy C. Hysterosalphingogram D. Prolactin level E. TSH determination

188 C. A negative estrogen-progesterone challenge test is most likely caused by an outflow tract obstruction. In this case, because of history of completion curettage for incomplete abortion, the main consideration in the diagnosis is Asherman syndrome/endometrial adhesions.Diagnosis is done through Hysterosalphingogram

28 year old sexually active female, nulligravid, presented with fever, nausea, vomiting and severe pelvic pain. Pelvic exam is positive for adnexal tenderness. Diagnosis was PID and she was managed accordingly. After several treatment regimens, symptoms still recur and there were noted infertility and abnormal uterine bleeding. Laparoscopy was done and revealed a "tobacco pouch appearance" of the oviducts. What is the diagnosis? A. Gonococcal PID B. Pelvic TB C. Non-gonococcal PID D. Actinomyces infection E. Endometriosis

191 B. The clinical symptoms and signs of pelvic tuberculosis are similar to the chronic sequelae of nontuberculous acute PID. The predominant presentations of this chronic infection are infertility and abnormal uterine bleeding. Pelvic tuberculosis may not be diagnosed until laparotomy or celiotomy, when the characteristic changes may be visualized. The distal ends of the oviduct remain everted, producing a “tobacco pouch†appearance.

Patient is G1P0 at 36 weeks AOG, had sudden gush of fluid in the vagina 2 hours prior to consult. Patient is now in labor. Which of the following is not included in the management of this patient? A. GBS prophylaxis B. Steroids to hasten lung maturity C. Deliver the baby D. All of the above E. None of the above

192 B. Steroid are given if AOG is <34 weeks.

What type of breech has the highest incidence of cord prolapse? A. Complete B. Footling C. Frank D. B and C E. A and B

193 B. Incomplete or footling breech has the highest incidence of cord prolapse. Lowest incidence in frank breech

In chloroquine-susceptible areas, what is the prophylactic drug of choice for pregnant women? A. Chloroquine B. Mefloquine C. Doxycycline D. Primaquine E. Co-Artem

195 A. Chloroquine prophylaxis is safe and well tolerated in pregnancy. It has been shown to decrease placental infection from 20 down to 4 percent in asymptomatic infected women in areas without chloroquine resistance. For travelers to areas with chloroquine-resistant P. falciparum, mefloquine is currently the only chemoprophylaxis recommended. Primaquine and doxycycline are contraindicated in pregnancy

Propylthiouracil is the drug of choice for thyrotoxicosis in pregnancy because A. It has a short half-life B. It is highly protein bound C. It blocks peripheral conversion of T4 to T3 D. It has less adverse effects E. All of the above

196 B. PTU is the drug of choice for thyrotoxicosis in pregnancy because it is highly protein bound, thus preventing from crossing the placental barrier.

At what AOG does hCG levels start to decline? A. 8-10 weeks B. 12-14 weeks C. 22-24 weeks D. 18-20 weeks E. 14-16 weeks

197 D. hCG secretion starts at 8 days, peaks at 8 weeks, and starts to decline at 18-20 weeks AOG. Mnemonics: HCG = 8CG

Pseudomenopause state is achieved by giving A. OCP single daily monophasic for 6-9 months B. Danazol 400-800mg/day for 6 months C. Leuprolide 3.75mg IM/month D. Clomiphene citrate 250mg/day E. All of the above

200 B. OCP is used to achieve pseudopregnancy state, Danazol for pseudomenopause and GnRH agonist (Leuprolide) for medical oophorectomy. Clomiphene is used for ovulation induction.

The Arias-Stella reaction refers to: A. Whitening of the vulva after application of acetoacetate B. Poor Pap smear quality during pregnancy C. Ferning of cervical mucus observed in preterm labor D. Violin string-like adhesions in perihepatitis E. Increased intensity of uterine contraction with cervical motion

201 B. Pregnancy is associated with both endocarvical gland hyperplasia and and hypersecretory appearance which makes the identification of of atypical glandular cells on Pap smear particularly difficult.

Amnioinfusion finds clinical use in the setting of: A. Fetal head compression B. Uteroplacental insufficiency C. Cord compression D. Pre-eclampsia E. Chorioamnionitis

202 C. Transvaginal amnioinfusion has been extended into three clinical areas: Treatment of variable or prolonged decelerations, prophylaxis for women with oligohydramnios, as with prolonged rupture of membranes, and attempts to dilute or wash out thick meconium.

Which hormone is reponsible for uterine quiescence? A. Estrogen B. Progesterone C. Prolactin D. Oxytocin E. Cortisol

203 B. Estrogen for uterine activation.

A 23 year-old nullipara is admitted at the labor room at 10:00 o'clock A.M. IE findings are: 4 cm dilated, 70% effaced, cephalic, station -2, intact bag of water. Oxytocin was given. Uterine contractions are strong and regular. At 12:00 o'clock P.M., IE findings are: 7 cm dilated, 80% effaced, cephalic, station -2, ruptured bag of water. Is this patient experiencing normal labor? Why or why not? A. Yes, this is normal labor. Uterine contractions are regular and strong. B. Yes, this is normal labor. Cervix is 7 cm dilated, 80% effaced. C. Yes, this is normal labor. Bag of water has ruptured. D. No, this is abnormal labor. There is no change in descent. E. No, this is abnormal labor. Bag of water has already ruptured.

204 D. Arrest of descent is defined as no change in fetal descent for 1 hour.

A 27 year-old G3P2(2002) at 34 weeks and 5 days age of gestation presents to the emergency department due to labor pains. IE findings are: 4 cm dilated, 70% effaced, cephalic, station -2, ruptured bag of water. What is the best course of action? A. Admit and administer tocolytics. B. Induction of labor. C. Cesarean section. D. Perform transvaginal ultrasound. E. Send the patient home.

205 B. For patients with preterm ruptured membranes at >34 weeks AOG, proceed to delivery usually by induction of labor. GBS prophylaxis is recommended.

A 34 year old multigravid at 35 weeks and 3 days AOG presents for a prenatal check up. After performing the Leopold's maneuver, you discovered that the fetus is in a transverse lie. If this abnormal lie is due to a placental abnormality, which condition is most likely present? A. Abruptio placenta B. Placenta accreta C. Placenta previa D. Battledore placenta E. Circumvallate placenta

206 C. Some of the more common causes of transverse lie include: high parity, preterm fetus, placenta previa, abnormal uterine anatomy, hydramnios, and contracted pelvis.

The results of a biophysical profile done to a 40 year old G1P0 at 32 weeks and 4 days AOG with suspected intrauterine growth restriction are as follows: Reactive NST, AFV 1 cm, 2 discrete body movements, 1 episode of breathing lasting >30 seconds, 2 episodes of extension with return to flexion. What should you do next? A. No fetal indication for intervention. B. Repeat test after 1 week. C. Repeat test twice weekly. D. Repeat test within 24 hours. E. Deliver.

207 E. BPS score is 6 with abnormal AFV. This translates to possible fetal asphyxia.

A 25 year old G1P0 at 37 weeks and 6 days of AOG presents to the emergency department due to labor pains. BP was 200/120. Dipstick analysis of urine revelaed 3+ proteinuria. Patient complains of blurred vision and severe epigastric pain. How should magnesium sulfate be given for this patient? A. 4 grams IV bolus followed by 5 gm IM on each buttock B. 5 grams IV bolus followed by 4 gm IM on each buttock C. 4 grams IM on each buttock followed by 5 gm IV bolus D. 5 grams IM on each buttock followed by 4 gm IV bolus E. None of the above

208 A. The dosages for severe preeclampsia are the same as for eclampsia. Because labor and delivery is a more likely time for convulsions to develop, women with pre-eclampsia-eclampsia usually are given magnesium sulfate during labor and for 24 hours postpartum.

The crown-rump length is compatible with the age of gestation up until how many weeks? A. 4 weeks B. 8 weeks C. 12 weeks D. 28 weeks E. 32 weeks

209 C. Up to 12 weeks, the crown-rump length is predictive of gestational age within 4 days.

A 32 year-old G3P3 (3003) delivered via normal spontaneous delivery a healthy baby boy. On her third postpartum day, the patient had febrile episodes at 39 degree Celsius accompanied by chills. She complains of abdominal pain. On bimanual examination, parametrial tenderness is elicited. Which among the following antimicrobial regimens is considered the gold standard for this patient's condition? A. Clindamycin + aztreonam B. Clindamycin + gentamicin C. Imipenem + cilastatin D. Extended spectrum penicillins E. Extended spectrum cephalosporins

210 B. Clindamycin 900 mg + gentamicin 1.5 mg/kg q8h IV

A 28 year-old G3P2 (2002) at 38 weeks and 4 days AOG came in due to vaginal bleeding. Her first pregnancy was delivered via low-transverse cesarean section due to malpresentation. For the second pregnancy, she delivered vaginally a healthy baby girl. At present, IE findings are: 5 cm dilated, 80% effaced, cephalic, station -2, intact bag of water. Which factor makes this patient a candidate for normal spontaenous delivery? A. Young age of patient B. Term pregnancy C. The patient is in active phase of labor. D. Prior vaginal delivery E. Low parity

211 D. Any previous vaginal delivery, either before or following a cesarean birth, significantly improves the prognosis for a subsequent vaginal delivery with either spontaneous or induced labor.

A 16 year-old presents with vaginal bleeding for two days. She estimates her last menstrual period to be 3 months ago. Pregnany test is positive. There is crampy right lower quadrant abdominal pain. On IE, the cervical os is closed but with adnexal tenderness on the right. The abdomen is positive for direct and rebound tenderness. Vital signs are BP 80/50, HR 105, RR 22, afebrile. What is the proper management? A. Laparotomy B. Perform TVS C. D&C D. Do serial b-HCG E. Discharge patient

212 A. This is a case of ectopic pregnancy. Patient is hemodynamically unstable. Proceed to surgery.

Women with the Rokitansky-Kuster-Hauser syndrome have an increased risk of developing abnormalities of what other organ system? A. Cardiovascular B. Gastrointestinal C. Renal D. Endocrine E. Nervous system

213 C. RKH syndome is congenital uterine agenesis. Congenital renal abnormalities occur in about one third of these individuals and skeletal abnormalities in about 12%.

This drug achieves “pseudomenopause†among patients with endometriosis. A. Estrogen B. Clomiphene C. Leuprolide D. Danazol E. Goserelin

214 D. Danazol produces a hypoestrogenic and hyperandrogenic effect on steroid-sensitive end organs.

In cervical carcinoma, involvement of the kidneys automatically places the condition at stage ______. A. IIA B. IIB C. IIIA D. IIIB E. IVA

215 D.

A four-year old female patient came in due to vulvar irching. On physical examination, you noted the labia minora to be fused with a translucent vertical midline line. This condition usually affects young girls between 2 and 6 years of age because of: A. Poor hygiene B. Hormonal factors C. Bacterial infection D. Genital trauma E. Congenital defect

216 B. This is adhesive vulvitis. Estrogen reaches a nadir during this time, predisposing the nonestrogenized labia to denudation.

Which of the following is a hormonal correlate of the hot flush of menopause? A. Low estrogen B. Low progesterone C. High FSH D. High LH E. Low ACTH

217 D. Hormonal correlates of flush activity: Increased LH, ACTH, B-endorphin. However, these occurences are thought to be epiphenomena that result as a consequence of the flush and not related to its cause.

A 23 year-old G1P0 at 12 weeks AOG came in due to low-grade fever for three days accompanied by a diffuse maculopapular rash that began on the face and has now spread to the trunk and extremities. On physical examination, she has prominent occipital and post-auricular lymphadenopathy. What is the single most common defect that her fetus might develop? A. Cataract B. Patent ductus arteriosus C. Sensorineural deafness D. Microcephaly E. Mental retardation

218 C. This is a possible case of congenital rubella syndrome.

A 24 year-old G1P0 at 15 weeks AOG came in for her first prenatal check up. She has no subjective complaints. Her BMI is 30. She has no family history of diabetes. When should you screen for diabetes in this patient? A. No need to screen. She is at low risk of developing diabetes. B. Screen her at 20-24 weeks AOG. C. Screen her at 24-28 weeks AOG. D. Screen her at 28-32 weeks AOG. E. Perform screening now.

219 E. High risk patients should be screened as soon as feasbile if one or more of the following are present: severe obesity, strong family history of type 2 diabetes, previous history of GDM, impaired glucose metabolism or glucosuria.

You are treating a 35 year-old female with syphilis. She is concerned whether or not her partner should be treated. You tell her that in syphilis, individuals who are exposed within the ____ days preceding the diagnosis in their sexual partners should be treated presumptively because they may be infected even when seronegative. A. 30 days B. 60 days C. 90 days D. 120 days E. 180 days

220 C.

A 34 y/o G1P1 suddenly developed lack of menstruation for the past 6 months. There were also growing of facial hairs and clitoral enlargement. What type of ovaian tumor should be considered? A. Granulosa theca cell tumor B. thecoma C. dysgerminoma D. sertoli leydig tumor E. none of the above

240 D. sertoli leydig cell tumors exhibit production of testosterone which is responsible for the amenorhea, hirsutism and clitoral enlargement of the patient

A 30 y/o G2P1 on her 18th week AOG came to you for her prenatal check up. She is asymptomatic, and PE revealed BP of 150/90, HR 80, RR 19. The rest of her PE was otherwise unremarkable. She claims that this was the first time that she had an elevated BP. What is your initial diagnosis?? A. Gestational htn B. Chronic htn C. Preeclampsia D. Superimposed preeclampsia on chronic htn E. none of the above

221 B. Chronic htn is BP greater than or equal to 140/90 mm Hg before pregnancy or diagnosed before 20 weeks’ gestation not attributable to gestational trophoblastic disease or Hypertension first diagnosed after 20 weeks’ gestation and persistent after 12 weeks postpartum.

Which of the following can be observed at 12 weeks AOG?? A. CRL of 12 cm B. Gender can be determined C. Center of ossification have appeared in most of the fetal bones D. All of the above E. None of the above

222 C. . The CRL of 12 cm is seen at 16 weeks AOG, while gender can be determined by experienced sonologist at 14 weeks AOG

40 y/o G6P5 32-33 weeks AOG complains of vaginal bleeding and severe abdominal pain. VS BP 160/100 PR 123 RR 24 T 37.2C. PE of the abdomen:FH 36cm FH 97bpm, UC: every minute, 50 sec duration, strong. what complication is associated with the above condition? A. placenta accreta B. uterine atony C. DIC D. puerperial infection E. all of the above

223 C. abruptio placenta is associated with DIC, renal failure (ATN), hemorrhagic shock, couvelaire uteurs, and fetal maternal hemorrhage.

What is the type of placental expulsion, wherein the placenta separates first from the periphery? A. Schultze mechanism B. Duncan mechanism C. McRobert mechanism D. Simpson mechanism E. Rubins mechanism

224 B. Duncan mechanism â€" placenta separates from the periphery first. As a result, blood collects between the membranes and the uterine wall and escapes from the vagina. In this circumstance, the placenta descends sideways, and the maternal surface appears first. Schultze mechanism â€" placenta separates from the center first. Blood from the placental site pours into the membrane sac and does not escape externally until after extrusion of the placenta

Which of the following is an absolute contraindication for aerobic exercise during pregnancy? A. Restrictive lung disease B. Severe anemia C. Chronic bronchitis D. Poorly controlled hypertension E. All of the above

225 A. Absolute contraindication include:Hemodynamically significant heart disease, Restrictive lung disease, Incompetent cervix/cerclage, Multifetal gestation at risk for preterm labor, Persistent second- or third- trimester bleeding, Placenta previa after 26 weeks, Preterm labor during the current pregnancy, Ruptured membranes, Preeclampsia/pregnancy- induced hypertension. Relative Contraindications include: Severe anemia, Unevaluated maternal cardiac arrhythmia, Chronic bronchitis, Poorly controlled type 1 diabetes, Extreme morbid obesity, Extreme underweight (BMI _12), History of extremely sedentary lifestyle, Fetal-growth restriction in current pregnancy, Poorly controlled hypertension, Orthopedic limitations, Poorly controlled seizure disorder, Poorly controlled hyperthyroidism, Heavy smoker

A 20 y/o G1P0 at 40 weeks AOG came to your hospital. She denied of having any prenatal check up. She then subsequently gave birth to a baby boy. Upon delivery, you noted that the baby had upturned nose, mild midfacial hypoplasia, and long upper lip with thin vermilion border, distal digital hypoplasia. What medication could the mother have been taking during the course of her pregnancy?? A. ACE inhibitor B. Phenytoin C. Warfarin D. Alcohol E. Valproic acid

226 B. This is the characteristic of a baby with fetal hydantoin syndrome.

A 28 y/o G2P1 mother at 35 week AOG came to your clinic with the following biophysical profile scoring: NST has 3 accelerations greater 15 bpm each for more than 15 seconds, 20 seconds of fetal breathing for 30 minutes, 2 discrete body movements, 1 episode of extremity extension then subsequent flexion. AFI 1.2cm. What is your next plan of treatment? A. No fetal indication for intervention. Repeat test weekly B. Deliver C. Repeat BPP D. Observe E. none of the above

227 C. BPP=4 (NST = 2, fetal breathing = 0, fetal movements = 0, fetal tone = 2, AFI =0). For a BPP of 4, there is probable fetal asphyxia. The intervention is to repeat BPP on the same day. If the result if less than or equal to 6, deliver the baby.

The following is a correct criteria for outlet forceps extraction? A. Scalp is visible at the introitus after separating the labia B. Fetal skull has reached pelvic floor C. Sagittal suture is in transverse diameter or right or left occiput anterior or posterior position D. Rotation does not exceed 60 degrees E. Station +2

228 B. The following are the criteria for outlet forcerps extraction: 1. Scalp is visible at the introitus without separating the labia, 2. Fetal skull has reached pelvic floor,3. Sagittal suture is in anteroposterior diameter or right or left occiput anterior or posterior position, 4. Fetal head is at or on perineum, and 5. Rotation does not exceed 45 degrees

What type of placenta is characterized by fetal membranes completely covered by functioning villi?? A. Placenta duplex B. Placenta fenestrata C. Placenta membranacea D. Succenturiate lobe placenta E. Battledore placenta

229 C. Placental membranacea is characterized by all or a large part of the fetal membranes are covered byfunctioning villi. Placenta membranacea may occasionally give rise to serious hemorrhage because of associated placenta previa or accrete. Placenta duplex, or bilobed placenta is characterized by the placenta that forms as separate, near equally sized disks. The cord inserts between the two placental lobesâ€"either into a connecting chorionic bridge or into intervening membranes. Placenta fenestrate is characterized by the missing central portion of a discoidal placenta. In some instances, there is an actual hole in the placenta,but more often, the defect involves only villous tissue, and the chorionic plate remains intact. Succenturiate lobe placenta is similar to placenta duplex, but smaller. One or more small accessory lobes develop in the membranes at a distance from the main placenta, to which they usually have vascular connections of fetal origin.

How many weeks will ovulation resume after delivery?? A. 3 weeks B. 4 weeks C. 3 months D. 4 months E. 2 weeks

230 A. Ovulation may resume as early as 3 weeks after delivery, even in lactating women.

All of the following are the correct pairs of embryonic structures and its derivatives, except? A. Gubernaculum: broad ligament of the uterus B. Metanephric duct: renal pelvis C. Urogenital sinus : greater vestibular glands D. Mesonephric duct: Gartner’s duct E. none of the above

231 A. Gubernaculum gives rise to round ligament of the uterus.

In what cell stage does the ovum in during its release from the ovary ? A. Prophase I B. Metaphase I C. Prophase II D. Metaphase II E. Anaphase I

232 D. The egg is released from the the ovary at its metaphase II

A 27 y/o G0 patient came to your clinic due to severe dysmenorrhea. She is married for 3 years, and is not able to conceive. Upon IE, you noted that the uterus is fixed and retroverted. There is nodularity on the uterosacral ligament. What is the definitive procedure of choice for the diagnosis? A. Pelvic ultrasound B. Fractional curettage C. Direct visualization and biopsy D. culdocentesis E. MRI

233 C. This patient presents with a classic case of endometriosis. The definitive diagnostic procedure of choice is direct visualization of the endometriotic lesion and pathologic examination via biopsy.

5. A 41 y/o came to your clinic due to profuse whitish vaginal discharge. Upon IE, you noted that the discharge was whitish frothy appearance with fishy amine odor. On wet mount, there are numerous clue cells. What is your next plan of management? A. 1% Clotrimazole cream, 5g for 7-14 days B. Metronidazole, 2 g, one dose C. Metronidazole, 500mg, BID for 7 days D. Miconazole, 100mg per vagina for 7 days E. Clindamycin 500mg/tab BID

234 C. This patient presents with bacterial vaginosis. Clue cells are pathognomonic for bacterial vaginosis. The drug of choice for this condition is metronidazole, 500mg, BID for 7 days. B - is for trichomonas infection

Which of the following is true for screening for cervical cancer? A. Initial screening should begin within 1 year after first sexual intercourse or by age 18 B. Annual screening until 30 y/o C. After 30 y/o, if patient has 3 consecutive negative results, can discontinue screening D. Women with their cervix removed for benign reasons need annual screening E. all of the above

235 B. The following are ACOG guidelines for screening for cervical cancer: Initial screening should begin within 3 years after first sexual intercourse or by age 21; annual screening until 30 y/o; after 30 y/o, if patient has 3 consecutive negative results, repeat every 2-3 years; women with their cervix removed for benign reasons do not need annual screening

Which of the following characterizes type 2 endometrial carcinoma? A. Induced by estrogen B. Post menopausal C. Responds to progestins D. Good prognosis E. All of the above

236 B. Type 1 endometrial carcinoma - Grade 1â€"2, Superficial or no myometrial invasion, Coexisting endometrial hyperplasia, Perimenopausal, Induced by estrogen, Responds to progestins, Good prognosis; Type 2 - High grade (grade 3, papillary serous, clear cell), Deep myometrial invasion, Postmenopausal women, Not induced by estrogen, Does not respond to progestins, Poor prognosis

What type of stress incontinence is characterized by involuntary leakage of urine during increased abdominal pressure in the absence of detrusor contraction during urodynamics?? A. Genuine stress incontinence B. Urge incontinence C. Functional incontinence D. Total urinary incontinence E.

237 A. Stress urinary incontinence is involuntary leakage of urine on effort or exertion or on coughing or sneezing. The sign of stress urinary incontinence must occur at the start of coughing and end with the coughing. Urge incontinence is the strong desire to urinate. The patient may sense a degree of urgency if voiding occurs despite the desire not to void. Functional incontinence is involuntary leakage despite normal bladder and urethral function in patients with dementia or disorientation. Total urinary incontinence occurs when the bladder is unable to store urine because the resting urethral pressure is so low that no resistance is present for the passage of urine.

A 68 y/o patient came in your clinic due to intense vulvar pruritus. Upon physical examination, you noted that the vulva is thin, white, wrinkled tissue with a cigarette paper appearance. There was agglutination of the labia minora and prepuce. Biopsy revealed hyperkeratotic epidermis with epithelial atrophy, and flattening of the rete pegs. There is cytoplasmic vacuolization of the basal layer of cells. What is your initial diagnosis? A. Lichen simplex chronicus B. Lichen sclerosus C. Lichen planus D. Squamous cell hyperplasia E.

238 B. This is a case of lichen sclerosus. Lichen simplex chronicus is characterized by thickened white epithelium on vulva. It is generally localized and unilateral. Biopsy reveals hyperkeratosis and acanthosis producing thickening of the epithelium and elongation of the rete pegs. Dermis reveals chronic inflammatory cells, fibrosis, and collagenization. Lichen planus is characterized by erosive lesions at the vestibule with or without vaginal synechiae resulting in stenosis. Biopsy reveals mils, localized, lichenoid, chronic inflammatory process at the epidermal-dermis junction to ulcerative process with fibrosis. Squamous cell hyperplasia is circumscribed, single or multifocal raised white lesions on the vulva or adjacent tissue. Biopsy reveals hyperkeratosis and acanthosis, producing thickening of the epithelium and elongation of the rete pegs. Dermis has no inflammatory infiltrate.

A 37 y/o G1P1 (1001) consulted because of severe dysmenorrhea for 3 consecutive years. Initial consult revealed bilateral adnexal masses about 7cm each with thickened uterosacral ligament. If surgery is an option, what is the most appropriate procedure for this patient? A. TAHBSO B. bilateral oophorocystectomy C. fluid aspiration of the bilateral mass D. no surgical procedure is warranted. E.

239 B. patient is still in a reproductive age. The goal is to remove the mass and to preserve as much as functional ovarian tissue.

A 17 year old female was diagnosed with primary amenorrhea, further work up revealed absent uterus but with normally developed breasts, normal looking external genitalia and pubic hair, which of the following conditions is implicated? A. turner's syndrome B. meyer rokitasky kuster hauser C. androgen insensitivity D. congenital adrenal hyperplasia E. none of the above

241 B. SIMILAR TO PREVIOUS BOARD EXAM CONCEPT/PRINCIPLE., meyer rokitansky has normal ovaries, normal karyotype, normal secodnary sexual char no uterus and sometimes no vagina or atrophic

A 24 y/o G2P1 (1001) was admitted due to passage of watery discharge and onset of regular abdominal cramps. Initial IE was 4cm already station -2, 4 hours later patient was 7cm dilated station -2, contractions came in regularly every 3- 5minutes lasting 50-60 seconds, which of the following actions are appropriate? A. augment labor with oxytocin B. observe progress of labor C. prepare patient for emergency CS D. give patient epidural anesthesia E. hydrate and place patient on left lateral decubitus

242 B. SIMILAR TO PREVIOUS BOARD EXAM CONCEPT/PRINCIPLE, monitor futher progress of labor first before intervention

Mother with preeclampsia was placed on a tocodynamometer and fetal heart monitor during labor, the resident suddenly remarked that there were late decelerations noted on the tracings, what does it connote? A. fetal head compression B. uteroplacental insufficiency C. umbilical cord compression D. placental abruptio E. impending eclampsia

243 B. SIMILAR TO PREVIOUS BOARD EXAM CONCEPT/PRINCIPLE, head compression early, cord compression variable

A 60 year old female complains of scanty vaginal spotting for the past 2 months, physical examination revealed thinned out vaginal mucosa, smooth cervix, uterus small with no palpable adnexal masses, which of the following most likely is the cause of A. endometrial polyp B. cervical polyp C. endometrial cancer D. atrophic vaginitis E. submucous myoma

244 D. SIMILAR TO PREVIOUS BOARD EXAM CONCEPT/PRINCIPLE, atrophic vaginitis, postmenopausal

A 30 year old female G5P1 with history of repeated abortions and preterm labor is pregnant, her doctor tells her to undergo a cerclage procedure to help prevent preterm labor, this procedure is generally done at what age of the pregnancy? A. 8 weeks B. 12 weeks C. 22 weeks D. 24 weeks E. 36 weeks

245 C. SIMILAR TO PREVIOUS BOARD EXAM CONCEPT/PRINCIPLE, usually done at the age of viability at around 20 weeks and above

A 40 year old female patient has depressed TSH levels with Increased T4 levels, she experienced palpitations, weight loss, heat intolerance and bouts of diarrhea, which of the following is expected regarding her menses? A. hypomenorrhea B. menorrhagia C. menometrorrhagia D. primary amenorrhea E. any of the above

246 A. SIMILAR TO PREVIOUS BOARD EXAM CONCEPT/PRINCIPLE, hyperthyoridism = hypo or and oligomenorrhea

A 16 year old patient comes in with grossly female genitalia, no menses ever since but with positive breast development, further work up revealed absence of ovaries, uterus, karyotyping revealed 46 XY, which of the following is the possible diagnoses A. meyer rokitansky kuster hauser B. androgen insensitivity syndrome C. turners syndrome D. congenital adrenal hyperplasia E. none of the above

247 B. SIMILAR TO PREVIOUS BOARD EXAM CONCEPT/PRINCIPLE, they mixed up the meyer rokitansky, androgen insensitivity, turners and CAH, androgen insensitivity = grossly female or phenotypically female genitalia and secondary sexual char but XY karyotype. Turners XO, meyer rokitansky XX, CAH = XX with virilization or XY with normal phenotype

A 36 year old female patient with previously regular menses consulted due to amenorrhea for 3 months with episodes of spotting occasionally, pregnancy test was negative, she was given progesterone for 7 days and after 3-5 days of withdrawal of the drug, she had heavy menses. Which of the following is true regarding her condition? A. Her irregular menses are due to a submucous myoma B. she has ectopic pregnancy C. she has endometrial hyperplasia D. she has anovulatory vaginal spotting E. she has an endometrial polyp

248 D. SIMILAR TO PREVIOUS BOARD EXAM CONCEPT/PRINCIPLE, progesterone induced withdrawal bleed indicates endometrium is estrogen primed, lack of ovulation causes it to slough off improperly due to lack of progesterone

Which of the following ultrasound findings are indicative of down's syndrome in the fetus ? A. increased nuchal thickening B. increase brain ventricle size C. ambiguous genitalia D. small biparietal diameter for AOG E. all of the above

249 A. SIMILAR TO PREVIOUS BOARD EXAM CONCEPT/PRINCIPLE, nuchal thickening is indicative of down's syndrome

Which of the following trisomies are associated with preeclampsia? A. trisomy 13 B. trisomy 15 C. trisomy 18 D. trisomy 21 E. none of the above

250 A. SIMILAR TO PREVIOUS BOARD EXAM CONCEPT/PRINCIPLE, trisomy 13 preeclampsia

The most common symptom in a patient with cervical cancer: A. dyspareunia B. vaginal bleeding C. foul vaginal discharge D. weight loss E. pruritus

336 B. Berek & Novaks Gynecology 14th edition p.1404

An 8 year old female was brought into consult due to difficulty in perineal hygiene, her mother noted that her labia was adherent to one another and was concerned about it hence the consult, which of the following treatments is advisable for this case? A. oral estrogen pills B. topical estrogen cream C. topical 1% hydrocortisone cream D. surgical adhesiolysis E. all of the above

251 B. SIMILAR TO PREVIOUS BOARD EXAM CONCEPT/PRINCIPLE a case of adhesive vulvitis, estrogen topical

Which of the following is considered the most important indicator of progress of labor? A. engagement B. descent C. cervical dilatation D. internal rotation E. all of the above

252 B. SIMILAR TO PREVIOUS BOARD EXAM CONCEPT/PRINCIPLE, descent according to APMC book, descent of the baby infers progress of labor and interaction between the passenger and passageway

Which of the following tests can be used to infer regarding the risk of preterm labor? A. fetal HCG B. D Dimer assay C. fetal fibronectin D. maternal CRP E. amniotic fluid AFP

253 C. SIMILAR TO PREVIOUS BOARD EXAM CONCEPT/PRINCIPLE, fetal fibronectin helps infer regarding risk of preterm labor

On instances of complete breach extraction, which of the following forceps is used to deliver the aftercoming head? A. simpsons B. tucker mcclane C. kielland D. pipers E. none of the above

254 D. SIMILAR TO PREVIOUS BOARD EXAM CONCEPT/PRINCIPLE - pipers aftercoming head

A 24 year old G3P3 desires contraception. In giving your advice regarding the various options of birth control, which of the following is a contraindication to placement of an IUD for family planning? A. presence of menstruation B. presence of previous ectopic pregnancy C. no consent from the husband D. presence of ongoing pelvic infection E. all of the above

255 D. SIMILAR TO PREVIOUS BOARD EXAM CONCEPT/PRINCIPLE - husband consent not needed, even in menstruation no contraindication

48 year old G3P3 presents to your clinic for annual check up, she has had no remarkable symptoms since her last check up, you decided to do a pap smear, a week after the results came out, it turned out to be high grade squamous intraepithelial neoplasm, you remember that physical examination including pelvic exam at that time was unremarkable what is the next step in evaluation? A. request a transvaginal ultrasound B. place her on progesterone and wait for withdrawal bleed C. do a repeat pap smear using cytobrush and liquid based cytology for confirmation D. do a colposcopy E. all of the above

256 D. SIMILAR TO PREVIOUS BOARD EXAM CONCEPT/PRINCIPLE, colposcopy with acetowhitening or biopsy of suspicious lesion

A 30 year old female G1P1 presented to the clinic with a 2 year history of progressive virilization, facial and bodily hair growth, oligomenorrhea and irregular menses, there was also acne, deepening of voice, examination revealed grossly female genitalia, clitoromegaly, and a palpable right adnexal mass. Ultrasound revealed right complex adnexal mass, which of the following is the possible condition? A. leydig cell tumor B. granulosa cell tumor C. yolk sac tumor D. choriocarcinoma E. dermoid cyst of the ovary

257 A. SIMILAR TO PREVIOUS BOARD EXAM CONCEPT/PRINCIPLE, adnexal mass, virilization and masculinization features leydig cell tumor

Which of the following risk factors is considered the greatest risk factor for ectopic pregnancy? A. repeated pelvic infections or PID B. history of tubal instrumentation C. infertility D. congenital uterine abnormalities E. familial chromosomal disorders

258 B. SIMILAR TO PREVIOUS BOARD EXAM CONCEPT/PRINCIPLE, tubal instrumentation is the strongest risk factor for having ectopic pregnancy, infection comes next

A 26 year old G1P0 was admitted to the labor room due to rupture of bag of waters without accompanying uterine contractions, after 18 hours, on monitoring, mother was slightly febrile at 37.8 degrees celsius and was continuously hydrated with IV fluids, electronic fetal monitoring was done which revealed fetal tachycardia, which of the following is the suspected condition? A. chorioamnionitis B. umblical cord prolapse C. maternal dehydration D. maternal sepsis E. endometritis

259 A. SIMILAR TO PREVIOUS BOARD EXAM CONCEPT/PRINCIPLE, fetal tachycardia = chorioamnionitis

A 35 year old female patient G1P1 came in due to dysmenorrhea and vaginal spotting for the past 3 weeks, PE revealed grossly female external genitalia, no lesions noted, cervix pink smooth, no discharge, cervix closed, uterus symmetrically enlarged to 2 to3 months size, no adnexal masses noted, which of the following conditions is suspected? A. endometriosis B. endometrial hyperplasia C. multiple submucous myomas D. adenomyosis E. endometrial polyp

260 D. SIMILAR TO PREVIOUS BOARD EXAM CONCEPT/PRINCIPLE - adenomyosis symmetrically enlarged uterus dysfunctional bleeding, dysmenorrhea

A 35 year old woman is complaining of increasingly severe mentrual pain with associated heavy and prolonged bleeding. Pelvic examination reveals a diffusely enlarged uterus that feels soft and boggy. What is the pathogenesis underlying the most likely condition that is causing this patient's symptoms? A. Presence of hormone-responsive endometrial tissue in the pelvic peritoneum B. Benign submucous myomatous proliferation C. Atypical endometrial hyperplasia D. Presence of endometrial glands within the myometrium E. None of the above

261 D. Characteristic case of adenomyosis

A sexually active 28 year old female had her first Pap smear revealing low-grade squamous intraepithelial lesion (LSIL). What will be the next step in your management? A. Hysterectomy B. LEEP procedure C. HPV testing D. Repeat Pap smear after 6 months E. Colposcopy-directed biopsy

262. E..ASC-H, LSIL, HSIL - Perform colposcopy with biosy; ASC-US - perform HPV testing or repeat Pap smear after 6 months

A G1P0 preganant comes for prenatal consult in your clinic. History reveals that she takes vitamin supplements because she is afraid of getting sick. You counsel her to avoid taking which of the following? A. Vitamin C B. Vitamin A C. Beta carotene D. Vitamin E E. Folic acid

263 B. Vitamin A megadoses is teratogenic. Beta carotene- rich foods has lesser vitamin A content and is less likely to cause teratogenesis.

Which of the following findings is expected in the cervical cytology of a menopausal woman? A. Parabasal cells B. Superficial cells C. Intermediate cells D. Superficial and intermediate cells E. All of the above

264 A. Menopausal - parabasal; Reproductive nonpregnant - superficial and intermediate; Pregnant - intermediate

A G2P2 patient is in labor undergoes pelvic examination revealing vesicular lesions in the genital area. You are highly suspecting a herpes simplex infection. How will you manage this patient? A. Treat the mother with IV acyclovir and deliever via SVD B. Administer oxytocin to accelerate labor C. Perform cesarean delivery D. Perform forceps extraction when fully dilated E. Continue labor watch.

265 C. Pergnant patients with herpes simplex genital lesions should have cesarean delivery to lessen risk of transmission to newborn

When is the usual onset of quickening among primigravids? A. 12-14 weeks AOG B. 13-15 weeks AOG C. 16-18 weeks AOG D. 18-20 weeks AOG E. 22-24 weeks AOG

291 D. Quickening among multi: 16-18 weeks, for primi: 18-20 weeks

A 30 year G3P3 patient gave birth to a live baby girl 2 days prior via NSVD. Currently, she claims that she feels febrile. Temperature taken was 37.9 degrees Celsius. Bimanual pelvic examination reveals a tender uterus and foul-smelling vaginal discharge. You will most likely isolate which of the following? A. Bacteroides fragilis B. Chlamydia trachomatis C. Neisseria gonorrheae D. Pseudomonas aeruginosa E. Gardnerella vaginalis

266 A. A case of postpartum endometritis. Most are caused by mixed aerobic and anaerobic microorganisms but Bacteroides is frequently isolated in culture

What will be the expected results of a Quad screen in a mother carrying a fetus with trisomy 21? A. Decreased B-HCG and inhibin, Elevated estriol and AFP B. Elevated B-HCG and inhibin, Decreased estriol and AFP C. Elevated B-HCG and inhibin, Elevated estriol and AFP D. Decreased B-HCG and inhibin, Decreased estriol and AFP E. None of THeses

267 B. Elevated AFP - neural tube defects; Decreased AFP, HCG, estriol - trisomy 18

A 34 year old patient comes to your clinic complaining of heavy menses. She claims that she occasionally pass clots and she feels a heavy sensation and a palpable mass on the lower abdominal area. Which of the following is the most likely diagnosis? A. Ovarian cyst B. Endometrial carcinoma C. Endometriosis D. Endometrioma E. Leiomyoma

268 E. Characteristic picture of a case of leiomyoma which is a common cause of heavy menstrual bleeding esp in premenopausal women

An incidental finding of a 6cm multilocular ovarian cyst with solid components was seen during ultrasound in a 30 year old woman. History is unremarkable. What is the next best step in the management? A. Observe for 2-3 months and repeat ultrasound B. Perform percutaneous biopsy C. OCP therapy D. Exploratory laparotomy E. Do nothing.

269 D. Simple ovarian cysts may be managed with observation except if size is >8cm, if 6-8 cm but doesn’t resolve after 2-3 months, if there is rapid increase in size, or if multiloculated or with solid components. These are suspicous for malignancy and have to be treated with surgery

A 38 weeks AOG G3P2 mother who has chronic hypertension presents to the ER due to vaginal bleeding associated abdominal pain and uterine contractions. Physical examination reveals a firm, tender uterus with noted regular contractions every 4-5 minutes and cervical dilatation of 4-5cm. Bleeding has subsided and vital signs of the mother is stable. Fetal monitoring is reassuring. Ultrasound reveals a high-lying placenta. How should this patient be managed? A. Stat cesarean delivery B. Give tocolytics C. Monitor progress of labor and do fetal status monitoring D. Send the patient home E. None of the above

270 C. A case of abruptio placenta. May be managed conservatively if vital signs are stable or fetal status is reassuring. If not, immediate delivery is necessary

A G2P0 patient has a previous history of incompetent cervix. If she is to undergo an elective cervical cerclage, at what age of gestation is the ideal time to perform the procedure? A. 8-10 weeks B. 10-12 weeks C. 12-14 weeks D. 18-20 weeks E. 24-28 weeks

271 C.

Examination of a woman reveals presence of fleshy growths covered with small, papillary surface prijections on her external genitalia. What is the most likely diagnosis? A. Condyloma acuminata B. Condyloma lata C. Genital herpes D. Phthirus pubis E. Molluscum contagiosum

272 A. Descriptive of condyloma acuminata/genital wart

A 16 year old female is brought to you by her mother because of amenorrhea. Upon physical examination, patient was noted to have a BMI of 16.1. Dry skin, thinning hair, parotid glnad swelling, and fine body hair were also noted. She tells you that she has not attained her ideal body weight and that she thinks she is fat. Based from these findings, you expect that the amenorrhea is caused by? A. Hyperprolactinemia B. Hypothalamic dysfunction C. Anterior pituitary hormone deficiency D. Increased androgen production E. Anovulatory disorder

273 B. Anorexia nervosa may cause hypothalamic dysfunction leading to imapired release of Gonadotropin releasing hormone

Which of the following is most predictive of recurrent preterm labor? A. Group B streptococcus colonization B. Prior preterm delivery C. History of cigarette smoking D. History of early trimester abortion E. None of the above

274 B.

A patient presents to the ER complaining of amenorrhea, lower abdominal pain, and vaginal bleeding. Pregnancy test is positive. Vital signs are stable. Physical examination reveals a uterus that is small for gestational age, closed cervix and bleeding. No adnexal mass is palpated. TVS fails to reveal an intrauterine pregnancy. Which of the following is the most appropriate management? A. Send the patien home. B. Admit the patient and prepare for dilatation and curettage. C. Repeat TVS when B-HCG level is >1500 D. Perform emergency exploratory laparotomy E. Admit the patient for therapeutic abortion

275 C. Equivocal findings necessitate repeat TVS. Absence of Intrauterine pregnancy at B-HCG levels of >1500 or failure of BHCG to double after 48 hours rules out uterine pregnancy

A 23 year old obese patient has been diagnosed with Polycystic ovarian syndrome. You, therefore, expect decreased levels of which of the following? A. FSH B. LH C. Estrogen D. Androstenedione E. Testosterone

276 A. Increased LH to FSH ratio is characteristic of PCOS. FSH is decreased while estrogen, androgen, and LH are increased

Physiologic changes during pregnancy will include which of the following? A. Increase in systemic vascular resistance B. Increase in tidal volume C. Decrease in hematocrit D. A and B E. B and C

277 E. There is an increase in tidal volume, cardiac output, GFR, and plasma volume. There is a decrease in SVR and hamatocrit

Which of the following cardinal movements of labor allows the narrowest fetal head diameter to present in the birth canal? A. Flexion B. Extension C. Internal Rotation D. Engagement E. Descent

278 A. Flexion allows the narrowest diameter (suboccipitobregmatic diameter) to present. Review cardinal movements of labor

A 30 year old patient who is desirious of pregnancy presented to your infertility clinic with associated symptoms of dysmenorrhea and dyspareunia undergoes fertility evaluation. You are highly suspecting endometriosis as the cause. How will you manage this patient? A. NSAIDS B. Continuous OCP's C. Watchful waiting D. Laparoscopic surgery and fulguration of endometriotic implants E. Danazol therapy

279 D. Endometriosis may be managed medically but if desire for fertility is an issue then treatment should be surgery

A 19 year old G1P0, 14-15 weeks AOG, consulted due to vaginal bleeding. Vital signs were stable. Fundic height was noted at 23 cm. Internal exam: cervix is soft, long and closed with uterus enlarged to 5 months size. UTZ showed a complex, echogenic mass with numerous cystic spaces with no fetus and aminotic sac seen. Serum Beta HCG is elevated at 170,000 miU/mL. What is the most appropriate management for this case? A. TAH B. TAHBSO C. Radiotherapy D. Suction Curettage E. Any of the above

281 D. Dx: H. mole. Treatment of choice females <35 years old and still desires pregnancy is suction curettage.

What is the maximum blood loss (in mL) to be considered in a normal menstrual cycle? A. 30 B. 40 C. 60 D. 70 E. 80

282 E. Menorrhagia: Prolonged (more than 7 days) or excessive (greater than 80 mL) uterine bleeding occurring at regular intervals.

A 27 year old mulitpara who delivered at home was brough to the ER due to profuse vaginal bleeding. At the ER, BP was 80/50 and HR was 126 bpm. She was visibly pale. Abdomen was soft and the fundus of the uterus was not appreciated. On inspection of the vulva, a pear shaped reddish pink mass was see protruding from the introitus. What is the most common cause of the above condition? A. Abruptio placenta B. Placenta previa C. Strong traction of the umbilical cord D. Short umbilical cord E. None of the above

283 C. DX: Uterine inversion which is most commonly brough about by excessive traction of the umbilical cord upon delivery of the placenta

Which of the following is not an indication for in- patient treatment of patients with Pelvic Inflammatory Disease? A. Acute appendicitis cannot be ruled out B. No response after 24 hours of treatment C. Cannot tolerate oral regimen D. Presence of tubo-ovarian abscess E. Currently pregnant

284 B. Other indications: severe illness, nausea or vomiting. Response to treatment is usually assessed after 3 days (72 hours)

A 38 year old G1P0, 30-31 weeks AOG with no known co-morbids, sought consult due to headache and blurring of vision. Patient had 8 prenatal checkups since 6 weeks AOG. BP monitoring was advised which ranges from 100/70 to 110/80 mmHg. She has a (+) family history for both HTN and DM. VS: BP 180/110 mmHg, PR 87/min, RR 21/min. FH 30 cm, FHT 120 bpm, LM1-cephalic. What is the most probable diagnosis for this case? A. Mild Preeclampsia B. Severe Preeclampsia C. Chronic hypertesion D. Preeclampsia superimposed on Chronic hypertension E. Eclampsia

285 B. In severe pre eclampsia: SBP >160, DBP >110. With proteinuria >5 g/24 hours, (+) headache, visual disturbance, upper abdominal pain, oliguria. Elevated crea. Thrombocytopenia

A 35 year old G4P4 (4004) underwent TAH-BSO with omentectomy, peritoneal washing and pelvic lymph node dissection for a right ovarian new growth. Histopathologic report stated that numerous hyaline droplets were seen. What specific type of ovarian tumor was removed from this patient? A. Yolk sac tumor B. Dysgerminoma C. Granulosa cell tumor D. Mucinous adenocarcinoma E. Sertoli Leydig cell tumor

287 A. Hyaline droplets represent the Schiller Duval bodies which are seen in Yolk sac tumor

What tumor marker would be characteristically elevated in association with the diagnosis in #187 ? A. HCG B. AFP C. LDH D. CA-125 E. CEA

288 B. The tumor marker associated with yolk sac tumor is Alpha feto protein (AFP)

A 40 year old G4P3 (3003), 31-32 weeks AOG, known hypertensive and a heavy smoker was brought to the ER due to severe hypogastric pain and vaginal bleeding. BP 140/90, HR 100/min, RR 21/min. FH 32 cm, FHT 90-100/min, LM1-cephalic, uterine contractions - every 2 minutes with a strong intensity, (+) uterine tenderness. What is the best management for this case? A. Corticosteroids alone B. Tocolysis and corticosteroids C. Labor induction D. Cesarean section E. None of the above

289 D. Dx: Abruptio placenta. Immediate delivery via CS is warranted due to evidence of fetal distress as noted by a decreased fetal heart rate ranging from 90-100 bom

Which of the following is the expected hormonal profile in individuals with Turner's syndrome? A. Hypogonadotropic hypogonadism B. Hypergonadotropic hypogonadism C. Hypogonadotropic normogonadism D. Hypergonadotropic normogonadism E. Normogonadotropic normogonadism

290 B. There is ovarian atresia resulting in depletion of oocytes in Turner's syndrome hence there is decreased estrogen secretion. However since the pituitary function is intact, the decreased estrogen will send feedback impulses to the pituitray increasing its output of FSH and LH

Which of the following is not a sign of midpelvis contraction? A. Prominent ischial spines B. Narrow subpubic arch C. Convergent sidewalls D. Shallow sacral concavity E. Narrow sacroiliac notch

294 B. A narrow subpubic arch is a sign of an inadequate pelvic outlet

A 38 year old G4P4 (4004) who is s/p LTCS due to PROM of 24 hours about 2 days ago now presents with fever and chills. BP 100/60, PR 110/min, RR23/min T 39.5 C. There is noted abdominal tenderness over the hypogastric area. Speculum: (+) foul smelling lochia rubra. IE: cervix soft, long and closed with (+) uterine tenderness. What is the single most important risk factor for this condition? A. Anemia B. Socioeconomic status C. Prolonged labor D. Route of delivery E. Malnutrition

295 D. Dx: Endometritis: Route of delivery is the single most important risk factor for this condition

What is the earliest stage of cervical cancer where in hydronephrosis, renal failure and uremia would be present? A. Stage IIIA B. Stage IIIB C. Stage IIA1 D. Stage IIA2 E. Stage IIB

296 B. Please see FIGO staging system for cervical CA. Of important note, uremia is the most common cause of death in cervical cancer patients.

A 37 year old G3P1 (1011), 39 weeks AOG, was admitted due to labor pains. VS are normal. FH 35 cm, FHT 145 bpm. IE: Cervix 3-4 cm dilated 70% effaced, ROT, st -2, (+) BOW. Labor admission test showed reassuring fetal heart rate pattern. 2 hours after a repeat IE was done: cervix 5 cm dilated 80% effaced, ROT, st -1, (+) BOW, FHT 140 bpm. After another 2 hours, a repeat IE was done: cervix 6 cm dilated, 90% effaced, ROA, st-1, (+) BOW, FHT 143 bpm. EFM tracing showed a reassuring fetal heart rate pattern with 190 Montevideo Units. How would you diagnose the labor pattern of the patient? A. Normal labor pattern B. Arrest in cervical dilatation C. Protracted cervical dilatation D. Prolonged latent phase E. Arrest in descent

297 C. Once in the active phase: rate of cervical dilatation should be 1.5 cm/hour for multiparas.

What would be the best approach for the problem in the previous case? A. None, it is a normal labor pattern B. Emergency CS C. Labor Induction D. Oxytocin Augmentation E. Early amniotomy

298 D. The patient is in the active phase and in cephalic presentation. Adequate Montevideo Units should be >200, hence we can augment labor with oxytocin

Which of the following is NOT an indication for classical cesarean section ? A. Anteriorly implanted placenta previa B. Multiple gestation C. Neglected transverse lie D. A large myoma in the lower uterine segment E. Invasive cervical carcinoma

300 B. Other indications: thinned out LUS, severely obese mothers w/c precludes safe access to LUS

what is the minimum age of gestation wherein fetal gender can be determined? A. 8 weeks B. 10 weeks C. 12 weeks D. 14 weeks E. 16 weeks

301 C. the external genitale are beginning to show definitve signs of male and female gender by the end of 12 weeks of gestation. Williams 22nd ed p93.

A 32 y/o G3P2 went for her 1st prenatal check up. Currently, she is at 18 weeks AOG. Upon doing routine pelvic ultrasound, there was an incidental finding of adnexal mass with solid and cystic structures measuring 13cm. What is the next best management for this case? A. observation with serial sonography B. immediate laparotomy C. laparotomy at 24th week AOG D. sonographic aspiration of the cyst E. laparotomy after delivery

310 B. it seems reasonable to remove all ovarian masses over 10cm because of the substantive risk of malignancy. Tumors from 6-10 cm should be carefully evaluated. For the possibility of neoplastic disease by ultrasound, MRI or both. If evaluation suggests a neoplasm, then resection is indicatied. if the corpus luteum is removed before 10 weeks, then 17OH progesterone IM is given weekly until 10 weeks. cystic mass that are benign with less than 6cm should undergo serial sonography. Williams 22nd ed pp 965-966.

A 68 y/o patient came in your clinic due to intense vulvar pruritus. Upon physical examination, you noted that the vulva is thin, white, wrinkled tissue with a cigarette paper appearance. There was agglutination of the labia minora and prepuce. Biopsy revealed hyperkeratotic epidermis with epithelial atrophy, and flattening of the rete pegs. There is cytoplasmic vacuolization of the basal layer of cells. What is your initial diagnosis? A. Lichen simplex chronicus B. Lichen sclerosus C. Lichen planus D. Squamous cell hyperplasia? E. none of the above

311 B. This is a case of lichen sclerosus. Lichen simplex chronicus is characterized by thickened white epithelium on vulva. It is generally localized and unilateral. Biopsy reveals hyperkeratosis and acanthosis producing thickening of the epithelium and elongation of the rete pegs. Dermis reveals chronic inflammatory cells, fibrosis, and collagenization. Lichen planus is characterized by erosive lesions at the vestibule with or without vaginal synechiae resulting in stenosis. Biopsy reveals mils, localized, lichenoid, chronic inflammatory process at the epidermal-dermis junction to ulcerative process with fibrosis. Squamous cell hyperplasia is circumscribed, single or multifocal raised white lesions on the vulva or adjacent tissue. Biopsy reveals hyperkeratosis and acanthosis, producing thickening of the epithelium and elongation of the rete pegs. Dermis has no inflammatory infiltrate.

A 25 y/o nullipara went for consult secondary to 7 years of irregular menses occuring every 2 to 3 months with menorrhaggia lasting for 8 days. Her BMI is 32kg/m2. BP 130/90. PE shows darkened skin flexures particularly in the dorsal neck. hairs are also noted in the upper lip, chin and anterior chest. which of the following is not needed as part of her work up? A. LH/FSH ratio B. TSH, T4 C. Testosterone D. 17-OH progesterone E. estradiol

313 A. exclusion of other causes of hyperandrogenism is needed before arriving at a diagnosis of PCOS. TSH, prolactin should also be checked. Testosterone levels >200ng/dl may suggest adrenal tumor. FSH and estradiol is requested for work up of POF. Although approximately 40-50% of PCOS patient have elevated LH/FSH ratio, it is not required for the diagnosis of PCOS.

A 24 y/o sexually active nullipara went for consult secondary to 4 days of fever and continuous lower abdominal pain. PE noted direct tenderness on (B) adnexal region without palpable mass. IE: (+) cervical motion tenderness. Speculum ex: hyperemic cervix with purulent discharge. pregnancy test is positive, urinalysis shows are within normal limits, CBC: neutrophilic leukocytosis. pelvic ultrasound is unremarkable. what is the next best step in terms of management ? A . IV clindamycin and gentamicin B. IV ampicillin and gentamicin C. IM spectinomycin and ceftriaxone D. levofloxacin and metronidazole E. Laparoscopy

314 A. patient is suffering from severe PID and at the same time, early pregnancy ( as noted by lack of UTZ findings with (+) hcg). The most appropriate treatment for this case is IV clindamycin and gentamicin. B, C- is ineffective since resistance is common in N gonnorrhea infection in the setting of PID. D- fluoroquinolone should never be given in pregnant patient. furthermore, this combination therapy is prescribed in milder cases of PID.

True about cervical cancer screening except? A. Pap testing should be discontinued in patients who underwent hysterectomy regardless of immune status B. Invasive cervical cancer is never been found in women less than 21 years old C. HPV DNA testing and Pap smear should be tested in women >30years old D. Fomite transmission can cause HPV infection. E. none of the above

315 A. A- recommendation indicates that continued Pap testing should be done on hysterectomized patients if she has a history of HSIL, immunocompromised or DES exposure in utero. D- this is the basis of screening at 21 years old even in the absence of sexual activity.

A 29 y/o G4P4 asked you about the best method of contraception. She is currently breastfeeding her 4 month old child and admitted to have a very tight schedule since she is a single parent. Patient is working both as a gym instructor and a fashion model. A. progesterone impregnated IUD B. progestin only pill C. Depot medroxyprogesterone acetate D. combined OCP E. any of the above

316 A. the patient who is currently on breastfeeding and has questionable compliance should be advised to have progesterone IUD as a form of contraception. Progesterone will not affect the quantity and quality of the milk in contrast to estrogen containing contraceptive methods.

Which of the following is true about dysgerminoma ? A. The median age of presentation is 45 years old B. Bilaterality is seen as high as 80% of the cases C. Analogous to seminoma in the male counterpart D. Approximately 90% of the tumors are highly differentiated and HcG producing. E. all of the above.

317 C. Dysgerminoma is the most common malignant germ cell tumors primarily occuring in women younger than 30 years old. Unlike other ovarian malignancy, bilaterality is more commonly seen in dysgerminoma with an estimate of around 10% of the cases. Only 10% of the tumor express HcG. ( SIMILAR TO PREVIOUS BOARD EXAM CONCEPT/PRINCIPLE). Compre Gyne 5th ed pa 867

Which of the following characterizes type 2 endometrial carcinoma? A. Induced by estrogen B. Post menopausal C. Responds to progestins D. Good prognosis E. all of the above

318 B. Type 1 endometrial carcinoma - Grade 1â€"2, Superficial or no myometrial invasion, Coexisting endometrial hyperplasia, Perimenopausal, Induced by estrogen, Responds to progestins, Good prognosis; Type 2 - High grade (grade 3, papillary serous, clear cell), Deep myometrial invasion, Postmenopausal women, Not induced by estrogen, Does not respond to progestins, Poor prognosis

what chromosomal abnormality accounts for the majority of spontaneous abortion? A. triploidy B. trisomy C. 45 XO D. translocation E. None of the above

319 B. ( SIMILAR TO PREVIOUS BOARD EXAM CONCEPT/PRINCIPLE). The most common abormality seen in spontaneous abortion is trisomy. While the most common single anomaly seen in abortion is 45XO.

What is the most important factor when considering a trial of labor after cesarean delivery? A. type of prior incision B. history of preterm cesarean delivery C. interdelivery interval D. maternal age E. maternal parity

324 A. Williams Obstetrics 23rd edition p.569

True regarding trial of labor after cesarean: A. Twin pregnancy may increase the risk of uterine rupture in VBAC. B. Obesity has no effect in the success of VBAC. C. The most favorable prognositc factor is prior vaginal delivery. D. A woman with a previous low segment rupture has a 16% recurrence risk. E. Women with prior vertical incision in the lower uterine segment without fundal extension are not candidates for VBAC.

325 C. Williams Obstetrics 23rd edition p.568

What is the most common manifestation of postabortal infection? A. septicemia B. endocarditis C. parametritis D. peritonitis E. endomyometritis

326 E. Williams Obstetrics 23rd edition p.222

Which confers the highest risk for ectopic pregnancy? A. infertility B. prior tubal damage C. multiple sexual partners D. previous genital infection E. intrauterine device use

327 B. Williams Obstetrics 23rd edition p.239

A physiologic change in pregnancy: A. transient loss of accommodation B. increase in serum creatinine C. decreased creatinine clearance D. tachypnea E. unchanged tidal volume

328 A. Respiratory rate is unchanged, creatinine is decreased, tidal volume is increased, and creatinine clearance is increased in pregnancy. Williams Obstetrics 23rd edition p.121-130

True regarding gestational diabetes: A. GD causes an increase in fetal anomalies. B. Fetus' brain is unaffected by macrosomia. C. Elective cesarean delivery was found to significantly decrease the incidence of brachial plexus injury. D. There is a 15% likelihood that women with gestational diabetes to develop overt diabetes mellitus within 20 years. E. There is a high risk of fetal death.

329 B. Fetal anomalies are not increased in GD. Elective cesarean delivery has no effect on the incidence of brachial plexus injury. There is a 50% likelihood that women with GD will develop overt DM within 20 years. Williams Obstetrics 23rd edition pp.1111-1112

Which satisfies one of the two clinical criteria for the diagnosis of antiphospholid antibody syndrome? A. >/= 1 unexplained fetal losses after 10 weeks B. >/= 2 consecutive miscarriages < 10 weeks C. preterm delivery for severe preeclampsia at 35 completed weeks D. preterm delivery for placental insufficiency at 35 completed weeks E. none of the above

330 A. Williams Obstetrics 23rd edition p.1153

ACOG recommends single dose treatment of antenatal steroids for women with preterm membrane rupture at an age of gestation of: A. 32-34 weeks B. <24 weeks C. 20-30 weeks D. 24-32 weeks

331 D. There is no concensus regarding giving antenatal steroids at 32-24 weeks. Steroids are not recommended at <24 weeks AOG. Williams Obstetrics 23rd edition p.820

Most frequently occurring congenital malformation in infants of women with overt diabetes mellitus: A. situs inversus B. anencephaly C. cardiac abnormalities D. rectal atresia E. caudal regression

332 E. Williams Obstetrics 23rd edition p.1114 table 52-8

True regarding should dystocia: A. McRoberts is a maneuver most likely to release an impacted shoulder. B. McRoberts is a reasonable approach when all other maneuvers fail. C. McRoberts requires 2 assistants to perform. D. all of the above E. A & B

333 C. There is no evidence that any one maneuver is superior to another in releasing an impacted shoulder. McRoberts is a reasonable INITIAL approach in shoulder dystocia. Williams Obstetrics 23rd edition p.485

A 40-year-old woman reports heavy and prolonged menstrual bleeding, associated with dysmenorrhea. On examination, the uterus is noted to be diffusely enlarged and soft, with its mobility not limited. There is no associated adnexal pathology. What is the most likely diagnosis? A. endometriosis B. ectopic pregnancy C. leiomyoma D. adenomyosis

334 D.

What is considered abnormal after the first two years from onset of menarche? A. cycle lasting 22 days B. menstrual bleeding lasting 10 days C. blood loss amounting 40 cc per cycle D. cycles lasting 30 days E. none of the above

335 B. Abnormal: >42 days cycle, <21 days cycle, bleeding >7 days. Average blood loss: 35 cc. Berek & Novaks Gynecology 14th edition pp.446-447

What is the single most important risk factor for intrauterine infection? A. Age B. Parity C. PROM D. Route of delivery

342 D.

What is the most important criterion for the diagnosis of postpartum metritis? A. Lochia B. Dysuria C. Fever D. Tenderness over the wound

343 C.

A 25 year old G2P1 (1001) just delivered by low forceps extraction to a live baby boy with BW 6 lbs. Continued oozing of bright red blood was noted with BP dropping to 80/50, CR 120/min, RR 20/min, T 37C. In 2009, she had a low transverse cesarean section for placenta previa. What is the most probable cause of the bleeding? A. Abruptio placenta B. Vaginal lacerations C. Rupture of CS scar D. Uterine atony

344 B. Since the patient underwent forceps extraction, the most likely cause of her bleeding would be due to lacerations. Uterine atony is unlikely since she is just only gravida 2, the birth weight is 6 lbs, and there are no other clues in the history that would make the patient at risk for this condition. Rupture of CS scar is also unlikely because her previous CS was a low transverse type.

Which of the following is the most common cause of postpartum hemorrhage? A. Retained placental cotyledons B. Coagulopathies C. Uterine atony D. Lacerations

345 C.

What is the most common cause of spontaneous abortions? A. Maternal systemic disease B. Chromosomal anomalies C. Uterine defects D. Immunologic factors

346 B.

What is the best approach in the management of unruptured tubal pregnancy 5-6 weeks with a live fetus? A. Laparoscopy B. Laparotomy C. Methotrexate D. Watchful observation

347 A. Laporotomy is usually indicated for patient who are hemodynamically unstable or for those with cornual pregnancies. Methotrexate therapy is only done when the ff. criteria are met: patient should be hemodynamically stable with no signs of hemoperitoneum gestational sac should be <=4cm no fetal cardiac activity no tubal rupture beta HCG less than 5000 mIU/mL

Which of the following is a marker of chronic hypoxia? A. Non stress test B. Contraction stress test C. Amniotic fluid index D. Fetal tone

348 C.

What procedure should be done next when a Pap smear result shows “high-grade squamous intraepithelial lesion� A. Repeat Pap smear B. Cone biopsy C. Colposcopy D. Random punch biopsies

356 C.

Which histologic type of cervical cancer is associated with a tendency for late diagnosis and a “barrel-shaped†morphology? A. Squamous carcinoma B. Sarcoma C. Adenocarcinoma D. Lymphoma

357 C.

A 16 year old, sexually active nulligravid, complains of vulvar pain. She noted yellowish green foul-smelling vaginal discharge for the past 3 days, and since 1 day ago has been experiencing pain at the start of urination. On PE: (+) firm, tender, bilateral inguinal lymph nodes, (+) multiple vesicular lesions and shallow superficial ulcers. What is the most likely diagnosis for this case? A. Chancroid B. Herpes C. Syphilis D. Lymphogranuloma venereum

358 B. Vesicular lesions, shallow ulcers, and most notably pain, are all characteristic of a herpes infection.

A 23 year old, nulligravid, recently married, consulted because of foul smelling, copious grayish vaginal discharge for the past week. On PE: speculum exam showed copious creamy, grayish, foul-smelling vaginal discharge; cervix is smooth and erythematous. What is the appropriate treatment for this patient? A. None B. Metronidazole C. Doxycycline D. Fluconazole

359 B. This is most likely a case of bacterial vaginosis, the treatment for which is metronidazole.

A patient is diagnosed to have Mayer-Rokitansky syndrome. This patient will have: A. 46XY karyotype B. Normal ovaries C. Infrequent association with skeletal abnormalities D. Normal uterus E. No involvement of the urinary tract

361 B. This syndrome is characterized by congenital absence of the vagina and uterus. These individuals have a 46,XX karyotype. Up to 50% of women with Mullerian agenesis have concur- rent urinary tract anomalies. One study described a 12% incidence of skeletal anomalies, usually involving congenital fusion or absence of vertebrae in these patients. Katz 6th pg 191-192

A 22 year-old primigravid at 35 weeks age of gestation was brought to the emergency room due to severe abdominal pain. Vital signs were BP = 160/100, HR 104, RR 22, T 37C. This was allegedly her first episode of elevated BP. Physical examination revealed a fundic height of 29cm. Stat albumin was 1+ while laboratory tests revealed elevated liver enzymes. What is your diagnosis? A. Gestational Hypertension B. Mild preeclampsia C. Severe preeclampsia D. Chronic hypertension E. HELLP syndrome

362 C.

A 25 year-old, G1P0 is on the 12th hour of labor, cervix 6cm dilated, at station -2 with contracted pelvis. Amniotomy was done. After 2 hours, cervix is now fully dilated, still at station -2. What is the preferred management for this case? A. Sedation and observation B. Augmentation with oxytocin C. Forceps delivery under anesthesia D. Abdominal delivery E. None of the above

363 D. This is a case of failure of descent. Preferred treatment in this case would be cesarean delivery if with CPD and oxytocin if without CPD. Williams 23rd pg 465

A 1 year old was brought to your clinic due to ambiquous genitalia. The patient has normal ovaries with normal development of the uterus. There was also note of virilization with scrotal formation and penile urethra. Karyotyping done revealed a 46XX karyotype. The patient most probably has: A. Congential adrenal hyperplasia B. Mullerian agenesis C. Androgen insensitivity syndrome D. Testicular feminization syndrome E. None of the above

364 A. Congenital adrenal hyperplasia is the most common cause of androgenic excess in fetuses with female pseudohermaphroditism. In this condition, müllerian-inhibiting substance is not produced. Androgen exposure is excessive, but variable, for a fetus genetically predestined to be female. The karyotype is 46,XX and ovaries are present. Because müllerian-inhibiting substance is not produced, the uterus, fallopian tubes, and upper vagina develop. If androgen levels increase earlier in embryonic develop- ment, then more severe virilization can be seen. This includes formation of labioscrotal folds; development of a urogenital sinus, in which the vagina empties into the posterior urethra; and development of a penile urethra with scrotal formation.

A 56 year-old G0 had abnormal postmenopausal bleeding and biopsy done revealed endometrial adenocarcinoma. Patient underwent EHBSO, PFC, BLND and final histopath showed that tumor is confined to the endometrium. All other pelvic structures and lymph nodes are negative. However, peritoneal fluid cytology was positive. What is the stage? A. IA B. IB C. II D. IIIA E. IIIC

373 A. This is a case of Stage IA endometrial adenocarcinoma since tumor is limited to the endometrium. Positive cytology has to be reported separately without changing the stage. Katz 6th pg 720

A 36 year old comes to your clinic because of amenorrhea. Her last normal menstrual period was 10 months ago. Physical and internal examination were essentially normal. Endocrine assays reveal estradiol levels as <40pg/ml and FSH was >30mIU/ml. What is your likely diagnosis? A. Hypothalamic-pituitary failure B. Polycystic ovarian syndrome C. Premature ovarian failure D. Hypothalamic-pituitary dysfunction E.

365 C. Premature ovarian failure (POI) or premature ovarian insufficiency (POI), which is the more recently used term, is defined as hypergonadotropic ovarian failure occurring prior to age 40. POI has occurred in 5% to 10% of women who are evaluated for amenorrhea. Patients will have abnormally low levels of estrogen with high levels of FSH.

TRUE of gestational diabetes: A. Screening should be done between 28-32 weeks in those women not known to have glucose intolerance earlier in pregnancy. B. Maternal hyperglycemia prompts fetal hyperinsulinemia particularly during the first half of gestation, which in turn stimulates excessive somatic growth. C. Women diagnosed with GDM have a 25-percent likelihood of developing overt diabetes within 20 years. D. Women with gestational diabetes who do not require insulin seldom require early delivery or other interventions. E. All of the above

366 D. Screening should be done between 24-28 weeks in those women not known to have glucose intolerance earlier in pregnancy. Maternal hyperglycemia prompts fetal hyperinsulinemia particularly during the second half of gestation, which in turn stimulates excessive somatic growth. Women diagnosed with GDM have a 50-percent likelihood of developing overt diabetes within 20 years. Williams 23rd pg 1106-1112

A 27 year old G3P2 on her 36th week of gestation, with a history of chronic hypertension, had severe abdominal pains of several hours with accompanying vaginal bleeding. Vital signs are BP = 140/90, PR = 108bpm, RR = 22/min and T = 37.2 C. Abdomen is tender and cervix is 3cm dilated, membranes are bulging. CTG is non-reactive with hypertonic uterine contractions. The most likely diagnosis is: A. Normal early labor B. Uterine rupture C. Placental abruption D. Placenta previa E. None of the above

367 C. Placental abruption presents with vaginal bleeding in 78 percent, uterine tenderness or back pain in 66 percent, and fetal distress in 60 percent of patients. Other findings included frequent uterine contractions and persistent uterine hypertonus. Williams 23rd pg 765-766

A 24 year-old primigravid complains of an ulcer in the genital area. According to her, it started as a shallow painless ulcer three weeks prior to consult. On physical examination, you noted tender inguinal lymph nodes. What is the most likely etiology? A. Chlamydia trachomatis B. Herpes simplex virus C. Hemophilus ducreyi D. Klebsiella granulomatosis E. Treponema pallidum

368 A. Lymphogranuloma venereum (LGV) is a chronic infection of lymphatic tissue produced by Chlamydia trachomatis. There are three distinct phases of vulvar and perirectal LGV. The primary infection is a shallow painless ulcer that heals rap- idly without therapy. One to 4 weeks after the primary infection, a secondary phase marked by painful adenopathy develops in the inguinal and peri- rectal areas. When the disease is untreated, the infected nodes become increasingly tender, enlarged, matted together, and ad- herent to overlying skin, forming a bubo (tender lymph nodes). Katz 6th pg 526

A 34 year old G2P2 presents at the OPD with white thick vaginal discharge associated with vaginal erythema and pruritus. Measurement of vaginal acidity reveals a pH of 4. What is your diagnosis? A. Bacterial Vaginosis B. Trichomoniasis C. Vulvovaginal candiasis D. Atrophic Vaginitis E. Gonorrhea

369 C. Bacterial vaginosis and Trichomoniasis will have a vaginal pH greater than 4.5.

A 26 year-old G2P1 consulted at the Emergency room. On internal examination, you noted that the cervix is 3 cm dilated, at midposition, with 40% effacement, with a soft consistency, and at station - 2. What is the Bishop score? A. 6 B. 7 C. 8 D. 9 E. 10

370 B. 3cm - 2, midposition - 1, 40% - 1, soft - 2, station -2 - 1

What is the mechanism of action of combined oral contraceptives? A. Progesterone suppresses LH action while estrogen suppresses FSH action. B. Progesterone suppresses FSH action while estrogen suppresses LH action. C. Progesterone suppresses GnRH action while estrogen suppresses FSH action. D. Progesterone suppresses FSH action while estrogen suppresses GnRH action.

371 A. Progestins prevent ovulation by suppressing LH and also thicken cervical mucus, thereby retarding sperm passage. In addition, they render the endometrium unfavorable for implantation. Estrogen prevents ovulation by suppressing FSH release. Williams 23rd pg 673

The most common adnexal mass in the adolescent age group. A. Germ cell tumor B. Dermoid cysts C. Infectious cyst D. Functional ovarian cyst

372 D. Most ovarian masses in the pediatric and adolescent age group are functional ovarian cysts, and if a tumor is present it most often is a benign teratoma (dermoid). Malignancies can, however, occur and are most often of germ cell origin, but they can also be sex cord tumors such as a granulosa cell malignancy. Katz 6th pg 211

The following are techniques employed to deliver the aftercoming head EXCEPT: A. Mauriceau maneuver B. Lovesets maneuver C. Modified Prage maneuver D. Pinard's maneuver

374 B. Methods used in the delivery of the aftercoming head include: Piper's Forceps, Mauriceau maneuver, Prague maneuver, Bracht maneuver and Pinard's maneuver. Lovesets maneuver is used to deliver the shoulder. Topnotch

The following can be observed in the puerperium EXCEPT: A. Women approach their self-reported pre- pregnancy weight 3 months after delivery. B. By 2 weeks postpartum, the uterus has descended into the true pelvis. C. Ovulation may resume as early as 3 weeks after delivery, even in lactating women. D. The bladder has increased capacity and relative insensitivity to intravesical pressure.

375 A. Women approach their self-reported pre- pregnancy weight 6 months after delivery but still retain an average surplus of 3 lbs. Williams 23rd pg 649

A 27 year-old G2P1 36 weeks of age of gestation came in the emergency room for uterine contractions. Internal examination revealed 2cm dilatation, station -2, with adequate pelvimetry. After 14 hours, IE revealed 3cm dilatation still at station -2. What is the preferred management in this case? A. Cesarean delivery B. Augment with oxytocin C. Advise bed rest D. Administer tocolytics

376 C. This is case of prolonged latent phase. Preferred management would be bed rest. Administration of oxytocin or abdominal delivery is done if with urgent problem Williams 23rd pg 465

In placenta accreta, what layer is defective or absent? A. decidua basalis B. decidua functionalis C. decidua stroma D. Nitabuch's layer

377 D.

The most common cause of obstetrical hemorrhage is A. Coagulopathy B. Placental abruption C. Placental Accreta D. Retained Placental fragments E. Uterine atony

378 E. Failure of the uterus to contract properly following delivery is the most common cause of obstetrical hemorrhage.

Triad of ectopic pregnancy: A. Amenorrhea, vaginal bleeding and adnexal mass B. Amenorrhea, vaginal bleeding and abdominal pain C. Positive pregnancy test, vaginal bleeding, and an adnexal mass D. Positive pregnancy test, vaginal bleeding and abdominal pain

379 B. The most common symptoms of ectopic pregnancy are abdominal pain, absence of menses, and irregular vaginal bleeding Katz 6th pg 367

The single most important risk factor for the development of uterine infecion after childbirth. A. Prolonged labor induction B. Route of delivery C. Bacterial colonization of the lower genital tract D. Meconium-stained amniotic fluid E. None of the above

380 B. The route of delivery is the single most significant risk factor for the development of uterine infection. Williams 23rd pg 661

Which of the following represents the shortest AP diameter of the pelvic inlet? A. Diagonal conjugate B. Obstetric conjugate C. True conjugate D. Interspinous diameter E. Intertuberous diameter

381 B. Diagonal conjugate: measured clinically and is roughly 1.5 cm longer than the obstetric conjugate.

Which of the following is the most common menstrual abnormality associated with hyperthyroidism? A. Oligomenorrhea B. Menorrhagia C. Metrorrhagia D. Polymenorrhea E. Intermenstrual bleeding

382 A. SIMILAR TO PREVIOUS BOARD EXAM CONCEPT/PRINCIPLE. Oligomenorrhea and hypomenorrhea are the most common menstrual abnormalities in hyperthyroidism.

Which uterine ligament provides the major support to the cervix and vagina? A. Broad ligament B. Cardinal ligament C. Uterosacral ligament D. Round ligament E. Infundibulopelvic ligament

383 B.

The Arias-Stella reaction is seen in which of the following conditions/periods? A. Pregnancy state B. Luteal phase C. Proliferative phase D. Cervical cancer E. Menopause

384 A. Arias-Stella reaction refers to benign endocervical gland hyperplasia of pregnancy. It can cause misdiagnosis of Pap smear specimens from pregnant patients as its appearance mimics endometrial CA.

Which of the following antihypertensive medications are contraindicated in pregnancy? A. Labetalol B. Methyldopa C. Hydralazine D. Captopril E. None of the above

385 D. ACEIs and ARBs are contraindicated in pregnancy.

During pregnancy, the uterus normally undergoes which of the following positional change? A. Rotates to the left B. Rotates to the right C. Flexes anteriorly D. Flexes posteriorly E. None of the above

387 B. The gravid uterus normally rotates to the right due to the presence of the sigmoid colon in the left aspect of the pelvic cavity.

Mittelschmerz pain occurs during which of the following periods? A. Early trimester pregnancy B. Late trimester pregnancy C. Ovulation D. Menstruation E. Perimenopause

388 C. Mittelschmerz refers to the lower abdominal and pelvic pain experienced by some women midway in their menstrual cycle, corresponding to the period of ovulation.

Among pregnant patients, when is the best time to perform screening for Group B streptococcal infection? A. on initial visit B. 24-28 weeks AOG C. 33-35 weeks AOG D. 35-37 weeks AOG E. once labor has begun

389 D.

In a pregnant patient without known risk factors, when is the optimal time to perform OGTT to screen for gestational diabetes? A. on initial visit B. 16-20 weeks AOG C. 24-28 weeks AOG D. 32-36 weeks AOG E. 20-24 weeks AOG

390 C. For high-risk patients, perform OGTT on initial prenatal visit.

A 33-year old pregnant patient on her 36-week AOG was admitted for sudden onset of painful vaginal bleeding. You diagnosed her to have abruptio placenta and decided to perform emergency cesarean delivery. Given the nature of this operation and anticipating severe blood loss, which is of the following types of anesthesia would you recommend for the patient? A. Regional anesthesia B. Epidural anesthesia C. Combined regional and epidural anesthesia D. General anesthesia E. Local anesthesia

391 D. SIMILAR TO PREVIOUS BOARD EXAM CONCEPT/PRINCIPLE. General anesthesia is recommended in the following cases: nternal podalic version, breech decomposition, inverted uterus, severe hemorrhage and emergent C/S. Epidural anesthesia is preferred for severe preeclampsia, eclampsia and CHF.

A G4P4 mother presented with complaints of dysmenorrhea and menorrhagia. Pelvic exam revealed a diffusely and symmterically enlarged uterus. Which of the following would you expect on histopathologic investigation? A. Presence of ectopic endometrial glands and stroma on the peritoneal lining of the uterus B. Presence of endometrial glands and stroma in the myometrium C. Increased endometrial thickness without invasion of the myometrium D. Proliferation of smooth muscle cells in the myometrium with areas of hyaline and cystic degeneration E. Atrophic endometrial mucosa with friable myometrial blood vessels

392 B. SIMILAR TO PREVIOUS BOARD EXAM CONCEPT/PRINCIPLE. Diagnosis: adenomyosis.

A 23-year old female came into your clinic due to 5-day history of vaginal discharge. On pelvic examination, you noted green-yellow frothy discharge and multiple erythematous punctate lesions in her vagina. With your diagnosis in mind, which of the following regimens would you prescribe to your patient? A. Metronidazole 2 g PO single dose B. Metronidazole 500 mg BID x 7 days C. Fluconazole 150 mg PO single dose D. Nystatin vaginal wash E. Either A or B is acceptable

393 A. SIMILAR TO PREVIOUS BOARD EXAM CONCEPT/PRINCIPLE. Diagnosis: Trichomoniasis

Which of the following forceps is used to assist in the delivery of the aftercoming head during breech extraction? A. Simpson forceps B. Tucker-McLane forceps C. Kielland forceps D. Piper forceps E. Elliot forceps

394 D. SIMILAR TO PREVIOUS BOARD EXAM CONCEPT/PRINCIPLE.

Which of the following substances is responsible for myometrial contractility and is implicated in dysmenorrhea? A. Oxytocin B. Prostaglandin F2-alpha C. Enkephalinase D. Endothelin E. BMP-15

395 B.

A 4-year old female presented at your clinic with symptoms of UTI. On physical examination, you noted labial adhesion with translucent vertical line at midline. Which of the following would you prescribe in relation to your finding? A. Topical clobetasol cream B. Topical estrogen cream C. Topical testosterone cream D. Low-dose oral estrogen therapy E. Surgical incision of the adhesion

396 B. SIMILAR TO PREVIOUS BOARD EXAM CONCEPT/PRINCIPLE. Diagnosis: adhesive vulvitis

A 35-year old female came into your clinic with a Pap smear result indicating "low-grade squamous intraepithelial lesion". What is the next appropriate step in your management? A. Request for transvaginal ultrasound to assess the depth of the lesion B. Request for CT scan to assess regional node status C. Request for colposcopy and biopsy D. Perform definitive management with cold-knife conization since this is a clinical diagnosis E. Performed loop electrosurgical excision procedure.

397 C. Pap smear showing ASC-US, LSIL, HSIL or SCC should always be followed up with colposcopy and biopsy. Therapeutic management would then depend on the result of the biopsy.

A 43-year old multigravid presented with 1-year history of gradually enlarging, friable mass arising from her cervix. On pelvic examination, you noted nodularities in the bilateral parametria. Abdominal ultrasound showed no liver metastasis; however, bilateral ureteropelvocaliectasia was noted. What would be the best management for this patient? A. Radical hysterectomy, followed by radiotherapy and adjuvant chemotherapy. B. Radical hysterectomy, followed by radiotherapy only C. Cisplatin-based chemotherapy with concurrent external beam radiotherapy, followed by brachytherapy D. Cisplatin-based chemotherapy, followed by EBRT, then brachytherapy E. Cisplatin-based chemotherapy only due to presence of ureteropelvocaliectasia

398 C. Diagnosis: cervical CA, stage IIIB. The presence of dilated ureters and renal pelves is an indicator of pelvic side wall involvement.

Which of the following regimens would you recommend for a 18-year female seeking emergency contraception? A. Progestin only pills for 7 days B. Mifepristone x 5 doses C. Copper IUD D. Ethinyl estradiol + levonorgestrel x 2 doses E. Misoprostol x 5 doses

399 D. Ethinyl estradiol + levonorgestrel combination, more popularly known as Yuzpe, is an effective emergency contraception when taken up to 2 days following coitus. Levonorgestrel x 2 doses is also an acceptable alternative.

A 35 year old patient was diagnosed with an Ovarian tumor, a biopsy was done revealing Transitional cells, similar to a urinary bladder-type of histology surrounded by a massive growth of a fibrous stroma. What would be the most appropriate management? A. Unilateral Oophorectomy B. Bilateral Oophorectomy C. Unilateral Salphingo-oophorectomy D. TAH-BSO E. TAH-BSO with lymph node dissection and PF analysis

401 C. This type of Branner's tumor is benign due to the presence of an outer fibrous stroma. All benign tumors require only a unilateral salphingo- oophorectomy. (SIMILAR TO PREVIOUS BOARD EXAM CONCEPT/PRINCIPLE)

Which of the following cardinal movements is the 1st prerequisite for birth? A. Engagement B. Descent C. Flexion D. Internal Rotation E. Extension

402 B. SIMILAR TO PREVIOUS BOARD EXAM CONCEPT/PRINCIPLE

Bartholin's Gland is a derivative of which of the following anlagen? A. Wolffian Duct B. Mullerian Duct C. Genital Tubercle D. Genital Swelling E. Urogenital Sinus

403 E. Derivatives of the Urogenital Sinus include the female urethra, lower vagina (2/3), Bartholin's Gland and Skene's Gland.

A 15-year old patient was brought to you by her mother apparently because she still has not menstruated, secondary sexual characteristics are present in this patient and there is no developmental delay. What would be your next step? A. Measure FSH and LH B. Do a transrectal UTZ C. Do CNS imaging D. Do Karyotyping E. Observe

404 E. Primary amenorrhea in a patient with the presence of secondary sexual characteristics is considered if the patient is already 16 years old. Our patient is just 15, a more appropriate step is to wait for her 16th birthday, if still with no menses then do diagnostics.

The most common symptom of Vulvar Intraepithelial neoplasia is: A. Pruritus B. Pain C. Palpable nontender mass D. Bleeding E. Acanthosis

405 A. Pruritus is the most common symptom of VIN hence in all pruritic valvualr lesions, biopsy is warranted.

True of Lichen sclerosus, EXCEPT: A. Treated with Testosterone cream B. Has a 5-15% risk for CA in premenopausal women C. Pruritus is the most common presentation D. May involve the vaginal wall E. Creates an hourglass appearance

406 D. Lichen sclerosus does not involve the vagina.

A 20-year old Nulligravid has just underwent a Pap Smear revealing a CIN II result, which of the following is the best option for this patient? A. Observe and repeat Pap smear after 3 months B. Ablative therapy C. Perform Hysterectomy only D. Do a TAHBSO E. Do chemoradiation only

407 B. For patients with High grade SIL (CIN II and III) and still desires infertility, ablative therapy or excisional procedures are the options, hysterectomy is only warranted for patients who has no desire for fertility/ or has completed child-bearing years.

Your sister is an excited new mom-to-be, being enthusiastic as she is, she then asks you when would they know the sex of their baby on UTZ? You said that it is as early as: A. 7 weeks AOG B. 9 weeks AOG C. 11 weeks AOG D. 14 weeks AOG E. 20 weeks AOG

414 D. The phenotypic sex is well formed at 17 weeks but can be identified by expert sonologists at 14 weeks.

Which of the following statements is NOT true regarding Cervical Cancer? A. HPV 6, 11 infections poses a high risk for developing Carcinoma B. Squamous Cell Carcinoma is the most common type C. Vaginal Bleeding is the most common symptom D. Uremia is the most common cause of death E. HPV vaccine can be administered starting 9 years old

408 A. HPV strains 16 and 18 poses a high risk for carcinoma, not 6 and 11 which causes only genital warts.

A 50-year old asymptomatic patient with a Myoma as large as a baseball, sought consult from an OB- GYN, what would be the most appropriate management for this patient? A. Observe B. Medical Therapy with a GnRH Analogue C. Do Myomectomy D. Do embolization E. Do a hysterectomy

409 A. Asymptomatic patients with myoma are managed conservatively.

A patient with a recently discovered endometriosis found on her lungs asks for an explanation from you why this occurred, you would explain which of the following theories of endometriosis? A. Retrograde Menstruation B. Metaplasia of Coelomic Epithelium C. Lymphatic and Vascular Metastasis D. Iatrogenic Dissemination E. Malignant Degeneration

410 C.

A patient with an ovarian fibroma, suddenly developed Meig's Syndrome, among which of the following would be seen in this patient? A. Pneumothorax B. Ascites C. Amenorrhea D. Menorrhagia E. Palpable abdominal mass

411 B. Meig's Syndrome: Ovarian Fibroma, Ascites, Hydrothorax

A patient presenting with a pelvic mass underwent a Pelvic ultrasound, which of the following characteristics will make you suspect that the mass is probably malignant? A. Unilateral Mass B. Absence of septations C. Noted Calcifications D. Multicystic or multilocular mass E. size of less than 8 cm

412 D. all others are probable signs that the mass is benign.

Drugs are categorized according to their safety profile for consumption of pregnant patients, Phenytoin is an example. Phenytoin is found to have positive evidence of human fetal risk based on studies in humans, but potential benefits may warrant use of the drug despite potential risks. What category is Phenytoin classified? A. A B. B C. C D. D E. X

413 D.

The earliest prenatal screening that can be done as early as 9 weeks AOG is among which of the following? A. Cordocentesis B. Chorionic villus sampling C. Early amniocentesis D. Percutaneous Umbilical cord blood sampling E. None of the above

417 B. Chorionic villus sampling can be done as early as 9 weeks to 12 weeks AOG.

A G2P1 patient on her 16th week AOG was rushed in the ED due to severe abdominal pain, hypotension, tachycardia and signs of peritonitis, which among the following would be your consideration? A. Rupture Ectopic Pregnancy, Ampulla B. Ruptured Ectopic Pregnancy, Isthmus C. Ruptured Ectopic Pregnancy, Interstitial D. Abruptio Placenta E. Placenta Previa

419 C. Ectopic pregnancy of the interstitial part of the FT will rupture only at around 16 weeks, rupture of the ampulla occurs at around 8-12 weeks, while the isthmus which is the narrowest will most likely rupture in the first 6-8 weeks of pregnancy. D and E are seen in the 3rd trimester of pregnancy.

CPD can be totally ruled out in a nulligravid patient by which of the following procedure? A. Xray Pelvimetry B. Clinical Pelvimetry C. History taking D. Pelvic CT-Scan E. None of the above

420 E. CPD can not be totally ruled out in a nulligravid patient. A history of CPD in a multipara will give a great idea to a clinician that the patient may again come out with the same problem.

A female patient was noted to be of short stature, with webbed neck, shield chest, and a congenital heart disease. She has normal mentation. Genetic analysis, however, revealed a normal karyotype. This patient has: A. Superfemale syndrome B. Turner syndrome C. Rokitansky Kuster Hauser Mayer syndrome D. Noonan syndrome

421 D. Noonan syndrome shares clinical features with Turner syndrome, including the signs mentioned in the item. However, the observation that patients with Noonan syndrome have normal karyotypes is important in allowing the distinction to be made between the two (Turner has the karyotype 45 XO). Significantly, Noonan syndrome patients are able to bear offsprings, however management is complicated with uterine anatomic anomalies which are also common in these patients.

A 35-year old patient was rushed to the OB ER after experiencing excessive pelvic pain and cramping with notable vaginal bleeding. She is febrile and tachycardic but with normal BP and good peripheral pulses and perfusion. During history taking, she admits that she has been working as a prostitute and does not use condoms or OCPs. The first test that you will order for this patient is: A. Pelvic sonography B. Blood culture C. Diagnostic laparoscopy D. β-hCG E. None of the above

422 D. All OB-GYN emergencies, especially if the patient is hemodynamically stable, warrants a pregnancy test prior to any intervention. This is a case of secondary dysmenorrhea with several possible etiologies including ectopic pregnancy, septic abortion, pelvic infection, etc.

Which of the following is not true about menopause: A. The age of onset is genetically determined. B. There is a significant decrease in the number of ovarian follicles accompanied by degeneration of granulosa and theca cells. C. Stromal cells of the ovaries sustain their capacity to produce androgenic hormones. D. The remaining estrogen levels in the circulation during menopause is dictated by the proportion of body fat. E. None of the above.

423 E. Menopause is defined as permanent cessation of menstruation, featured with 3 consecutive months of amenorrhea with elevations of FSH and LH. The mean age of onset among Filipinos is said to be at 51. The increase in the gonadotropins as well as the cessation of mestrual cycle is brought about by the loss of ovarian follicles secreting inhibin and estrogen. Estrone (E1) is the predominant estrogen in menopause. It is converted from androstenedione which comes from body fat.

Ferning or arborization of the cervical mucus is brought about by the crystallization of sodium chloride on mucus fibers in the presence of elevated levels of estrogen. This pattern can be normally seen in which phase of the menstrual cycle? A. Proliferative phase B. Secretory phase C. Luteal phase D. Any phase

424 A. On the other hand, BEADING is seen during the secretory phase of the menstrual cycle and is due to high levels of progesterone. When ferning is seen in all phases of the cycle, it suggests infertility, autonomic ovarian failure or menopause.

On endometrial biopsy, glycogen-rich subnuclear vacuoles were seen in the base of the cells lining the glands. The hormone that is predominant in this phase is: A. Estrogen B. FSH C. LH D. Progesterone E. Gonadotropin

449 D. Basal vacuolation is the earliest histological evidence of progesterone action.

A 24-year old G2P1 at 38 weeks AOG is admitted at the OB ward. She reports that she has been experiencing regular uterine contractions for 19 hours now. IE reveals cervical dilation remaining at 2 cms. She is afebrile with normal heart rate and BP. Fetal monitoring is also reassuring. You consider prolonged latent phase of labor. What will be your next plan? A. Expectant management B. Induction of labor C. Augmentation of labor D. Refer to a perinatologist

425 A. Prolonged latent phase is considered if the patient has been experiencing regular uterine contractions for >20 hours if nulliparous, or >14 hours if with previous delivery, with cervical dilation remaining at 2 cms. Primary manegement is still expectant and supportive, although one should consider that the mother may suffer from exhaustion or uterine infection.

Cesarean section is an operative procedure requiring a uterine incision which aims to deliver the fetus abdominally . Which of the following is an absolute indication for this procedure? A. Massive maternal obesity B. Transverse lie C. Contracted pelvis D. Placenta previa

426 C. Dystocia, on the other hand, is the most common indication for a primary cesarean section.

A 23-year old G2P2 has just given birth to a live healthy term 2.8 kg baby girl via spontaneous vaginal delivery. She is stable with adequate response to post-delivery care and heading to an unremarkable peuperal stage. At what time is her uterus expected to descend into the true pelvis? A. At 1 week post-partum B. At 2 weeks post-partum C. At 3 weeks post-partum D. At 4 weeks post-partum E. Soon after delivery of the placenta

427 B. Peurperium usually lasts up to 6 weeks. Decidua differentiates in 2-3 days after delivery. At 2 weeks, uterus returns to the true pelvis; at 3 weeks, the entire endometrium becomes restored; and at 4 weeks, the uterus reaches its non-gestational size.

The type of anesthesia ideal for severe pre- eclampsia and eclampsia and is also considered as the gold standard in obstetrical anesthesia is: A. Spinal anesthesia B. Pudendal anesthesia C. Epidural anesthesia D. General anesthesia

428 C. Pudendal block is used in manual exploration of uterine cavity, in outlet forceps delivery and in repair of vagina and cervix. Spinal anesthesia is more commonly used in elective CS (the duration of procedure is more time-limited, since the anesthesia is given as a single shot). While general anesthesia is indicated in breech decomposition, replacement of inverted uterus, and internal podalic version of second twin.

A 33-year old G3P1 at 36 weeks AOG was referred to your hospital due to abnormal placentation detected sonographically at 29 weeks AOG. She had CS for placenta previa 3 years ago. At the admitting section, the patient reports vaginal spotting but no severe abdominal complaints. Repeat ultrasound reveals a viable baby and CS was immediately performed. The surgeon however noted profuse bleeding that was not ameliorated by any means. Hysterectomy was done. This abnormal condition can be best explained by: A. Velamentous insertion of the umbilical cord B. Absence of the decidua basalis with imperfect development of the Nitabuch layer C. Avulsion of anchoring villi of low implanted placenta D. Uterine ischemia secondary to prolonged myometrial contractions

429 B. This is a case of placenta accreta where chorionic villi penetrate the decidua but not the myometrium (accreta vera, 75%); or deeper into the myometrium but not the seros (increta, 15%); or may reach and even perforate the serosa and invade adjacent structures (percreta, 5%). Option A explains vasa previa while option C pertains to placenta previa.

Which of the following conditions will least likely cause uterine inversion? A. Strong umbilical cord traction B. Myometrial fatigue C. Placenta accreta D. Abruptio placenta

430 D.

A 17-year old primi at 37 weeks AOG, who is expected to deliver in a week, returned to your clinic after observing multiple pruritic vesicular rashes on her torso and head. You know that this is infectious and requires intervention, therefore you will consider: A. Giving pregnancy-graded oral anti-viral medications to prevent spread in the circulation saving the feto-maternal blood circuit. B. Giving vaccine to the mother to prevent transplacental transfer. C. Administering immune globulins to the newborn. D. An emergency cesarean section. E. All can be considered.

431 C. VZV Ig should be given to neonates born to mothers who have clinical evidence of the infection 5 days before or up to 2 days after delivery. Varicella vaccine is not recommended to pregnant women, or to those who expect to conceive in the month following vaccination.

Which Leopold's maneuver will determine the degree of flexion (attitude) of fetal head? A. Leopold's I B. Leopold's II C. Leopold's III D. Leopold's IV

432 D. A FAQ as well. Leopold I is the fundal grip and is used to determine the fetal part that lies in the fundus (presentation). Leopold II identifies the location of fetal back and small parts (fetal lie). While Leopold's III determines engagement and is also known as the Pawlick's sign.

A pre-eclamptic primigravid is admitted at the maternal ICU. Her condition is adequately controlled by MgSO4 drip. You ask the clerk-on- duty to monitor the possible adverse effects of such medication. In your mind, you expect that this patient will start having respiratory depression if her MgSO4 blood level reaches: A. 7 mEq/L B. 10 mEq/L C. 14 mEq/L D. 18 mEq/L E. 30 mEq/L

433 C. There are 3 parameters that we monitor in patients maintained on MgSO4: the deep tendon reflex, RR, and urine output. The therapeutic level of MgSO4 is at 4-7 mEq/L; disappearance of the patellar reflex is set at 10 mEq/L. On the onter hand, respiratory depression sets in if value is >12 mEq/L, and it may require mechanical ventilation if >15 mEq/L. Cardiac arrest is expected to happen at levels >30- 35 mEq/L. Calcium gluconate (1 gm IV) is the antidote.

Which of the following is not found in HELLP syndrome? A. Elevated LDH B. Thrombocytopenia C. Elevated liver enzymes D. Hemorrhage with prolonged PT/PTT E. None of the above

434 D. HELLP means hemolysis (high LDH - enzyme released from RBCs), elevated liver enzymes,and low platelet.

What CNS anomaly in infants is considered to be most specific to maternal DM? A. Down syndrome B. Sacral agenesis C. Cretinism D. Lissencephaly

435 B. However, other causes are presumably involved, as demonstrated by the rare incidence of sacral agenesis compared to diabetes and certainly, not all children born with the condition have diabetic mothers.

Which will you give/recommend to a lactating woman who desires contraception? A. Combined oral pill B. Levonorgestrel C. Minipill D. Copper T IUD

436 C. Please study the methods of contraception, especially the hormonal pills. Minipill is also known as progestin only pill. It has 0.5 mg of progesterone and is considered safe and appropriate for breastfeeding women.

A 27-year old G3P2 mother who is now at her 14 weeks AOG, is rushed to the OB ER after complaining of minimal vaginal spotting. She is afebrile with normal BP and heart rate. On IE, her cervix is closed with uterine size estimating 10 weeks AOG. Which of the following is the next best step? A. Discharge the patient and advise her to have complete bed rest. B. Perform an ultrasound and perform an elective dilatation and curettage. C. Perform a prompt dilatation and evacuation. D. Perform an ultrasound, observe the patient for recurrence of spotting, and advise for expectant management.

437 B. The manifestations are most consistent with a case of missed abortion. Expected findings in the UTZ include an empty gestational sac in blighted ovum or a fetus without cardiac activity. Elective D&C is Complete abortion will have no bleeding and will show an empty cavity on UTZ.

Vulvar carcinoma is strongly related to HPV with an excellent orverall survival rate. It is usually of the squamous cell CA type and may present either as a flat, raised, plaque-like, ulcerated or polypoid masses on the vulva. If a patient with this condition would come to you for consult she would most likely complain of vulvar: A. Pain B. Bleeding C. Itching D. Adhesions

438. C..Pruritus is the most common symptom of vulvar carcinoma.

A subseptate uterus can result in 1st trimester pregnancy loss because of: A. Poor vascularization B. Limited uterine space hence restricting fetal growth C. Increased risk of bacterial invasion and chorioamnionitis D. High chance of placental fragmentation and loss

439 A. Septate and subseptate are associated with first trimester miscarriage due to inadequate vascularization. Pregnancy losses during 2nd trimester can be seen in unicornuate and bicornuate uteri. With these anomalies, fetal growth is more physically restricted.

What is the most common type of uterine myoma? A. Subserosal B. Submucosal C. Intramural D. Cervical

440 C. *FAQ. The submucosal type is the one most commonly associated with heavy and prolonged bleeding.

The presence of heavy concentration of coccobacilli surrounding vaginal epithelial cells with loss of distinct cell margins is the appearance of: A. Donovan bodies B. Chancre C. Clue cells D. Inclusion cells E. Koilocytes

441 C. Clue cells are epithelial cells of the vagina that get their distinctive stippled appearance by being covered with coccobacilli. Donovan bodies are rod- shaped, oval organisms that can be seen in the cytoplasm of mononuclear phagocytes or histiocytes in tissue samples from patients with granuloma inguinale. A chancre is a painless ulcer most commonly found in primary stage of syphilis. Koilocytes are cells found with HPV infection.

Speculum exam of a 27 year-old female complaining of leucorrhea showed copious frothy greenish vaginal discharge with strawberry-like mucosa. This is most likely due to: A. Candida albicans B. Trichomonas vaginalis C. Neisseria gonorrheae D. Chlamydia trachomatis E. Gardnerella vaginalis

442 Trichomoniasis is caused by Trichomonas vaginalis manifested with green-yellow frothy vaginal discharge associated with strawberry cervix. Candidiasis has cottage cheese-like discharge, gonorrhea and chlamydia infection has no dischrage in infected women, Bacterial vaginosis has grayish-white discharge.

A 28 year-old, G1P0 woman was diagnosed of pregnancy-induced hypertension was seen at the ER due to convulsion. The anticonvulsant of choice is magnesium sulfate. Which of the following findings would limit administration of the magnesium sulfate? A. PR of 70bpm B. bradypnea C. +2 tendon reflexes D. BP of 140/90 E. urine output of 35cc/hour

443 B. Magnesium sulfate is the anticonvulsant of choice for preeclampsia and eclampsia. Administration is limited if the patient had depressed tendon reflexes, respiratory rate <12cpm, and urine output <30cc/hour hence, these should be monitored.

It is a metastatic tumor to the ovary, usually bilateral, consisting of signet ring cells, usually originating from gastrointestinal tract: A. Mucinous tumor B. Serous tumor C. Sex-cord stromal tumor D. Krukenberg tumor E. Brenner tumor

444 D. A Krukenberg tumor refers to a malignancy in the ovary that metastasized from a primary site, classically the gastrointestinal tract, although it can arise in other tissues such as the breast. Krukenberg tumors are often found in both ovaries, consistent with its metastatic nature, most commonly from gastric adenocarcinoma. Lymphogranuloma venereum (LGV) is an uncommon sexually transmitted disease (STD) caused by Chlamydia trachomatis.

What ulcerative lesion of the genital tract is characterized by the presence of "groove sign"? A. Granuloma inguinale B. Lymphogranuloma venereum C. Chancroid D. Syphilis E. TB of genital tract

445 B. Lymphogranuloma venereum (LGV) is an uncommon sexually transmitted disease (STD) caused by Chlamydia trachomatis. This condition is characterized by self-limited genital papules or ulcers followed by painful inguinal and/or femoral lymphadenopathy. The ‘groove sign’ characteristic of LGV is seen if both the inguinal and the femoral nodes are involved.

A 25 year-old G1P1 was diagnosed with cervical intraepithelial neoplasia involving the entire thickness of the cervical epithelium. This is best managed by: A. Cryosugery B. Electrocautery C. Conization D. Hysterectomy E. CO2 laser ablation

446 C. Conization is a treatment of choice for women who are still desirous of pregnancy with high grade cervical dysplasia. Conization removes a cone shaped piece of tissue from the cervix. It is also called a cone biopsy and can be used to help diagnose cervical cancer. Cocaine, a small molecule, is able to cross the placenta into the bloodstream of the fetus.

Cocaine use in pregnancy is associated with: A. Spontaneous abortion B. Prematurity C. Abruptio placenta D. A and B E. All of the above

447 E. Cocaine is a a small molecule which is able to cross the placenta into the bloodstream of the fetus. Cocaine-using pregnant women deliver prematurely. There are also data showing that spontaneous abortion and low birth weight. The increased risk of placental abruption with cocaine use has been well documented.

The third stage of labor commences: A. After expulsion of placenta B. After delivery of fetus C. When cervix is fully dilated D. After episiorrhapy E. Cervix is fully effaced

448 B. Delivery of the fetus commences the third stage of labor and is also the end of second stage of labor. First stage of the labor ends when cervix is fully dilated and fully effaced.

The order of the 4 divisions of the fallopian tubes from the ovary to the uterus are: A. Infundibulum, isthmus, ampulla, interstitium B. Infundibulum, ampulla, isthmus, interstitium C. Infundibulum, ampulla, interstitium, isthmus D. isthmus, ampulla, interstitium, infundibulum E. interstitium, isthmus, ampulla, infundibulum

450 B. The order of the 4 divisions of the fallopian tubes from the ovary to the uterus are infundibulum, ampulla, isthmus, interstitium.

Abdominal enlargement and a positive hormonal test maybe considered: A. Positive signs of pregnancy B. Presumptive signs of pregnancy C. Probable signs of pregnancy D. Negative signs of pregnancy E. None of the above

451 C. Abdominal enlargement, changes in the servix, hegar's sign, goodell's sign, braxton hick's contractions, ballottement, physical outlining of fetus in the uterus and positive pregnancy test are the probable evidence of pregnancy.

Immunization during pregnancy may be given except the one which is absolutely contraindicated is: A. Tetanus B. Rubella C. Hepatitis B D. Poliomyelitis E. Cholera

452 B. Vaccines contraindicated for pregnant women are MMR and varicella.

A 27 year-old, G2P1, 41 weeks AOG was admitted in labor. Fundic height was measured at 30cms with good fetal heart tone. Cervix is 4cm dilated, not effaced, station -1. About 1 hour after admission, IE findings remained the same. What would be the most appropriate thing to do? A. deliver the baby by CS B. observe and evaluate C. give oxytocin drip to augment labor D. ask patient to walk around to hasten labor E. do amniotomy

453 B. The parturient is in the active stage of normal labor. Monitoring, observation is the only appropriate thing to do.

Seven minutes after a normal delivery under pudendal anesthesia, the patient has not completed the third stage of labor. The uterus is discoid and firm, no bleeding is evident. You should: A. Manually remove the placenta B. Pull the cord vigorously C. Invert the uterus D. Gently massage the uterus and wait E. Remove placenta from inverted uterus

454 D. The length of the third stage itself is usually 5-15 minutes. Expectant, or physiologic, management involves waiting for the typical signs of placental separation such as fundal rise, a gush of blood, and lengthening of the umbilical cord, then allowing the placenta to deliver spontaneously.

In vaginal delivery for breech presentation, the forceps of choice in delivery of the aftercoming head is: A. Kielland B. Simpson's C. Piper's D. Tucker-Mclane E. Barton

455 C. Piper's forceps is used in the delivery of the aftercoming head in a vaginal delivery for breech presentation. Simpson forceps is the most common forceps to deliver babies with molded head in nulliparas. Tucker-Mclane is used to deliver babies with rounded head in multiparas. Kielland forceps is ideal for rotating the head with occiput transverse. Barton forceps is used for rotation of the head in transverse arrest.

A 30 year-old, who just delivered a healthy male neonate, had fever, hypogastric pain and odorous vaginal discharge. Infection developed is most likely in the form of: A. Vaginitis B. Salpingitis C. Pelvic abscess D. Endometritis E. Peritonitis

456 D. Endometritis is a uterine infection with polymicrobial cause. Fever is the most important in 2-3 days postpartum associated with abdominal pain and malodorous lochia.

A 26 year-old female with past history of gonococcal infection was seen at the Emergency Room due to moderate to severe lower abdominal pain associated with intermittent vaginal spotting about 6 days duration. Her LMP was 6 weeks prior. Your initial diagnosis is: A. ectopic pregnancy B. ruptured ovarian cyst C. endometriosis D. recurrent gonorrhea E. salpingitis

457 A. Ectopic pregnancy is the implanation of trophoblast other than the endometrium of the uterine cavity. The most frequent site is ampulla of the fallopian tube. Clinical manifestations begin at 6-8 weeks with the triad of amenorrhea, vaginal bleeding and abdominal pain.

A 32 year-old, G1P0 at term suddenly had severe continuous low abdominal pain and tenderness associated with hypotension, tachycradia and nonreassuring fetal heart tone. She is most probably having: A. Start of labor B. Placenta previa C. Abruptio placenta D. Amniotic fluid embolism E. Uterine rupture

458 C. Abruptio placenta is usually a 3rd trimester painful bleeding with crampy abdominal pain in a patient with hypertension or a history of trauma. Stabilization of the mother and immediate delivery of the fetus is warranted.

Overriding of the fetal skull bones in x-ray at term is: A. Halo sign B. Spalding sign C. Indicates fetal prematurity D. Possible cephalopelvic disproportion E. No significance

459 B. Spalding sign is the overlapping of fetal skull bones, a radiographic evidence to establish fetal death.

A 27 year-old woman who has been amenorrheic for 12 weeks has an elevated serum HCG titer. D&C was performed on the patient due to an incomplete abortion. Pathology report was available describing a generalized trophoblastic proliferation, hydropic villi without blood vessels and fetal parts. Diagnosis is most likely: A. Choriocarcinoma B. Partial mole C. Complete mole D. Incomplete abortion E. Complete abortion

460 C. Complete mole is a dyspermic fertilization of an empty egg by one normal sperm. It is characterized by severe trophoblastic hyperplasia, hydropic or swollen chorionic villi and absent fetus and blood vessels.

A 3 year old child was noticed by her mother to frequently scratch her vulva. On examination, labia minora adheres in the midline with a translucent vertical line. What is the treatment? A. topical androgen B. topical combined estrogen and progesterone C. topical progesterone D. Topical estrogen E. Topical steroid

461 D. this is a case of adhesive vulvitis, in which the treatment is topical estrogen

A 32 year old G2P2 mother is on her 2nd week postpartum. She noticed vaginal secretions that are pinkish in color. What do you call this secretions? A. Lochia alba B. Lochia rubra C. Lochia serosa D. Normal secretions E. None of the choices

462 C. The 3 types of secretions during puerperium are as follows: lochia rubra - red color on days 1-3 postpartum; lochia serosa - more pale in color or pinkish on days 4-10; and lochia alba -white to yellowish white on days 10 to 4-8 weeks postpartum.

In preparation for labor, the uterus has increased responsiveness to uterotonins and increased contractility which is mediated primarily by: A. estrogen B. progesterone C. Beta HCG D. glycosaminoglycans E. Prostaglandins

463 A. In preparation for labor, estrogen is the principal mediator, while progesterone level decreases

It is defined as menstrual cycle occuring every >35days with normal flow. A. oligomenorrhea B. polymenorrhea C. hypomenorrhea D. menorrhagia E. Metrorrhagia

464 A. oligomenorrhea is defined as menstrual cycle occuring every >35 days with normal flow, while polymenorrhea is every <21 days. Hypomenorrhea is scanty menstruation, while menorrhagia is excessive heavy menstruation. Metrorrhagia is any bleeding between normal menses.

The nerve supply of suprapubic area comes from: A. Ilioinguinal nerve B. Iliolumbar nerve C. Iliohypogastric nerve D. Obturator nerve E. Pudendal nerve

465 C.

The following are the major criteria for polycystic ovarian syndrome: A. Chronic anovulation B. hyperandrogenemia C. Insulin resistance D. All of the above E. Only A and B

466 E. the major criteria for PCOS are chronic anovulation, hyperandrogenemia and exclusion of other causes. Insulin resistance is one of the minor criteria.

Round ligament is continuous with the broad ligament and extends from the lateral portion of the uterus. In relation to the oviducts, it is located: A. Below and posterior to origin of the oviducts B. Below and anterior to origin of the oviducts C. above and anterior to origin of the oviducts D. above and posterior to the origin of oviducts E. lateral to the origin of the oviducts

467 B. roud ligament arises below and anterior to the origin of the oviducts

A 55 year old nulligravid presents with amenorrhea for 6 months followed by irregular vaginal bleeding. Endometrial sampling done which shows complex hyperplasia with atypia. What is your management? A. Total abdominal hysterectomy with bilateral salpingo-oophoectomy B. high dose progestin C. hysterectomy D. progestin E. repeat endometrial sampling after 3 months

469 C. this is a case of endometrial hyperplasia. The management depends on the age, cytologic atypia and type of hyperplasia. In patients with complex hyperplasia with atypia in menopausal women, the management is hysterectomy, while for premenopausal women, it is high dose progestin.

What is the method used in breech delivery if the entire body is extracted by the doctor? A. Spontaneous breech delivery B. Total breech delivery C. Partial breech delivery D. Internal podalic version E. External cephalic version

470 B. When the entire body is extracted by the doctor, it is total breech delivery; it is partial if breech is delivered spontaneously as far as umbilicus, but the remainder of the body is assisted; it is spontaneous if the baby is expelled entirely without any traction other than support.

A 35 year old G5P5 smoker, hypertensive mother at 34 weeks AOG presents with vaginal bleeding associated with crampy abdominal pain. Upon abdominal palpation, there's extreme tenderness. What is your primary consideration? A. Placenta previa B. Placenta accreta C. Placenta abruptio D. vasa previa E. normal labor

471 C. painful vaginal bleeding after 20 weeks AOG with abdominal or uterine tenderness supports the diagnosis of abruptio placenta. Hypertension, multiparity, advanced age, and smoking are risk factors of abruptio placenta. Placenta previa on the other hand, will present with painless vaginal bleeding.

What is the most common type of cervical cancer? A. Squamous type B. adenocarcinoma C. squamoadenocarcinoma D. Transitional type E. None of the choices

472 A.

A 38 year old female presents with dysmenorrhea and heavy vaginal bleeding. Upon examination, you noted a diffusely enlarged uterus approximately 2x its normal size. What is your diagnosis? A. Endometriosis B. Pregnancy C. Myoma D. Adenomyosis E. none of the choices

473 D. adenomyosis is usually symptomatic, however it is symptomatic in women >35 years old presenting with dysmenorrhea and menorrhagia with classice pelvic exam of diffusely enlarged uterus 2-3x larger.

One of the following is not a fetal risk factor causing intrauterine growth restriction. A. Chromosomal abnormalities B. Congenital infection C. Tobacco D. Multiple gestation E. Congenital anomalies

474 C. Tobacco is a maternal risk factor, while the rest of the choices are fetal risk factors.

What is the single most important risk factor for the development of uterine infection? A. Duration of labor B. Route of delivery C. Duration of membrane rupture D. Number of internal vaginal examination

481 B. Cesarean section is the single most significant risk factor for post-partum infections.

Intake of oral contraceptive pills increases which type of cancer? A. cervical B. endometrial C. breast D. Ovarian

482 A. Studies show that the use of OCPs lead to a 2x increase in cervical cancer even when confounding factors such as age at first intercourse, number of sexual partners, exposure to HPV, cytologic screening, and the use of barrier methods were excluded. Some references say that this is due to OCPs effect in everting the transformation zone.

The following are known sequelae for babies of HBV-infected mothers, EXCEPT: A. Low birth weight B. Prematurity C. Congenital malformations D. There is no exception

483 C.

A sexually active female presents with a beefy red lesion with granulation tissue on her vulva. Which is the most likely diagnosis? A. chancre B. chancroid C. granuloma inguinale D. LGV

484 C. Granuloma inguinale or donovanosis is caused by Klebsiella granulomatis. Chancre is a painless ulcerated lesion of primary syphilis. Chancroid is a painful genital ulcer of H. ducreyi. LGV presents with buboes or enlarged, tender lymphnodes.

A prostitute presents at the emergency room with recurrent fever, foul smelling vaginal discharge and hypogastric pain. The gold standard to diagnose the most likely disease of this patient is: A. ultrasound B. laparoscopy C. laparotomy D. endometrial biopsy

485 B. The gold standard for the diagnosis of acute PID is laparoscopy.

How long does it take for the sperm to reach the egg during intercourse? A. 1 minute B. 30 minutes C. 6 hours D. 24 hours

486 B. SIMILAR TO PREVIOUS BOARD EXAM CONCEPT/PRINCIPLE. References say that the travel of the sperm from the vagina to reach the egg would take around 15-30 minutes.

At 20 weeks AOG, the fundus of the gravid uterus can be palpated: A. At the level of the symphysis pubis B. Midway between the symphysis pubis and umbilicus C. At the level of the umbilicus D. Within the pelvic cavity

487 C. 12 weeks AOG is at the level of the symphysis pubis, 16 weeks is midway between the symphysis pubis and umbilicus.

A patient at the OPD consulted for post-coital bleeding associated with foul-smelling discharge. On internal exmaination, the cervix is converted to a 5x5 cm nodular and friable mass with no extension to the vagina. The corpus is small. On rectovaginal examination, there was involvement of the parametria. What stage is she in? A. Cervical cancer stage 1B B. Cervical cancer stage 2B C. Cervical cancer stage 3B D. Cervical cancer stage 4B

488 B. Involvement of the parametria is the key phrase for stage 2B. SIMILAR TO PREVIOUS BOARD EXAM CONCEPT/PRINCIPLE. Study the recent FIGO staging.

The pathognomonic symptom of menopause is: A. Hot flushes B. Cessation of menses C. Atrophic vaginitis D. Osteoporosis

489 A. The rest of the choices are also symptoms of menopause.

A 7-year-old girl is seen by her pediatrician for left lower quadrant pain. You identified an ovarian tumor by ultrasound. Of the following, the most common ovarian tumor in this age group is? A. Dermoid cyst B. Germ cell tumor C. Fibrosarcoma D. Papillary serous adenoma

490 B. Germ cell tumors are more likely in the pediatric age group. Dermoid cysts or mature teratomas are more likely in the young adult.

The drug of choice for a patient complaining of dyspareunia, severe cyclical dysmenorrhea and infertility is: A. estrogen B. estrogen with progesterone C. leuprolide D. danazol

491 C. GnRH antagonists are the drug of choice for endometriosis.

A pre-eclamptic patient on prolonged labor has vaginal bleeding accompanied by non-reassuring fetal heart rate pattern. Which is the most likely complication which occurred? A. Placenta previa B. Uterine rupture C. Vasa previa D. Abruptio placenta

492 D. Preeclamptic patients are at risk for developing abruptio placenta.

A syndrome of multiple congenital anomalies including microcephaly, small palpebral fissues, short nose, cleft lip and plate and CNS abnormalities can be see if the mother, during pregnancy: A. Drinks alcohol B. Smokes cigarettes C. Takes phenytoin D. Is diabetic

493 A. This is a classic case of fetal alcohol syndrome.

The following should be monitored in patients being given magnesium sulfate, EXCEPT: A. Deep tendon reflexes B. Respiratory rate C. Urine output D. None of the above

494 D. DTR and RR should be monitored to detect signs of toxicity. Monitoring of urine output is vital because MgSO4 is renally excreted.

A 32-year-old G4P3 underwent suction curettage for hydatidiform mole. A chest xray was done revealing essentially normal findings. beta-HCG levels were high. Which is the most appropriate next step? A. Do a hysterectomy B. Monitor beta-HCG levels C. Give methotrexate prophylaxis D. Advice EMACO therapy

495 B. A similar trophoblastic disease question was asked during our board exam. Methotrexate prophylaxis can be considered for high-risk patients (e.g. Metastasis to other organs).

Depot medroxyprogesterone acetate can cause: A. Irregular menstrual bleeding B. Immediate fertility resumption C. Weight loss D. Infertility

496 A. Some disadvantages of DMPA include irregular menstrual bleeding, delay in fertility resumption and weight gain.

The phase normally comprising 95% of pregnancy is: A. Phase 1 B. Phase 2 C. Phase 3 D. Phase 4

497 A. Phase 1, which composes 95% of pregnancy, is characterized by maintenance of cervical anatomical and structural integrity.

A 35-year-old primigravid at 36 weeks AOG consulted for her prenatal care. She complained that her abdomen seems to be smaller and she feels as if "the baby dropped". This is termed as: A. descent B. engagement C. lightening D. Labor progression

498 C.

A 45-year-old G6P6 consulted at the OPD with a finding of LSIL in her Papsmear. The next step in the management of her case is: A. Total hysterectomy with bilateral salpingo- oophorectomy B. conization C. observation D. Colposcopy

499 D. Colposcopy is a diagnostic procedure to closely examine the cervix. This is the next step in order to evaluate an abnormal finding in papsmear.

The presence of a uterus and fallopian tubes in an otherwise phenotypically normal male is due to: A. Lack of mullerian-inhibiting factor B. Lack of testosterone C. Increased levels of estrogen D. Presence of ovarian tissue

500 A. The default sex is female, hence without the mullerian-inhibiting factor, the female internal organs would develop despite the predominant presence of testosterone.

This provides surgical access to the peritoneal cavity: A. Anterior fornix B. Right lateral fornix C. Left lateral fornix D. Posterior fornix E. All of the above

501 D. Topnotch handouts: posterior fornix- for culdocentesis

A 3 year old little girl was brought to the ER because of perineal discomfort and yellowish vaginal discharge. You suspect rape. During the first coitus, which part of the hymen would most likely rupture first? A. 12 o' clock position B. 3 o' clock position C. 6 o' clock position D. 9 o' clock position E. none of the above

502 C. Topnotch handouts

Which of the following is not true regarding perineal body? A. It anchors the anorectum and the vagina. B. It helps maintain urinary and fecal continence. C. It provides physical barrier between the vagina and rectum. D. It prevents expansion of the urogenital hiatus. E. None of the above.

503 E. also prevents expansion of the urogenital hiatus

This is the shortest distance between the promontory of the sacrum to the lower margin of the symphysis pubis that can be measured clinically. A. True conjugate B. Obstetrical conjugate C. Diagonal conjugate D. All of the above E. None of the above

506 C. diagonal conjugate has to >11.5cm to be adequate

This is the male homologue of the vagina: A. Prostate gland B. Prostatic utricle C. Bulbourethral gland D. Seminal colliculus E. Penis

507 B. A. Prostate gland - Skene's glands B. Prostatic utricle - vagina C. Bulbourethral gland- greater vestibular glands D. Seminal colliculus-hymen E. Penis - clitoris

A 6 year old girl was incidentally found to have a cystic right adnexal mass measuring 4 cm. What course of management should you advise the mother of patient? A. Observe for 8-12 weeks B. Do exploratory laparotomy C. Reassure mother D. Do serial Transabdominal ultrasound E. Do scout film of the abomen

508 B. Exploratory laparotomy is advised in premenarchal patients with adnexal mass >2cm. -topnotch handout

The following are definitive evidence of prenancy, except: A. Identification of fetal heart action. B. Perception of fetal movement by the doctor. C. Recognition fetus by sonographic exam. D. Beta- HCG titer of more than 1500 IU/L E. None of the above

509 D.

CTG tracing showed mirror images of uterine contractions and fetal heart rate deceleration. What caused this pattern? A. Fetal movement B. Head compression C. Umbilical cord compression D. Uteroplacental insufficiency E. Any of the above

510 B. Early decelerations- head compression Variable decelerations- umbilical cord compression Late deceleration-Uteroplacental insufficiency

Marcia 21 year old G1P0 14 weeks AOG was rushed to the hospital because of vaginal bleeding, watery vaginal discharge and hypogastric pain. Examination revealed cervix to be 4 cm dilated with no fetal cardiac activity on ultrasonography. What is your impression? A. Threatened abortion B. Inevitable abortion C. Complete abortion D. Incomplete abortion E. Missed abortion

511 B. Inevitable or imminent abortion

Maria, 19 year old G1P1, previously treated for PID 2 years ago was brought in the hospital because of severe hypogastric pain and vaginal spotting consuming 3 pads per day, lightly soaked. History revealed delayed menses for 3 weeks now. What is the gold standard procedure for the diagnosis of Ectopic pregnancy? A. Quatitative serum B-HCG B. Sonography C. Laparotomy D. Laparoscopy E. Culdocentesis

512 D.

The 3rd stage of labor includes the period from fetal delivery to placental expulsion. What does Calkin sign refer to? A. Uterus become globular and firmer B. Sudden gush of blood C. Uterus rise in the abdomen D. Lengthening of the umbilical cord E. None of the above

513 A. SIMILAR TO PREVIOUS BOARD EXAM CONCEPT/PRINCIPLE:all refer to the signs of placental separation

What is the therapeutic level of Magnesium sulfate? A. 5 mEq/L B. 10 mEq/L C. 12 mEq/L D. >15 mEq/L E. 30-35 mEq/L

514 A. SIMILAR TO PREVIOUS BOARD EXAM CONCEPT/PRINCIPLE: 4-7 mEq/L-therapeutic level; 10 mEq/L- disappearance of patellar reflex; >12 mEq/L- respiratory depression; >15mEq/L- respiratory depression with mechanical ventilation; >30-35-cardiac arrest

This syndrome result from meiotic nondysjunction leading to 47, XXY genotype, with associated testicular atrophy, eunuchoid body shape, tall stature, long extremities, gynecomastia and female hair distribution: A. Hermaphroditism B. Klinefelter Syndrome C. Turner Syndrome D. Androgen insensitivity E. Down Syndrome

515 B.

This is the first sign of puberty in females: A. Axillary hair B. Onset of menses C. Growth spurt D. Breast bud formation E. None of the above

516 D. Thelarche or breast bud formation is the first sign of puberty in females

This syndrome is characterized by hypothalamic- pituitary dysfunction caused by congenital absence of GNRH associated with anosmia: A. Savage Syndrome B. Kallman Syndrome C. Turner Syndrome D. Premature ovarian Failure E. None of the above

517 B. Kallman Syndrome- Hypogonadotropic hypogonadism characterized by isolated gonadotropin deficiency associated with anosmia

Shally is a 37 year old G2P2 who came in to your clinic because of absence od menses for the last 8 months. Her mother had her menopause at age 39. What is the most probable diagnosis of Shally's condition? A. Asherman syndrome B. Premature ovarian failure C. Polycystic Ovarian Syndrome D. All of the above E. None of the above

518 B. Asherman syndrome- intrauterine adhesion with history of previous endometrial curettage Premature ovarian failure- cessation of ovarian function before age 40

What is the treatment of choice for hot flushes of menopause? A. Progesterone supplement B. Leuprolide C. Estrogen D. Calcium + vitamin D E. DEXA

519 C.

A 19 year old primigravid at 40 weeks AOG undergoes fetal heart monitoring. Pregnancy has been uncomplicated. External monitoring shows a baseline heart rate of 140 bpm with good variability; over a period of 30 minutes, the rate increases twice to 160 bpm for 25 to 30 seconds. Which of the following is the most appropriate step in management? A) Reassurance B) Biophysical profile C) Oxytocin challenge test D) Induction of labor

521 A.

A 32 year old G2P1 at 40 weeks AOG is brought to the ER by her husband because she has been confused for 45 minutes. Her husband says that she has been in labor for 3 days at home, and she has received all her prenatal care from an alternative provider. On arrival, she is obtunded. Her pulse is 140 bpm, and palpable systolic blood pressure is 60 mmHg. Abdominal examination shows distention and rigidity and a 25-cm irregular, mobile mass in the upper right quadrant. The cervix is 3 cm dilated and 50% effaced. Which of the following is the most likely cause of these findings? A) Coagulopathy B) Uterine rupture C) Endomyometritis D) Uterine atony

522 B.

A previously healthy 23 year old G2P1 at 32 weeks AOG comes to the physician because of urinary urgency and pain with urination or 3 days. Her pregnancy has ben uncomplicated. Examination shows a uterus consistent in size with a 32 week gestation. There is mild suprapubic tenderness but no costovertebral angle tenderness on palpation. Urinalysis reveals numerous WBCs and RBCs and 3+ bacteria. The patient is at greatest risk for which of the following? A) Chorioamnionitis B) Renal failure C) Pyelonephritis D) Abruption placentae

523 C.

A 32 year old woman comes for follow-up examination 1 week after a Pap smear showed a high-grade squamos intraepithelial lesion. Examination shows no abnormalities. Which of the following is the most appropriate next step in management? A) Trichloroacetic acid therapy B) Colposcopic-direct biopsy C) Cone biopsy of cervix D) Repeat Pap smear

524 B.

An 82 year old woman comes to the physician because of a 9 month history of progressive urinary incontinence. At least once daily, she has a strong urge to void, is unable to reach a bathroom in time, and spontaneously passes a large amount of urine. She had a mild cerebral infarction 2 years ago with no residual weakness. She has severe arthritis of her knees and hips, which severely limits her mobility. Current medications include lovastatin and aspirin. Examination, including pelvic examination, shows no abnormalities. Which of the following is the most likely explanation for this patient’s incontinence? A) Detrusor hyperactivity B) Intrinsic weakness of the urethral sphincter C) Poor pelvic support D) Outflow obstruction

525 A.

An 18-year-old primigravid woman comes for her initial prenatal visit at 16 weeks' gestation. She is not sure about the date of her last menstrual period but says that the pregnancy probably occurred immediately after she stopped taking oral contraceptives 5 months ago. Maternal serum α- fetoprotein (MSAFP) level is increased to 3 multiples of the median. Which of the following is he most appropriate next step in management? A) Repeat measurement of MSAFP level B) Triple screening for MSAFP, serum β-hCG, and serum estriol levels C) Ultrasonography D) Amniocentesis for measurement of α- fetoprotein level

526 C.

A previously healthy 87-year-old woman comes to the physician because of a 4-month history of vulvar itching. Examination shows excoriated areas from scratching and a white, thin vulva. The labia minora are absent, and there are small fissures at the introitus. The remainder of the examination shows no abnormalities. Which of the following is the most likely diagnosis? A) Lichen planus B) Lichen sclerosus C) Squamous cell carcinoma D) Atrophic vaginitis

527. B..

Four hours after undergoing a cesarean delivery at term followed by tubal ligation, a 37-year-old woman, gravida 2, para 2, has dizziness and confusion. The operation was uncomplicated, and blood loss is estimated to be 800 mL. Patient- controlled epidural analgesia has been moderately effective for pain. Her blood pressure now is 80/40 mm Hg, decreased from 120/72 mm Hg intraoperatively, and pulse is 152/min, increased from 96/min intraoperatively. Breath sounds are decreased bilaterally. No murmurs are heard. Abdominal examination shows distention and tenderness. Bowel sounds are absent. The incision is intact with no drainage. She is disoriented to person, place, and time. Her hematocrit is 23%; preoperative hematocrit was 35%. Which of the following is the most likely cause of the hemodynamic changes? A) Epidural-related hypotension B) Insufficient intraoperative fluid replacement C) Postoperative intra-abdominal hemorrhage D) Supine hypotensive syndrome

528 C.

A healthy 24-year-old woman comes for a routine health maintenance examination. Menses occur at regular 28-day intervals and last 5 to 6 days. Her last menstrual period was 3 weeks ago. She takes no medications. Bimanual examination shows a 5- cm, mildly tender left adnexa. A pregnancy test is negative. Which of the following is the most appropriate next step in management? A) Repeat examination in 2 weeks B) Measurement of serum CA 125 level C) Measurement of serum α-fetoprotein level D) CT scan of the pelvis

529 A.

A 32-year-old woman at 38 weeks' gestation comes for a routine prenatal visit. During routine screening at 28 weeks' gestation, she tested positive for hepatitis B surface antigen. Her pregnancy has been otherwise uncomplicated. Examination shows a uterus consistent in size with a 38-week gestation. Which of the following measures is most likely to decrease the risk for hepatitis B infection in her newborn? A) Recommendation of bottle-feeding rather than breast-feeding B) Maternal administration of hepatitis B immune globulin (HBIG) now C) Neonatal administration of HBIG after delivery and hepatitis B vaccine at 3 months of age D) Neonatal administration of HBIG and hepatitis B vaccine immediately after delivery

530 D.

Five weeks after vaginal delivery of a healthy full- term newborn, a 22-year-old woman, gravida 1, para 1, is brought to the physician by her mother because of depressed mood for 2 weeks. Her mother is concerned that her daughter is not able to take care of her infant. Physical examination shows no abnormalities. She is quiet and tearful and does not engage in conversation easily. She states that she lives alone with her infant and has had thoughts of suicide and infanticide. Which of the following is the most appropriate next step in management? A) Reassurance B) Long-term outpatient counseling C) Admission to the hospital for treatment D) Selective serotonin reuptake inhibitor therapy

531 C. thoughts of suicide and infanticide >> indications for hospitalization

A 23-year-old woman has pain, cramping, and swelling of the right calf 3 days after an uncomplicated labor and delivery. The right foot is swollen, and there is marked tenderness with dorsiflexion and palpation of the right calf. Examination shows no other abnormalities. A complete blood count and serum electrolyte levels are within normal limits. Which of the following is the most likely cause of this condition? A) Hypercoagulable state of pregnancy B) Hyperuricemia C) Peripheral artery aneurysm D) Platelet embolus

532 A.

A 28-year-old nulligravid woman comes for a routine health maintenance examination. She has had progressively severe dysmenorrhea over the past 6 months adequately controlled by nonsteroidal anti-inflammatory agents. Pelvic examination shows a normal vagina and cervix. The uterus is retroverted and fixed, and there is nodularity of the cul-de-sac. A 6-cm left adnexal mass is palpated. Transvaginal ultrasonography shows a 7-cm septated adnexal mass. Four weeks later, there is no change in the size of the adnexal mass. Which of the following is the most appropriate diagnostic test? A) Measurement of serum CA 125 level B) Laparoscopy C) CT scan of the pelvis D) MRI of the pelvis

533 B.

A 32-year-old woman, gravida 2, para 2, comes to the physician because she has been amenorrheic for 4 months. Examination shows a well- estrogenized vagina and no evidence of virilization or other abnormalities. A serum pregnancy test is negative. She is given medroxyprogesterone and has the onset of bleeding 3 days later. Which of the following is the most likely cause of her condition? A) Anovulation B) Asherman's syndrome C) Hypopituitarism D) Premature ovarian failure

534 A. (+) Progestin challenge test >> anovulation

A healthy 37-year-old primigravid woman at 12 weeks' gestation comes for a routine prenatal visit. The pregnancy was achieved by in vitro fertilization. She does not use tobacco, alcohol, or drugs. She is a hematologist and works 10 to 12 hours daily. Two previous ultrasonographies have shown a triplet gestation. She weighs 66 kg (145 lb) and is 178 cm (70 in) tall. Her blood pressure is 116/70 mm Hg, and pulse is 72/min. Examination shows a uterus consistent in size with a 16-week gestation. Her pelvis is normal-sized. This patient is at greatest risk for which of the following? A) Abruptio placentae B) Hepatitis B C) HIV infection D) Preterm labor

535 D.

A 30-year-old woman, gravida 2, para 1, is brought to the emergency department in labor. An episiotomy is performed. Following delivery of the head, the shoulders do not follow with the usual traction and maternal pushing. Which of the following is the most appropriate next step in management? A) Flexing the woman's knees toward her shoulders B) More forceful traction and fundal pressure C) Delivering the posterior arm D) Symphysiotomy

536 A.

A 13-year-old girl is brought for a well-child examination. Menses have occurred every other month since menarche 10 months ago. Her last menstrual period was 1 week ago. She is not sexually active. Sexual development is Tanner stage 3. Examination shows no abnormalities. Which of the following is the most appropriate next step in management? A) Discussion of pregnancy prevention B) Measurement of serum luteinizing and follicle- stimulating hormone levels C) Urine β-hCG test D) Pelvic examination

537 A.

A 52-year-old woman comes to the physician because of decreased libido; this symptom began 8 months ago, after she underwent a total abdominal hysterectomy and bilateral salpingo-oophorectomy for leiomyomata uteri and menorrhagia. She has been taking hormone replacement therapy with conjugated estrogen since the operation. Examination shows a moist, rugated vagina. Which of the following is the most likely cause of these findings? A) Decreased androgens B) Decreased estrogen C) Decreased follicle-stimulating hormone (FSH) D) Decreased luteinizing hormone (LH)

538 A.

A 38-year-old woman, gravida 2, para 1, at 38 weeks' gestation has had no fetal movement for 36 hours. Her prenatal course, prenatal tests, and fetal growth have been normal. Fetal heart tones are heard by Doppler. Which of the following is the most appropriate next step in management? A) Routine prenatal visit in 1 week B) Maternal hydration C) Nonstress test D) Immediate induction of labor

539 C.

A 22-year-old primigravid woman at 16 weeks' gestation is brought to the emergency department because of progressive shortness of breath over the past 48 hours. Her temperature is 37 C (98.6 F), blood pressure is 120/70 mm Hg, pulse is 100/min, and respirations are 24/min. Scattered wheezes are heard. Pelvic examination shows a uterus that extends to the umbilicus. Her hematocrit is 32%, leukocyte count is 11,000/mm3, and serum β-hCG level is 300,000 mIU/mL. Pulse oximetry on room air shows an oxygen saturation of 92%. An x-ray film of the chest shows multiple round densities throughout all lung fields. Which of the following is the most likely diagnosis? A) Bacterial pneumonia B) Choriocarcinoma C) Pulmonary embolism D) Tuberculosis

540 B. cannon ball lesions

Which of the following bears the greatest risk for ectopic pregnancy? A. Previous ectopic pregnancy B. Tubal corrective surgery C. Infertility D. Previous genital infection E. Prior Abortion

541 A.

The appearance of stromal nodules in the prostate occurs in the: A. 2nd decade of life B. 3rd decade of life C. 4th decade of life D. 5th decade of life E. 6th decade of life

542 C.

A hot flush is usually followed by A. increased digital perfusion B. increased peripheral skin temperature C. decreased LH D. decreased HR E. decrease in cortisol

543 B.

Correct use of lactation amenorrhea EXCEPT A. Mother must breast feed day and night B. Baby is more than 6 months old C. Mother has not started menses D. Infant must receive at least 85% of feeding as breastfeeding E. None of the above

545 B. The use of lactation amenorrhea must first fulfill 3 criteria: exclusive or almost exclusively breastfeeding, have not had menses since giving birth, and mother should be less than 6 months post-partum.

A 28 year old patient G2P1 (1001) 39 weeks AOG CIL, is undergoing contraction stress test. You note that there are fetal heart rate decelerations that occur in the presence of contractions lasting longer than 90 seconds. You interpret this as: A. Negative CST B. Positive CST C. Equivocal-suspicious D. Equivocal-hyperstimulatory E. Unsatisfactory

546 D.

Which of the following INCORRECTLY describes the perineum? A. The pelvic diaphragm consists of the levator ani muscles plus the coccygeus muscles posteriorly. B. The levator ani muscles form a broad muscular sling that originates from the posteriorsurface of the superior pubic rami, from the inner surface of the ischial spine, and between these two sites, from the obturator fascia. C. The major blood supply to the perineum is via the internal pudendal artery and its branches. These include the posterior rectal artery and inferior labial artery. D. The innervation of the perineum is primarily via the pudendal nerve and its branches. The pudendal nerve originates from the S2, S3, and S4 level of the spinal cord.

547 C. there is no posterior rectal and inferior labial arteries. Its supposed to be inferior rectal and posterior labial

A 34 year old G2P1 (1001) patient on her 28th week APG came in at the OBAS for consult. She was referred from the OPD due to BP of 140/90. According to the patient, she has been diagnosed with hypertension since she was 32 yo. Her stat albumin is (-). the diagnosis is A. Gestational Hypertension B. Chronic Hypertension C. Preeclampsia, mild D. Preeclampsia, severe E. Eclampsia

548 A.

L.Q. a 28 yo nulligravid, desirous of pregnancy, presents with 7x4x3 cm myomatous growth by UTZ. She is symptomatic with intermittent colicky hypograstric pain but with no menstrual disturbance. What would be the best management for her? A. hysterectomy B. Myomectomy C. Observation D. Conceptual trial E. Pain relievers

549 B.

Aside from health benefits, the use of OCP has been proven to reduce the incidence of: A. thromboembolic events B. Colon CA C. Benign Breast D. Endometrial CA E. Cervical Ca

550 D.

R.L. 36 year old G1P1 (1001) is 2 weeks post partum. She consults her physician for tenderness of the breast on the upper outer quadrant of the left breast. On Physical examination, it was noted erythema and warmth on the area. Which of the following conditions most likely explains her problem? A. breast abscess B. nipple trauma C. breast cyst D. acute mastitis E. inflammatory breast ca

551 D.

A 32 yo primigravid on her 34th wk AOG came to your clinic for lots of spider veins in the abdomen. What would you advice her? A. she has to have liver function test done to rule out liver disease B. refer her to dermatology for further work-up and evealuation C. This is normal during pregnancy and resolves spontaneously after delivery D. she needs abdominal support to relieve pressure E. Work up for possible liver disease

552 C.

Which of the following is NOT a documented outcome of uncontrolled thyrotoxicosis in pregnancy? A. Preeclampsia B. Preterm delivery C. Still Birth D. Neonatal Goiter

553 D.

A 34 yo G2P1 came to OBAS for labor pains. She has no records of prenatal check-up and was noted to have BP elevations during her 3rd preganancy. She gives you an AOG based on LNMP of 33 wks and 6 days. On PE: BP 160/90 with grade I bipedal edema. Urinalysis reveals albumin +1. On IE, she has a patulous cervix with hard ballotable mass. What is your admitting impression? A. PU 33 6/7 by LNMP, cephalic, NIL, preeclampsia, mild B. PU 33 6/7 by LNMP, cephalic, NIL, chronic HPN w/ superimposed preeclampsia C. PU 33 6/7 by LNMP, cephalic, NIL, gestational HPN D. PU 33 6/7 by LNMP, cephalic, NIL, gestational HPN w/ superimposed preeclampsia E. PU 33 6/7 by LNMP, cephalic, NIL, chronic hypertension

555 A.

A 29 year old female G2P1 (1001) experiences biliary colic for the past 1 week. She is diagnosed to have acute cholecystitis. How do you treat this case? A. Treat as if the patient is not pregnant B. Wait until the fetus is viable then perform surgery C. Medical management and perform intervention after delivery D. Delay treatment and perform surgery at second trimester

556 C.

A 26 year old G1P0 PU 39 weeks AOG CIL, is noted to have a BP of 160/100. You have managed the hypertension by give hydralazine. The next step is toprevent eclampsia by giving magnesium sulfate. How will you administer magnesium sulfate in this patient? A. Loading dose of 4g IV and 5g on each buttock then 5g in alternating buttock every 4 hours to complete a cycle of 24 hours B. Loading dose of 4g IV and 5g on each buttock then 5g in alternating buttock every 4 hours upto 24 hours post delivery C. Loading dose of 4g IV and 5g on each buttock then 5g in alternating buttock every 4 hours until delivery D. Loading dose of 4g IV then 5g in alternating buttock every 4 hours until delivery

557 B.

A 38y y.o. G7P5 came in a stretcher. She appears to be pale, incoherent with cold, clammy extremities. There is note of profuse vaginal bleeding. History from the husband: came from a lying-in clinic where the midwife performed fundal pressure. Rigid abdomen, (-) FHT. What could the patient have? A. Bleeding placenta previa B. Abruptio placenta with full blown DIC C. Ruptured vasa previa D. Uterine rupture

558 D.

A 35 y.o. G1P0 37wks AOG with a history of chronic hypertension presents with severe abdominal pains for several hours. BP 140/90, PR 110, RR 18, T 37C, tender abdomen, cervix 4cm dilated, membranes bulging. CTG is non reactive with hypertonic contraction. Diagnosis? A. Intrauterine infection B. Normal early labor C. Ruptured uterus D. Abruptio placenta

559 D.

What is the complete obstretic score of the patient given the following history? 1. 1970 FT via SVD male, alive 2. 1971 Spontaneous abortion 15 weeks AOG 3. 1980 H. mole 4. 1985 Preterm twin gestation via SVD, both females, 1 died after 1 yr, the other, alive 5. 1986 FT via SVD male, FDU 6. 1987 Ectopic pregnancy 7. 1990 Present pregnancy A. G7P3 (1232) B. G7P3 (2132) C. G7P3 (1232) D. G7P3 (2133)

560 B.

What is the most common presenting feature of adult hypopituitarism? A. Hypogonadism B. Hypothyroidism C. Hypoprolactinemia D. Short stature

561 A. Most common feature of adult hypopituitarism even when other pituitary hormones are deficient Trophic hormone failure associated with pituitary compression or destruction usually occurs sequentially: GH>FSH>LH>TSH>ACTH Childhood: Growth retardation is often the presenting feature Adult: Hypogonadism is the earliest symptom

Most important step in the evaluation of male infertility A. Serum testosterone B. HCG stimulation test C. Semen Analysis D. Testicular biopsy

562 C.

Fetal lung maturity is expected to be completed at: A.) 28 wks AOG B.) 30 wks AOG C.) 34 wks AOG D.) 32 wks AOG

563 C.

Around the middle of the menstrual cycle, the thickening of the cervical mucus is brought about by this hormone: A.) progesterone B.) estrogen C.) oxytonin D.) FSH

564 A.

During hysterectomy, ligating this ligament would cause injury to the uterine artery, vein and ureter. A.) cardinal ligament B.) ligament of Treitz C.) Mackenrodt's ligament D.) A & B E.) A & C

565 E. Cardinal ligament aka Mackenrodt’s ligament

In a fully mature fetal lung, the expected Lecithin/Sphingomyelin ratio should be at least: A.) 1.75 B.) 2.0 C.) 1.0 D.) 1.5

566 B.

Kassandra, a 38-year old G7P6 (6006) just gave birth to a healthy 2.7 kg baby boy via spontaneous vaginal delivery. The Obstetrician on duty noticed continued vaginal bleeding despite absence of any vaginal or cervical lacerations. What is the most prudent thing to do to stop the bleeding? A.) continue packing with OS B.) nipple stimulation C.) administer Oxytocin drip D.) IV fluids and vasopressors

567 C.

On endometrial biopsy, you saw saw-toothed glands, edematous stroma, prominent arterioles and glycogen vacuoles at the cell surfaces. What phase of the menstrual cycle is this? A.) Luteal B.) Proliferative C.) Secretory D.) Follicular

568 C. secretory phase of the menstrual cycle since the presence of glycogen already marks the ovulatory phase

The most common cause of benign vaginal bleeding among post menopausal women is: A.) leiomyosarcoma B.) Atrophic vaginitis C.) foreign body vaginitis D.) infectious vaginitis

569 B.

Mary, a 32-year old non-diabetic and non- hypertensive, is currently pregnant with her second child (at 34 wks AOG) suddenly experienced non-painful vaginal bleeding while asleep. Her husband Roel, woke her up in a pool of blood and accompanied her to the nearest private hospital. Roel is anxious to know the cause of her bleeding. As the physician on duty, your primary diagnosis is: A.) abruptio placenta B.) placenta previa C.) pre-eclampsia D.) inevitable abortion

570 B.

Which of the following is responsible for production of beta HCG? A. Syncytiotrophoblast B. cytotrophoblast C. Decidua D. Endometrial glands

571 A.

What is the histologic date of an endometrium showing subnuclear vacuoles and pseudostratified nuclei? A. Day 16 B. day 18 C. Day 20 D. Day 22

572 A.

Which of the following is an expected feature in the placenta of a mother with severe preeclampsia? A. Neutrophilic infiltrates B. granulation C. Infarcts D. Chronic inflammatory pattern

573 C.

The most common predisposing factor to tubal pregnancy? A.Chronic salphingitis B. Congenital tubal defects C. Functional tubal disturbances D. Salphingitis isthmica nodosa

574 A.

As the villi of placenta mature, they begin to form vessels. At how many weeks do the villous vessels become apparent? A. 4 B. 5 C. 6 D. 7

575 C. Answer: 6 weeks

Your patient comes to you 28 weeks pregnant. She asked if she can go to Tacloban by plane to visit her relatives who have been victims of the typhoon Yolanda. Till how many weeks is she considered safe to travel by air? A. 30 B. 32 C. 34 D. 36

577 D.

Strawberry cervix/colpitis macularis. What is the treatment? A. Metronidazole B. clotrimazole C. Ciprofloxacin D. Fluconazole

578 A. This is a case of trichomoniasis treated with Metronidazole

Which of the following vaccines has established evidence of causing harm to the fetus? A. Rubella B. varicella C. Measles D. Smallpox

579 D.

In the menstraul cycle, what is the earliest sign that the endometrium is in its secretory phase? A. Formation and coiling of the spiral arteries B. Decidualization of the endometrium C. Luteinization of the corpus and secretion of progesterone D. Ovum has reached the fallopian tube E. Presence of subnuclear glycogen vacuoles

582 E. Presence of basal vacuolization in the endometrium is the earliest sign of progesterone action in the endometrium. Luteinization of the corpus and ovum reaching the tubes comes before the vacoulization but are not signs that the endometrium has reached the secretory phase. They are signs that the ovaries has reached the luteal phase.

Which of the following is a presumptive symptom of pregnancy? A. Anatomic breast changes B. Perception of fetal movement C. Increased in basal body temperature D. Englargement of the abdomen E. Braxton-Hick's contractions

583 B. Anatomic breast changes and thermal signs are presumptive signs, while enlargement of the abdomen and Braxton Hick's contraction are painless and irregular.

In assessing results of fetal heart monitoring, what is the single best indicator that the fetus is well? A. Reactive Non-stress test B. Negative Contraction stress test C. Presence of accelaration D. Absence of deceleration E. Moderate variability

584 E. The presence of moderate variability (beat to beat variation) will tell us that the fetus is not compromised more than a reactive NST or negative CST.

When the fetal head is engaged, this tells us that the fetal head has pass through which narrowest diameter? A. True/Anatomic conjugate B. Obstetric conjugate C. Diagonal conjugate D. Interspinous diameter E. All of the above

585 B. The shortest and narrowest dimensions that the fetal head must pass through for engagement to occur is the obstetric conjugate. The ischial spine is at the level of the presenting part but the fetal head has not pass through it yet.

A 32 year old primigravid went to your clinic complaining of loss of perceived fetal movements. The fetus is 33 3/7 weeks AOG by early ultrasound. You did a biophysical profile (BPP) with a score of 6 with normal AFV. Lung maturity testing revealed L/S ratio of 2. What is your next step in management? A. Observe and reassure the patient B. Admit the patient and repeat BPP after 24 hours. If on repeat shows improvement, do expectant management and repeat per protocol C. Admit the patient, give Dexamethasone and repeat BPP daily. If not improving, deliver. D. Deliver immediately E. Refer to a perinatologist

586 D. Deliver immediately since we have already documented lung maturity (L/S ratio of 2) and with a BPP of 6, the fetus may be chronically asphyxiated.

Which of the following scenarios will it be LESS likely to form the pathologic retraction ring of Bandl? A. Precipituous labor B. Cephalopelvic disproportion C. Prolonged 2nd stage of labor D. Labor augmentation E. Nulliparous women

587 A. The longer is the duration of labor, the more prone is the uterus to form the pathologic band, precipituous labor would have lesser chance of having this complication.

Which of the following statements regarding multifetal gestation is TRUE? A. Fertilization of two different ova forms fraternal twins. B. Identical twinning has been known to have a hereditary basis. C. Dichorionic diamniotic twins have higher morbidity than those enclosed in one chorion. D. Ischiophagus is the most common type of conjoined twinning. E. Pregnant mothers of twins have a higher incidence of GDM than other pregnant women.

592 A. Fraternal twins have genetic basis, identical twins is an abnormal event. Dichorionic twins have the least complications. Thoracophagus is the most common form of conjoined twins. Mothers have a higher rate of preeclampsia, hypertension or premature birth than other pregnant population groups.

The following statements regarding pre-eclampsia is true EXCEPT? A. The visual disturbances of pre-eclampsia such as serous retinal detachment improves several weeks after the onset of symptoms. B. The most consistent anatomic finding in pre- eclampsia is glomerular capillary endotheliosis in the kidneys. C. The main pathogenesesis of pre-eclampsia is vasospasm and endothelial cell activation leading to increase sensitivity to vasopressors. D. Pathognomonic lesion in eclampsia is the presence of periportal hemorrhagic necrosis in the liver E. Pre-eclampsia tends to be more severe as the parity of women increases.

593 E. Being nulliparous is a risk factor for pre-eclampsia.

A mother brought her 15 year old daughter because of absence of menarche. Physical examination of the child is normal. Sexual maturity index is 3. What is your management? A. Reaasure mother and observe B. Measure FSH and LH C. Request for a transrectal ultrasound D. Karyotyping E. Cranial MRI

594 A. Reasure mother, primary amenorrhea is defined as no menses by age 16 with the presence of sexual characteristics. The patient exhibited secondary sexual characteristic and it will just be a matter of time till menarche sets in.

A couple came in your office who wants to consult regarding fertility issues. They have been trying to conceive for over a year now, however they were unsuccessful. The wife narrated that she was the eldest of 3 siblings and her mother had the same problems and was able to conceive only after 3 years of marriage with consult to a reproductive endocrinologist. Physical examination of the couple is unremarkable. What is your initial procedure in this couple's work-up for infertility? A. Do Semen Analysis B. Request for a Hysterosalpingogram C. Request for a Transvaginal ultrasound D. Do Clomiphene citrate challenge test E. Perform Laparoscopy

595 A. In any work-up of infertility, one must do a semen analysis first no matter what history the husband or wife give. Rationale: Easier to rule out a male factor than a female factor in the extensive work-up of infertility.

A 25 year old model with multiple sexual partners came to your office complaining of vaginal pruritus and discharge accompanied by intermittent pelvic pain. TVS showed normal anatomy of the uterus, oviducts and ovaries. Speculum exam was done which showed mucopurulent discharge with cervical tenderness. Gram stain revealed Gram(-) intracellular diplococci. What is your next step? A. Treat with 250 mg IM Ceftriaxone B. Treat with 1g Azithromycin C. Do Laparoscopy D. Use both ceftriazone and azithromycin E. Do surgical debridement

596 D. Treat with both, you assume there is concomittant chlamydial infection with gonorrheal PID.

Which will confer the highest risk factor for the development of cervical cancer? A. Multiple sexual partners B. HPV 6 infection C. Intercourse at an early age D. Genetic predisposition E. Exophytic mass on ectocervix

597 E. It is known that HPV infection confers the highest risk but it should be the oncogenic serotypes and the presence of a mass in the cervix will confer the highest risk for malignancy

Which ovarian tumor has a higher chance of concommitant endometrial hyperplasia? A. Serous epithelial tumor B. Endometrioid epithelial tumor C. Granulosa-theca cell tumor D. Dysgerminoma E. Meigs syndrome

598 C. Granulosa theca cell tumor are funcional and secrete estrogen, so there is a risk for endometrial hyperplasia or carcinoma.

Which parameter is not included in the Sassone criteria in diagnosis of ovarian malignancy by ultrasound? A. Inner wall structure B. Wall thickness C. Size of mass D. Septa E. Echogenicity

599 C. Size of mass is not one of the 4 criteria of Sassone scoring.

Which of the following statements is/are true regarding the normal menstrual cycle : A. It has an average blood loss of 100 ml B. The average interval between cycles is 28 days C. The average duration of flow is 7 days D. Luteal phase has a variable duration, while secretory phase is constant E. All of the above

602 B. The average interval between cycles is 28 days. A normal menstrual cycle lasts from 21 to 35 days (average of 28 days, +/- 7 days) with 2 to 6 days of flow (average of 4 days) and with a blood loss of 30 - 80 ml (average of 50 ml). Studies of large numbers of normally cycling women have shown that only approximately 2/3 of adult women have cycles lasting 21 - 35 days. Duration of luteal phase is always constant (14 days), while that of the secretory phase is variable.

A 34-year old G4P3 (3003) was sent to the OR for an emergency CS. Aftter repair of the uterus, the obstetrician noted that the left ovary has a cyst with a honeycomb appearance. The physician should: A. Do unilateral oophorectomy B. Obtain a biopsy specimen of the cyst C. Refer intra-op to general surgery for possible presence of gastric CA D. Leave the cyst and do follow-up ultrasound E. Proceed with TAHBSO

603 D. Theca-lutein cyst is common in pregnancy. It is a benign cyst that usually presents with honeycomb appearance on gross inspection. This cyst spontaneously regresses after completion of pregnancy and menstruation resumes.

A 26-year old G1P1 (1001) seeks consult for contraception. She has no co-morbidity and had no complication during her previous pregnancy. Which of the following will be most effective for this patient? A. Intra-uterine device (IUD) B. Spermicidal agent C. Condom D. Oral contraceptives E. Coitus interruptus

604 D. Method effectiveness refers to the pregnancy rate of 100 women using a particular contraceptive method correctly for 1 year (100 women-years of use). The effectiveness reflects failures due to patient misuse of the contraceptive method and is less than method effectiveness. Oral contraceptive is proven by studies to be most effective among female patients when properly used. Coitus interruptus is an unreliable contraceptive method in which the penis is withdrawn from the vagina prior to ejaculation.

Inlet contraction can be diagnosed with which of the following clinical finding: A. Obstetric conjugate of less than 11.5 cm B. Diagonal conjugate of less than 11.5 cm C. Interspinous diameter of less than 10.5 cm D. Sidewalls that are convergent E. All of the above

605 B. The obstetric conjugate is the most important anteroposterior diameter of the inlet which is the shortest distance between the sacral promontory and the mid posterior aspect of the symphysis pubis through which the fetal head must pass. It measures 10 cm or more but may be considerably shortened in abnormal pelvis. The obstetric conjugate cannot be measured directly with the examining finger. Measurement is indirectly done by getting the diagonal conjugate which is the distance between the sacral promontory and inferior margin of the symphysis pubis and subtract 1.5 to 2 cm. The diagonal conjugate length may vary depending on the height and inclination of the symphysis pubis. Therefore inlet contraction is usually defined as a diagonal conjugate of less than 11.5 cm. Obstetric conjugate in inlet contraction should be less than 10cm.

Cytomegalovirus is the most common cause perinatal infection. This may be transmitted by which of the following routes : A. droplet infections B. coitus C. uteroplacental D. all of the above E. A and C only

606 D. CMV is the most common cause of perinatal infection. This is transmitted horizontally by droplet infection and contact with saliva and urine, vertically from mother to fetus, infant and as a sexually transmitted disease. Williams, 21th ed, p 1468.

A 33-year old G0 came to your clinic complaining of genital lesions. Suspecting syphilis, the most appropriate drug therapy is: A. Ceftriaxone 1gm IM single dose B. Erythromycin 500 mg orally 4 x 1 day for 7 days C. Benzathine Penicillin G, 2.4 million units IM single dose D. Cefoxitin 2 gms IM single dose

607 C. Pen G is the recommended treatment for adult with primary, secondary or early latent syphilis. Patients with latent (patients seroactive without evidence of disease) syphilis should be given 3 doses of 2.4 million. Benzathine Pen G IM, at 1-week intervals.

A 19 year old G1P0 PU 40 weeks, not in labor, was seen at the OPD for decreased fetal movement. She was hooked to an electronic fetal monitor and tracing showed: Baseline FHT- 140’s, good variability, with more than 2 accelerations of 20 bpm lasting for 20 secs. The tracing is interpreted as: A. reactive B. non-reactive C. positive D. negative E. inconclusive

608 A. Reactive non-stress test biophysical scoring (8-10) shows normal findings and indicates favorable fetal survival. A non-reactive non-stress test (< 6) refers to poor fetal condition and may be an indication for an emergent delivery.

An obstetrician who attempts to do forceps extraction with the knowledge that there is certain degree of disproportion at the level of the midpelvis is doing a: A. Failed forceps B. Prophylactic forceps C. Elective forceps D. Trial forceps E. Structural forceps

609 D. In trial forceps, the OPB is fully aware that vaginal delivery may not be successful. In elective forceps, vaginal delivery is possible even without operative delivery.

A patient underwent THBSO omentectomy and lymph node dissection for an ovarian cancer. Histopathological report showed the tumor in the right ovary measured 10x8x8 cm ruptured with extension to the fallopian tube and omentum. One para aortic node showed malignant cells. Using the FIGO staging, this patient can be categorized under: A. Stage II-C B. Stage III-A C. Stage III-B D. Stage III-C

610 D. Stage I Growth limited to the ovaries. IA Growth limited to one ovary; no ascites present containing malignant cells. No tumor on the external surface; capsule intact. IB Growth limited to both ovaries; no ascites present containing malignant cells. No tumor on the external surfaces; capsules intact. IC Tumor either stage IA or IB but with tumor on surface of one or both ovaries; or with capsule ruptured; or with ascites present containing malignant cells; or with positive peritoneal washings. II Growth involving one or both ovaries with pelvic extension. IIA Extension and/or metastases to the uterus and/or tubes. IIB Extension to other pelvic tissues. IIC Tumor either stage IIA or IIB, but with tumor on surface of one or both ovaries; or with capsule(s) ruptured; or with ascites present containing malignant cells; or with positive peritoneal washings. III Tumor involving one or both ovaries with peritoneal implants outside the pelvis and/or positive retroperitoneal or inguinal nodes. Superficial liver metastasis equals stage III. Tumor is limited to the true pelvis but with histologically proven malignant extension to small bowel or omentum. IIIA Tumor grossly limited to the true pelvis with negative nodes but with histologically confirmed microscopic seeding of abdominal peritoneal surfaces. IIIB Tumor of one or both ovaries with histologically confirmed implants of abdominal peritoneal surfaces, none exceeding 2 cm in diameter. Nodes are negative. IIIC Abdominal implants greater than 2 cm in diameter and/or positive retroperitoneal or inguinal nodes. IV Growth involving one or both ovaries with distant metastases. If pleural effusion is present, there must be positive cytology to allot a case to stage IV. IVA Parenchymal liver metastasis equals stage IV

In the above case, what is the chance that the patient may still live for the next 5 years? A. 11.1% B. 23.4% C. 36.7% D. 41.1% E. 55.1%

611 B. Stage IA 86.9%, IB 71.3%, IC 79.2%, IIA 66.6%, IIB 55.1, IIC 57.0, IIIA 41.1, IIIB 24.9, IIIC 23.4, IV 11.1 (Comprehensive Gynecology, 4th ed)

The following statement is TRUE about Polycystic Ovarian Syndrome, EXCEPT: A. This disorder begins perimenarcheally B. Clinical manifestations include hirsutism, menstrual irregularity and obesity C. Ovaries contain multiple (>10) small (2- to 10- mm diameter) follicles in the periphery D. Sonographic findings also include echogenicity of the stroma and ovarian enlargement E. None of the above

612 E. Polycystic Ovarian Syndrome (PCOS) is an endocrinologic disorder characterized by excessive ovarian androgen production, abnormal gonadotrophin secretion, and chronic anovulation with morphologic changes in the ovary consisting of multiple small subcapsular follicles, increased amounts of stromal tissue, and ovarian enlargement. This disorder begins perimenarcheally, and its clinical manifestations include hirsutism, menstrual irregularity (oligomenorrhea or amenorrhea), and obesity. Findings upon sonographic visualization of the ovaries include multiple (>10) small (2- to 10-mm diameter) follicles in the periphery, increased echogenicity of the stroma, and ovarian enlargement. (Comprehensive Gynecology, 4th ed)

The major androgen produced by the ovaries is: A. Testosterone B. Dehydroepiandrosterone sulfate C. Androstenedione D. Etiocholanolone E. The ovaries do not produce androgens

613 A. The sources of androgen production in the human female are the ovaries and the adrenal glands. The major androgen produced by the ovaries is testosterone and that of the adrenal glands is dehydroepiandrosterone sulfate (DHEA-S). (Comprehensive Gynecology, 4th ed)

During active labor, the fetus was observed to have episodes of decreased fetal heart rate and return to baseline in association with a contraction. The degree of deceleration is generally proportional to the contraction strength and does not fall below 100 beats/min.Which of the following can be attributed to this phenomenon? A. Fetal hypoxia B. Utero-placental insufficiency C. Umbilical cord occlusion D. Head compression E. A and B

614 D. Early deceleration of the fetal heart rate consists of a gradual decrease and return to baseline associated with a contraction. Freeman and co- authors (2003) defined early decelerations as those generally seen in active labor between 4 and 7 cm dilatation. In their definition, the degree of deceleration is generally proportional to the contraction strength and rarely falls below 100 to 110 beats/min or 20 to 30 beats/min below baseline. Importantly, early decelerations are not associated with fetal hypoxia, acidemia, or low Apgar scores. Ball and Parer (1992) concluded that fetal head compression is a likely cause of early deceleration. Head compression probably causes vagal nerve activation as a result of dural stimulation and that mediates the heart rate deceleration (Paul and co-workers, 1964). (William's obstetrics)

The following are associated with increased risk of placenta previa, EXCEPT: A. Advance maternal age B. Previous caesarean section C. Multiparity D. Smoking E. Family history of placenta revia

615 E. According to William's obstetrics, placenta previa may be assocaiated with 4 etiologies: advanced maternal age, multiparity, prior CS and smoking.

A 26-year old G1P0 at 39 weeks AOG was noted to be 1 cm dilated but without regular uterine contractions during her pre-natal check up. If her obstetrician decides to admit her and plans to do an early amniotomy, the type of amniotomy would be a/an: A. Elective amniotomy B. Amniotomy induction C. Amniotomy augmentation D. Therapeutic amniotomy E. Late amniotomy

616. B..Types of amniotomy include: elective amniotomy, amniotomy induction and amniotomy augmentation. Artificial rupture of the membranes can be used to induce labor, but it implies a commitment to delivery. The main disadvantage of amniotomy when used alone for labor induction is the unpredictable and occasionally long interval to the onset of contractions. In a randomized trial, Bakos and Backstrom (1987) found that amniotomy alone or combined with oxytocin was superior to oxytocin alone. Mercer and colleagues (1995) randomized 209 women undergoing oxytocin induction to amniotomy either at 1 to 2 cm dilatation (early amniotomy) or at 5 cm (late amniotomy). Early amniotomy was associated with significantly shorter labor by approximately 4 hours. There was, however, an increased incidence of chorioamnionitis and cord-compression patterns with early amniotomy.

The most common malignancy gicing rise to a placnetal metastasis is: A. Gastric carcinoma B. Pilocytic astrocytoma C. Rhabdomyosarcoma D. Lung cancer E. Melanoma

617 E. Malignant tumors rarely metastasize to the placenta. Of those that do, melanoma accounts for nearly one third of reported cases, and leukemias and lymphomas comprise another third.

A 3 year-old girl was brought to the ER because of vaginal bleeding. On inspection of the external genitalia, there was a mass protruding through the vaginal introitus. It measured 4x3 cm and resembled a cluster of grapes forming multiple polypoid masses. The most likely diagnosis is: A. Sarcoma botyroides B. Epidermoid carcinoma C. Vaginal adenosis D. Condyloma acuminata E. Congenital vulvar malformation

618 A. Sarcoma Botryoides (Embryonal Rhabdomyosarcoma) is usually diagnosed in the vagina of a young female. Rarely does it occur in a young child over 8 years of age, although cases in adolescents have been reported. The most common symptom is abnormal vaginal bleeding, with an occasional mass at the introitus. The tumor grossly will resemble a cluster of grapes forming multiple polypoid masses.

Which of the folloqwing is true regarding maternal mortality in the Philippines? A. Fifty (50%) percent of the women who died had prenatal care B. Hypertension is the leading cause of mortality C. Pulmonary disease is the leading medical cause D. Highest rate is seen in the 40-44 years old age group E. Cervical cancer remain as the second leading cause of maternal mortality

623 D. Highest rate is seen in age group 40-44.

The first index/indices of declining ovarian function in perimenopausal women is/are: A. Decrease in FSH B. Decrease in LH C. Decrease estradiol D. Decrease inhibin E. A and C

619 D. Inhibin is a glycoprotein produced by the granulosa cells of developing follicles during the follicular phase of the cycle. The gonadal production of inhibin is stimulated by FSH, and inhibin suppresses pituitary FSH secretion as part of a closed-loop feedback system. MacNaughton et al. have shown that circulating follicular phase inhibin levels are significantly lower among women aged 45 to 49 than among women younger than age 45 (Table 42-1) . The fall in inhibin levels may be due to the decreased number of ovarian follicles or to altered granulosa cell function that accompanies increasing age. Because estradiol levels do not undergo a similar significant decrease between ages 45 and 49, it is possible that synthesis of these two hormones are a result of separate functions of the granulosa cells. As inhibin levels fall there is a concomitant rise in FSH, which initially results in greater secretion of estradiol from the follicle.

A 33 year-old G3P2 (2002) went to OPD due to spotting. Her LMP was 28 weeks ago and she claims that she has not felt any fetal movement for the past month. On examination, fundal height is 18 cm, no fetal heart tone detected. Ultrasound was done, revealing no cardiac acitivity. Management of this patient would be: A. Wait for spontaneous labor B. Induction of labor C. Dilatation & Curettage D. Hysterotomy E. Prostaglandin

620 A. Waiting for spontaneous labor is the treatment of choice for intra-uterine fetal death (IUFD). Induction of labor may be done 72 hours without spontaneous labor.

In the clinical pelvimetry of a pateint in the third trimester of pregnancy, which of the following can be measured clinically? A. Transverse diameter of the inlet B. Obstetric conjugate C. Interspinous diameter D. Posterior sagittal diameter of the outlet E. None of the above

621 C. Interspinous diameter and Diagonal conjugate can be measured clinically.

The iron deficiency anemia of pregnancy is due to: A. Expected decreased absorption of iron from the GI tract during pregnancy B. Nausea and vomiting that causes dehydration and electrolyte imbalance C. Expansion of plasma volume without normal expansion of maternal hemoglobin mass D. Decreased expansion of RBC mass because of decreased erythropoietin activity during pregnancy E. Any of the above choices

622 C.

At what age of gestation is scanning for congenital abnormalities best done? A. 5-6 weeks B. 10-14 weeks C. 16-18 weeks D. 24-28 weeks E. 30-32 weeks

624 C. congenital scanning is usually done at 16-18weeks

The following are components of quadruple marker screen, EXCEPT? A. AFP B. B-HCG C. Estriol D. Estradiol E. Inhibin

625 D. Quadruple screen- AFP, BHCG, Estriol and Inhibin

Th e following structures originate from ureteric bud/ metanephric duct , EXCEPT? A. Ureter B. Renal pelvis C. Calyces D. Collecting system E. Renal glomerulus

626 E. glomerulus originates from metanephric mesenchyme.

In females, what is the remnant of mesonephric duct/ wolffian duct ? A. Gartner's Duct B. Bartholin's Gland C. Skene's gland D. Uterus E. Nabothian cyst

627 A. Gartners duct is a remnant of mesonephric duct

The most sensitive test for the diagnosis of abdominopelvic tuberculosis is? A. Tissue biopsy B. Peritoneal fluid culture C. ELISA D. Laparoscopy E. Exploratory laparotomy

628 D. Laparoscopy is the gold standard in the diagnosis of abdominopelvic tuberculosis

Forcep delivery is NOT recommended in which of the following conditions? A. Class II-IV gravidocardiac B. Vaginal delivery after Cesarian section C. Presence of acute pulmonary edema D. Presence of persistent late decelerations at station +4 E. None of the above

629 D.

A 25 year old at 32 weeks AOG is noted to have a BP of 160/100 mmHg with the presence of +2 proteinuria. The platelet count and liver function tests were abnormal. After few hours, patient went into seizure. What is the best management plan in this case? A. Induction of labor B. Intelligent expectancy C. Control the seizure then expectant management D. Cesarian section E. Forcep delivery

632 D. CS is the only cure for eclampsia.

A 26 year old came in to the emergency department due to vaginal bleeding and severe abdominal pain. She was amenorrheic for about 2 months. On physical examination, patient was pale looking and hypotensive. Which of the following events would be most likely predispose this patient to your diagnosis? A. Pelvic inflammatory disease B. Use of IUD C. Previous tubal surgery D. Exposure to diethylstilbestrol E. None of the above

633 A. This is a case of ectopic pregnancy. PID is the most common predisposing factor.

Amor is pregnant and visits your clinic for a regular pre-natal check up. She had one ectopic pregnancy at 4 weeks. She has one baby born at 40 weeks and one born at 32 weeks which is a set of twins. What is her OB score? A. G3P3 (1113) B. G4P2 (1113) C. G3P3 (2113) D. G4P2 (2113) E. G4P2 (1213)

634 B. G4P2 (1 1 1 3)

A primigravid consulted for amenorrhea of 12 weeks. Home pregnancy test was positive but you were unable to appreciate fetal heart tones by doppler. Transvaginal ultrasound showed a fetus within an enlarged uterus without cardiac or somatic activity. IE revealed a closed cervix. The most likely diagnosis is? A. Blighted ovum B. Missed abortion C. Threatened abortion D. Imminent abortion E. Inevitable abortion

635 B. This is a case of missed abortion

The structure that is cut during median episiotomy is the: A. Ischiocavernosus B. Bulbocavernosus C. Internal anal sphincter D. Rectal mucosa E. None of the above

636 B.

When the fetus is in "military attitude", the presenting diameter is? A. Suboccipitobregmatic B. occipitomental C. Occipitofrontal D. Submentobregmatic E. None of the above

637 C.

The most common clinical manifestation of molar pregnancy is: A. Vagial bleeding B. Uterine size smaller than the age of gestation C. Uterine size larger than the expected AOG D. Fetal heart tones absent at 20 weeks AOG E. None of the above

638 A.

A 41 year old had a baby with Down syndrome 5 years ago. She is now anxious to know the chromosome status of her fetus in her current pregnancy . The test that has the fastest lab processing time for karyotyping is? A. Amniocentesis B. Cordocentesis C. Chorionic villous sampling D. Doppler flow ultrasound E. None of the above

639 C. Chorionic villous sampling is usually done in the 1st trimester.

Zavanelli, as a management of shoulder dystocia refers to: A. Cephalic placement into pelvis followed by Cesarian section B. Pressure is applied to the infants jaw and neck in the direction of the mother's rectum with strong fundal pressure applied by the assistant as anterior shoulder is freed C. Cutting of the clavicle with scissors or other sharp instruments D. Surgical incision into the fibrocartilage of the symphysis pubis E. None of the above choices

640 A. Zavanelli- cephalic placement into pelvis followed by CS

Maja Rotzky, a 19 y/o primigravid consulted on your clinic for her first prenatal check-up. The fundic height was measured to be at the level of umbilicus. Assuming that this is NOT a mutifetal pregnancy and there is NO structural, or medical abnormality, approximately how many weeks AOG is this pregnancy? A. 12 weeks B. 16 weeks C. 18 weeks D. 20 weeks E. 24 weeks

641 D. at 12 weeks, FH is at the level of symphysis pubis. At 16 weeks, FH is halfway between the symphysis and umbilicus. At 20 weeks at the level of umbilicus. At 32 weeks about the level of xiphoid process.

Cathy Poe is a 31 year old female who is complaining of cyclic pelvic pains since 8 months PTC. She thought this was just an ordinary case of dysmenorrhea hence she self-medicated with NSAID's. However, 2 weeks PTC, patient experienced defecating with blood. You requeste dultrasound which revealed hyperechoic sacs on both ovaries which possibly blood-containing. What is your diagnosis? A. Teratoma B. Theca Lutein Cysts C. Endometriosis D. Adenomyosis E. Polycystic ovarian syndrome

642 C. Endometriosis is the presence of endometrial tissue outside the uterus. Hence accumulation of blood to solid organs during menstrual periods, causes cyclic pain.

Halley is a 23 year old promiscuous lady who came in due to right upper abdominal pain. Further history revealed previous episodes of STD but did not comply with the complete treatment course. Laparoscopy was done and showed periportal hepatitis with characteristic violin string adhesions. What is your impression? A. Fitz-Hugh Curtis Syndrome B. Weil's Syndrome C. Waterhouse Friderichsen syndrome D. Sheehan's Syndrome E. Asherman Syndrome

643 A. Fitz Hugh Curtis Syndrome is a rare complication of PID due to ascending infection from lower genital tract towrds uterus, oviduct and upto the hepatic capsule.

You are consulted by a 28 year multigravid who is on her 2nd week post-partum because she was unable to express milk for her child. Further history revealed profuse bleeding during her last childbirth. With that kind of history, you are able to diagnose the patient with? A. Fitz-Hugh Curtis Syndrome B. Weil's Syndrome C. Waterhouse Friderichsen syndrome D. Sheehan's Syndrome E. Asherman Syndrome

644 D. Sheehan's syndrome is the postpartum pituitary necrosis. During gestation, the pituitary gland increases it's size to almost 130%. If there is acute blood loss, ischemic necrosis may occur hence oxytocin is not produced, leading to lack of milk ejection, and amenorrhea.

This is an abnormal uterine bleeding characterized by scanty menstruation. A. Menorrhagia B. Hypomenorrhea C. Oligomenorrhea D. Polymenorrhea E. Metrorrhagia

658 B. Hypomenorrhea refers to scanty menstruation. Oligomenorrhea refers to prolonged intervals of menstruation.

The most common cause of secondary amenorrhea is? A. Ovarian tumor B. PCOS C. Imperforated hymen D. A or B E. None of the above

659 E. SPERM is the most common cause of secondary amenorrhea ! ! !

A 24 year-old primigravid with 9 weeks AOG consulted due to passing of meaty material. History started a day PTC, patient had crampy abdominal pain (7-8/10 in pain scale) asscociated with spotting. Few hours PTC, patient noted passing-out of meaty material per vagina. Speculum exam showed closed cervix. To be sure, you requested ultrasound and revealed empty uterine cavity. What is your next step? A. Dilatation and curettage B. Give Tranexamic acid 500mg, 2 capsules 3x a day for 3 days C. Administer 10 units of Oxytocin via 30 minute drip D. Administer 1.2 grams of Co-amoxiclav IV every 12 hours for 3 days E. Do watchful observation

645 E. this is a case of Complete abortion, hence watchful observation is warranted to prevent severe bleeding, sepsis and alike. D and C is not indicated, just like Oxytocin, Tranex and antibiotics (as long that this is NOT an instrumentally-induced)

The most common type of breech presentation is characterized by? A. Lower extremities extended at the hips and extended at the knees B. Lower extremities flexed at the hips and flexed at the knees C. Lower extremities extended at the hips and flexed at the knees D. Lower extremities flexed at the hips and extended at the knees E. None of the above

646 D. Frank breech is the most common type of breech and is characterized by lower extremities flexed at the hips and extended at the knees

The most common symptom found in complete mole is? A. Vaginal bleeding B. Abdominal pain C. Uterine enlargement D. Nausea and vomiting E. A and D

647 A. the question is asking for the MOST COMMON SYMPTOM hence vaginal bleeding is the only and definite answer

A 25 year-old female, with several previous episodes of PID, came in due to severe right lower quadrant pain. No associated fever, diarrhea or vomiting noted. Further history revealed missed menses for 2 months. Pregnancy test revealed positive and you are entertaining ectopic pregnancy. If you are to perform ultrasound, you should know that a gestational sac should be seen once the level of hCG is? A. > 1,500 units B. > 1.50 units C. > 2, 500 units D. > 2.5 units E. > 3,500 units

648 A. The one being asked is the discriminatory level of beta-hCG. Meaning, if beta-hCG is >1,500 units, a gestational sac should be seen via ultrasound

Which among the following statements is/are tru regarding medical treatment of Ectopic pregnancy? A. < 6 weeks AOG B. < 3.5 cms in length C. Beta hCG of < 15,000 miU/mL D. A and C only E. All of the above

649 E.

What is the most common cause of DIC in pregnant women? A. Placenta Previa B. Vasa Previa C. Placenta Acreta D. Abruptio Placenta E. Placenta Increta

650 D. premature separation of placenta causes activation of coagulation cascade via exposure to tissue factor which leads to a viscious cycle called DIC.

Vaginal delivery follows a series of fetal movement in relation to uterine contractions, which are called Cardinal movements. Which cardinal movement is the most important and is the first prerequisite for vaginal birth? A. Engagement B. Descent C. Flexion D. Extension E. Internal rotation

651 B. Descent occurs even before engagement and is continuous until the fetus is delivered.

Which of the following is/are criteria for using Oxytocin? A. Cervix should atleast 4cms opened B. CPD is ruled out C. Cephalic presentation D. B and C only E. All of the above

652 E. The judicial use of Oxytocin should start when the labor is already at the active phase (4cms cervical dilatation), no CPD and the child is on cephalic presentation.

A 27 year-old nulligravid have undergone explore laparotomy due to ovarian tumor. Grossly, the tumor contains teeth, hair and bone. What is your impression? A. Dermoid cyst B. Seminoma C. Teratoma D. A and C only E. All of the above

653 D. Dermoid cyst is the most common ovarian tumor in patients less than 30 years old. Since it is derived from all germ layers, it can differentiate into any other tissues like bones, teeth, or cartilage.

Which of the following is/are probable sign/s of pregnancy? A. Positive pregnancy test B. Braxton-Hicks contractions C. Goodell's sign D. A and B only E. All of the above

654 E. Probable signs are changes related to placenta or uterus

Human Placental Lactogen is being pointed-out as the cause of gestational diabetes. Its peak levels can be detected on what age of gestation? A. 20-22weeks B. 22-26weeks C. 24-28 weeks D. 28-32weeks E. 30 weeks

655 C. HPL is at its peak between 24-28 weeks, and this is the ideal time to screen for GDM

The minimum criteria for diagnosing Pelvic Inflammatory Disease includes A. Cervical tenderness B. Uterine tenderness C. Adnexal tenderness D. A and C only E. All of the above

656 E. These are the 3 minimun criteria for diagnosing PID.

Call Exner bodies are the pathognomonic finding in? A. Sertoli-Leydig cell tumor B. Granulosa-theca tumor C. Endodermal Sinus tumor D. Struma Ovarii E. Choriocarcinoma

657 B.

True of physiologic discharge of puberty? A. Occurs 6-12 months before menarche B. Due to desquamation of vaginal epithelium C. Greenish white in color D. A and b E. All of the above

661 D. Physiologic discharge is grayish - white in color.

True of nabothian cyst, except? A. Most common cervical cyst B. Most common in postmenopausal women C. Due to intermittent blockage of endicervical gland D. Treatment is supportive E. All of the above

662 B. A nabothian cyst (or nabothian follicle) is a mucus- filled cyst on the surface of the cervix. They are most often caused when stratified squamous epithelium of the ectocervix (toward the vagina) grows over the simple columnar epithelium of the endocervix (toward the uterus). This tissue growth can block the cervical crypts (subdermal pockets usually 2-10 mm in diameter), trapping cervical mucus inside the crypts. The transformation of tissue types is called metaplasia. Occurs most comonly in menstruating women.

A 35 year old obese female presented with bleeding in between periods, what would be the initial diagnostic modality of choice? A. Ultrasound B. Biopsy C. Laparotomy D. Progesterone withdrawal test E. None

663 A. An endometrial polyp or uterine polyp is a mass in the inner lining of the uterus.They may have a large flat base (sessile) or be attached to the uterus by an elongated pedicle (pedunculated). Can be detected by vaginal ultrasound (sonohysterography), hysteroscopy and dilation and curettage.

Triad of polycystic ovary syndrome includes which of the following? A. Hirsutism B. Amenorrhea C. Obesity D. Abdominal Pain E. A, b and c

664 E. The principal features are anovulation, resulting in irregular menstruation, amenorrhea, ovulation- related infertility, and polycystic ovaries; excessive amounts or effects of androgenic hormones, resulting in acne and hirsutism; and insulin resistance, often associated with obesity, Type 2 diabetes, and high cholesterol levels.

Normal semenalysis values? A. Concentration >20M/mL B. pH 8 - 9 C. Motility>30% D. Normal forms>50% E. A and b only

665 A. Concentration >20M/mL Volume 2 - 5 mL pH 7.2 - 7.8 mL motility >50%

Grade of pelvic organ prolapse wherein the prolapsed organ is halfway past the hymenal ring? A. 1 B. 2 C. 3 D. 4 E. 5

666 C. Grade 1 - descend halfway to hymen Grade 2 - level of hymen Grade 3 - halfway past hymenal ring Grade 4 - Prolapsed organ completely outside vagina

What is the most frequent site of eccyesis? A. Ampulla B. Isthmus C. Interstitial D. Supero-posterior part of uterus

688 A. Answer: A. Ampulla (pp.35, OB-Gyne, Topnotch Handouts) Notes: Eccyesis is another term for ectopic pregnancy.

The fetal heart tones can be heard by Doppler by how many weeks AOG? A. 6 - 7 B. 10 C. 12 D. 19 E. 20

671 B. 6 - 7 weeks by Transvaginal UTZ 10 weeks by UTZ 19 weeks by Stethoscope

This is the earliest histological evidence of progesterone action in the endometrial lining. A. Glandular mitoses B. Stromal edema C. Basal vacuolization D. Secretion E. Pseudostratification of nuclei

672 C. At the time of ovulation the endometrium slows in its growth, and it ceases mitotic activity within days after ovulation, at which time the corpus luteum is producing progesterone in addition to estrogen. The postovulatory endometrium is initially marked by secretory vacuoles beneath the nuclei in the glandular epithelium . This secretory activity is most prominent during the third week of the menstrual cycle, when the basal vacuoles progressively push past the nuclei.

For pregnancy to occur, implantation of what structure must occur on the 8th day of fertilization? A. zygote B. blastomere C. morula D. blastocyst E. Fetus

673 D. Blastocyst implants at around 7 days post - conception within the superior wall of the uterus.

Goodell sign? A. Softening and compressibility of the isthmus occuring on the 6th - 8th week AOG B. Bluish/purplish discoloration of the vagina at 6 weeks AOG C. Mask of pregnancy D. Softening of the cervix E. Ferning pattern

674 D. Goodell's sign is an indication of pregnancy. It is a significant softening of the vaginal portion of the cervix from increased vascularization. This vascularization is a result of hypertrophy and engorgement of the vessels below the growing uterus.

Definitive evidence of pregnancy, EXCEPT? A. Fetal heart action identification B. Fetal movement perception by examiner C. Recognition of embryo or fetus by UTZ D. Positive pregnancy test E. FHT by doppler at 10 weeks AOG

675 D. Pregnancy test is a probable evidence of pergnancy

Definition of a reactive NST: A. 2 or more accelerations B. Peak at 15 bpm above baseline C. Each acceleration lasts more than 30 seconds D. A and b E. All of the above

676 D. Reactive NST - presence of two or more fetal heart rate accelerations within a 20-minute period, with or without fetal movement discernible by the woman. Accelerations are defined as 15 bpm above baselines for at least 15 seconds if beyond 32 weeks gestation, or 10 bpm for at least 10 seconds if at or below 32 weeks.

Biophysical profile of a pregnant patient was done which revealed BPS of 8 with decreased AF, AOG<37 weeks, what is the management of choice? A. Repeat test per protocol B. Administer steroids C. Deliver D. No fetal indication for intervention E. None

677 A. iHowever, this would be dependent on the AOG. According to our lecture, if AOG <37 weeks, repeat test per protocol. If AOG>37 weeks, deliver.

Type of deceleration wherein a drop of heart rate is seen to start at the peak of contraction but would return to baseline HR after the contraction ? A. Early deceleration B. Head compression C. Late deceleration D. Variable deceleration E. Cord compression

678 C. Late deceleration may signal uteroplacental insufficiency.

An 18 year old female came to the ER presenting with vaginal bleeding. She stated to observe passage of meaty material form the vagina. What would be the management of choice for this patient? A. tocolysis B. Completion curettage C. bed rest D. IV antibiotics E. prostaglandins

679 B. This is a case of incomplete abortion wherein there is already passage of parts of the fetus. The cervical os is open, bag of waters ruptured.

type of version used for delivery of the second twin? A. Internal podalic version B. External cephalic version C. Internal cephalic version D. External podalic version E. None

680 A. Podalic version is an obstetric procedure wherein the fetus is turned within the womb such that one or both feet present through the cervix during childbirth. It is used most often in cases where the fetus lies transversely or in another abnormal position in the womb.

A 25 yo, G3P4 comes to your office for a routine prenatal check-up, complaining of vaginal bleeding. Patient is 16 weeks AOG based on LMP. Upon PE, her uterus is at the level of the umbilicus and no FHTs can be heard. BP is 150/90 mmHg with no prior history of hypertension. Urinalysis showed 2+ proteinuria on dipstick. Which of the following condition should you rule out first? A. IUFD B. Molar pregnancy C. Preeclampsia D. Gestational hypertension

681 B. Answer: B. Molar pregnancy (pp. 19, High-Yield OB- Gyne, 2nd edition) Notes: Molar pregnancy is the only time preeclampsia is diagnosed prior to 20 weeks. * note that the uterine size is larger than the gestational age. *SIMILAR TO PREVIOUS BOARD EXAM CONCEPT/PRINCIPLE

What is the normal position of the uterus? A. Anteverted and anteflexed B. Anteverted and retroflexed C. Retroverted and anteflexed D. Retroverted and retroflexed

682 A. Answer: A. Anteverted and anteflexed (must know)

What provides the major support of the uterus and the cervix? A. Round ligament B. Uterosacral ligament C. Transverse sacral ligament D. Mackenrodt ligament

683 D. Answer: D. MaCkenrodt ligament (pp. 4, OB-Gyne, Topnotch handouts) Notes: Cardinal ligament synonyms â€" transverse CerviCal ligament or Mackenrodt ligament *There is no such thing as transverse sacral ligament.

Proper recording and evaluation of the periodic changes of the fetal heart rate (FHR) are needed for proper intrapartum assessment. What is the probable etiology if there is gradual decrease in the FHR below the baseline with onset to nadir of at least >30 secs? A. Normal B. Head compression C. Umbilical cord compression D. Uteroplacental insufficiency

684 D. Answer: D. UteroplaCental insuffiCienCy (pp. 21, OB-Gyne, Topnotch Handouts) * Kinds of deceleration were asked 4-5x during the Feb 2013 Boards.

Which sequence of cardinal movements of labor best applies to a fetus delivered via face presentation? A. Descent , Engagement, Internal Rotation, Lateral Flexion B. Descent, Internal rotation, Flexion, Restitution C. Engagement, Descent, Flexion, Internal Rotation D. Engagement, Descent, Extension, External Rotation

685 B. Answer : B. DesCent, Internal rotation, Flexion, Restitution (pp. 27, OB-Gyne, Topnotch Handouts)

How many weeks post-partum does the uterus regresses or involutes to its non-pregnant size? A. 2-3 days B. 2 weeks C. 3 weeks D. 4 weeks

686 D. Answer: D. 4 weeks (pp.32, OB-Gyne, Topnotch Handouts) SIMILAR TO PREVIOUS BOARD EXAM CONCEPT/PRINCIPLE

Benign GTD can be classified as complete and incomplete mole. An incomplete mole has 3 sets of chromosomes due to which of the following reasons? A. A haploid egg is fertilized by 2 normal sperms B. A haploid egg is fertilized by a diploid sperm C. A diploid egg is fertilized by a haploid sperm D. All of these

687 A. Answer: InComplete means one haploid egg is fertilized by 2 haploid eggs, so the outCome is diploid

A 26 yo, G1P1 (1001) complains of amenorrhea. She had hypotensive episodes during her delivery 7 months ago and was transfused 10 units of fresh whole blood. She was not able to breastfeed because her breasts started to sag. She also lost weight, became anorexic and weak. What explains the amenorrhea in this patient? A. Simmond’s syndrome B. Lactational amenorrhea C. Sheehan’s syndrome D. Anorexia nervosa

689 C. Answer: C. Sheehan’s syndrome Notes: Sheehan’s syndrome â€" destruction f pituitary gland as a result of severe bleeding.

A 4 yo girl was brought to the clinic for slight enlargement of the left breast with no other associated signs and symptoms since two years ago. You will tell the parents that premature thelarche is: A. A condition that needs surgical removal of the ovaries B. A benign self-limiting condition that does not require treatment C. A serious condition leading to central precocious puberty D. A condition that requires treatment with hypothalamic suppressive therapy

690 B. Answer: B. A benign self-limiting condition that does not require treatment

A 40 yr-old G4P3 woman comes to the E.R. complaining of vaginal bleeding, pelvic pain, flank pain, foul-smelling discharge and disorientation. Her past medical history is significant for 3 NSVDs and 1 miscarriage. In addition, she did have a history of abnormal Papsmears approx.3 years earlier. What initial lab work-up must be done? A. Complete metabolic panel B. Complete blood count C. B-BHCG levels D. Urinalysis E. All of the above

691 E. Answer: E Differentials include PID, miscarriage, DUB, cervical lesions (including cervical CA), UTI leading to pyelonephritis. CBC & complete metabolic panel are impt.because patient already has disorientation.

Potential sites for ureteral injury during abdominal hysterectomy with bilateral salpigo-oophorectomy include all of the following except: A. Transaction of the round ligament B. Transaction of uterine arteries C. Transaction of cardinal ligaments D. Transition of infundibulopelivc ligaments E. None

692 A. Answer: A In general, ureters do not travel near the round ligament, in a pelvis with normal anatomy.

A 60 yr-old womans visits your clinic with complaints of pelvic pressure and abdominal fullness. Her LMP was 3 yrs ago. BPE and IVP are normal. Staging explore lap is performed. You fin stage 1a ovarian CA. What is the best intervention in this case? A. Start patient on chemotherapy immediately B. Perform TAH-BSO only C. Perform TAH-BSO and infracolic omentectomy D. Cytoreductive debulking will suffice

693 B. Answer: B Histology suggests the CA is confined to the ovary but the 5-yr survival rate is only 80%.

A 19 yr-old primigravida at 32 weeks’ gestation comes to the office for a routine prenatal visit. Her BP is 150/95 mmHg. Her previous BPs have been 120/7- mmHg range. 2 hours later: While receiving IV MgSO4 therapy, her RR have decreased from 20- 5 rpm. Findings are consistent with A. Gestational HTN B. Mild preeclampsia C. Severe preeclampsia D. Eclampsia E. Magnesium toxicity

694 E. Answer: E Antidote for magnesium toxicity is IV calcium gluconate.

A primigravida at 16 weeks by LMP has a fundal height at umbilicus. She has abnormality elevated levels of MS-AFP and B-hCG. You: A. schedule a sonogram to rule out multiple gestation B. are confident it’s Down’s syndrome C. diagnose patient as having molar pregnancy D. admit patient and watch out for variable decelerations

695 A. Answer: A The combination of fundus larger than dates and abnormally elevated levels of MS-AFP and B-hCG suggests multiple gestation.

A 43 yr-old woman comes to the office complaining of involuntary urine loss. Loss of urine occurs continuously day and night along with pelvic pressure. Residual volume is 450 ml. a. Genuine incontinence b. Bypass incontinence c. Overflow incontinence d. Motor urge incontinence

696 C. Answer: C Overflow incontinence occurs uniquely when intravesical pressure from an overdistended hypotonic bladder exceeds urethral pressure.

How many weeks after abortion does ovulation usually occur? A. 2 to 3 weeks B. 4 to 5 C. 5 to 6 D. 6 to 7

697 A.

Duration of pregnancy is most correctly measured clinically by which of the following units? A. Number of weeks, rounded to the nearest whole week since the first day of LMP B. Completed weeks since first day of LMP C. Completed weeks since estimated date of conception D. Numbers of weeks rounded to the nearest whole week since the estimated date of conception

698 B.

In which presentation is the fetal head partially flexed and a large anterior fontanel presenting? A. Vertex B. Face C. Brow D. Sinciput

699 D.

Which of the following is not an indication of severe pregnancy-induced hypertension? A. Upper abdominal pain B. Oliguria C. Creatinine 0.6 mg/dl D. Fetal growth restriction

700 C. Answer: C. Creatinine > 1.2 mg/dl.

18. A 52 year old female came in for complaints of hot flashes as it disrupts her sleep most days of the week. She asks for your advise on management of her hot flashes. Which of the following is true? a. Underweight women have more severe symptoms b. SSRI/SNRI are the mainstay nonhormonal treatment of hot flashes c. SSRI are FDA approved d. lifestyle interventions do not decrease symptoms e. vaginal dryness is the primary reason women seek care during menopause

98 B. overweight women and those who smoke have more severe symptoms. SSRI/SNRI are not FDA approved. Being in a cool environment is associated with fewer subjective and objective hot flashes. Hot flashes is the primary reason women seek care during menopause. Berek and Novak’s Gynecology 15th edition p.1235

A 29 year old woman underwent vaginal delivery. A midline episiotomy most likely damages which of the following? A. Superficial transverse perineal muscle B. Levator ani muscle C. Puborectalis muscle D. Pubococcygeus muscle E. All of the above

1 A. The other structures are located too deep to be injured in a midline episiotomy. The muscles attached to the perineal body are the superficial and deep transverse perineal muscle, bulbospongiosus and external anal sphincter. Source: Moore Clinically oriented anatomy 4th ed Chapter 3 (Ebook)

Engagement occurs when the BPD of the fetal head descends below the level of A. Midpelvis B. Pelvic inlet C. Pelvic floor D. Ischial tuberosity E. NOTA

46 B.

A 32 year old G3P2 2002 with history of 2 uncomplicated NSD is admitted for labor at 39 weeks AOG . 2 hours prior, her IE was 5cm/60% effaced/-2 station. Her IE now is 6cm/60% effaced/-2 station. What abnormal labor pattern is she exhibiting? A. She is not exhibiting any abnormal labor pattern B. Prolonged latent phase C. Protracted active phase dilation D. Arrest in descent E. Arrest in dilation

10 C. This is not arrest in descent. Arrest in descent occurs in the second stage of labor. Patient is still in the 1st stage of labor. Since the patient is a multipara, she should dilate 1.5cm/hr and should at least already be 8cm. The next step in management is to determine if there are adequate contractions and to rule out a fetopelvic disproportion. (Note: not cephalopelvic disproportion. We already know her pelvis is adequate because she delivered 2 VSDs already.) Source: Williams Obstetrics 23RD ed p465

20. A 11 year old came in for complaints of a rash on the perineum. On close examination, umbilicated pinkish pruritic papules was noted. Which of the following is the most likely etiology of the rash? a. poxvirus b. papiilloma virus c. herpes virus d. spirochete e. cocci

100 A. molluscum contagiosum

Which of the following may be given postpartum after expulsion of placenta to prevent postpartum hemorrhage? A. Ergonovine B. Ergotamine C. Methysergide D. A and B E. All of the above

11 A. Ergonovine and methylergonovine are 5HT2 receptor antagonist that are uteroselective and used for postpartum hemorrhage. Ergotamine, methysergide are also 5HT2 receptor antagonist that are vasoselective which are used for migraine treatment. Topnotch handout in pharmacology

In which part of the uterus is the incision made for a low transverse cesarean section? A. Fallopian tube B. Fundus C. Body D. Cervix E. Lower uterine segment

12 E. Source: Williams Obstetrics 23RD ed p550

A 22 year G3P3(3003), 3 weeks postpartum seeks advice regarding contraception. Which postpartum contraception is ideal for the patient if she is breastfeeding? A. Combined OCP B. Progestin only pills C. IUD D. Withdrawal method E. All of the above

13 B. Combined OCPs may reduce the amount of breast milk. IUDs are not used while the uterus is undergoing involution due to risk of expulsion and uterine perforation. Withdrawal is not effective. Source: Williams Obstetrics 23RD ed 652

A neonate is born to a 25 year old G1P1 1001 who has active chronic hepatitis B infection. How should the neonate be managed? A. Give anti-HBs IgG only B. Give Hepatitis B vaccine only C. Give Both HBIG and vaccine at birth D. Give HBIG then start vaccine after 4 weeks E. Give Acyclovir + Lamivudine

14 C. Give both HBIG and vaccine. The infant may be breastfed if given prophylaxis and vaccinated.

Patient presents at 7 weeks AOG with vaginal bleeding and denies passage of meaty material. On PE, the cervical OS is open at 5cm. US shows cardiac activity. What is the diagnosis? A. Threatened abortion B. Inevitable abortion C. Incomplete abortion D. Complete abortion E. Missed abortion

15 B. Management for this patient is expectant or D and C. Source: Williams Obstetrics 23RD ed Chapter 9 Abortion

A 60 year old postmenopausal female comes in with 1 month history of spotting not associated with pain. An ultrasound showed thickened endometrium. And biopsy confirmed malignancy. Which of the following is not a risk factor? A. PCOS B. Multiparity C. Early menarche D. Obesity E. Liver disease

16 B. Nulliparity is a risk factor for endometrial CA. Multiparity is protective. Source: Topnotch handout on Obstetrics and Gynecology

What is the least invasive definitive treatment for adenomyosis? A. GnRH antagonist B. NSAIDs C. OCPs D. Endometrial ablation E. Hysterectomy

17 E. Hysterectomy is the definitive treatment for adenomyosis. Source: Topnotch handout on Obstetrics and Gynecology

A 60 year old postmenopausal female presents with postmenopausal spotting. A TVS showed thickened endometrium and a right ovarian mass. There was also elevated inhibin levels. Biopsy revealed Schiller duval bodies. Which of the following is the most likely diagnosis? A. Theca lutein cyst B. Dysgerminoma C. Yolk sac tumor D. Granulosa cell tumor E. Teratoma

18 D. A granulose cell tumor may release estrogen resulting to endometrial hyperplasia and sometimes even endometrial carcinoma. Source: Topnotch handout on Obstetrics and Gynecology

A 25 year old primipara gives birth to a neonate with IUGR, microcephaly, craniofacial dysmorphism, hypopastic nails and distal phalanges. Which of the following teratogen may be responsible for this? A. Ethanol B. Lithium C. Phenytoin D. Thalidomide E. Methimazole

2 C. This case describes fetal hydantoin syndrome. It is due to ingestion of the antiseizure drug phenytoin. Ethanol - Fetal alcohol syndrome Lithium - Ebstein anomaly Phenytoin - Fetal hydantoin syndrome Thalidomide - phocomelia Methimazole - aplasia cutis congenita Source: Topnotch handout on pharmacology

A 19 year old nulliparous sexually active female seeks consult for malodorous, greenish, frothy vaginal discharge. On obtaining specimen for TMG smearing, you noted petchiae all over the cervix. What is the treatment for this patient? A. No treatment necessary B. Clotrimazole 2% cream 5g intravaginally for 7- 14days C. Metronidazole 500mg BID x 7 days D. Metronidazole 2g single dose E. Oral acyclovir 800mg QID x 7 days

20 D. This is a case of trichomoniasis which is caused by a flagellated motile protozoan seen on saline wet mount microscopy. It is associated with strawberry cervix. Source: Topnotch handout on Obstetrics and Gynecology

Appropriate antibiotic treatment for chancroid is A. Penicillin B. TMP-SMX C. Azithromycin D. Doxycycline E. Co-amoxiclav

21 C. There are four antibiotics that can be used to treat chancroid: azithromycin, ceftriaxone, ciprofloxacin, and erythromycin. Chancroid is a contagious sexually transmitted ulcerative disease of the vulva caused by Haemophilus ducreyi, small gram-negative rods that exhibit parallel alignment on Gram staining ("school of fish"). After a short incubation period, the patient usually develops multiple painful soft ulcers on the vulva, mainly on the labia majora and, less commonly, on the labia minora or involving the perineal area. The chancroid ulcer has ragged, irregular borders and a base that bleeds easily and is covered with grayish exudates. Approximately half the patients will develop painful inguinal lymphadenitis within 2 weeks of an untreated infection, which may undergo liquefaction and presents as buboes. These may rupture and discharge pus. Diagnosis is made by Gram stain and, less commonly, by culture.

A 26-year-old woman underwent a Pap smear, which revealed a low-grade squamous intraepithelial lesion. There was no inflammation found. She was also found to be HIV negative, and a Pap smear 2 years ago was negative. The most appropriate next step in her treatment is: A. Colposcopy and biopsy B. Cervical conization C. Cryotherapy or laser therapy D. Return in 4-6 months for repeat Pap smear E. Hysterectomy

22 A. The correct answer is colposcopy with biopsy. Cervical cancer mortality is so preventable because it is easily revealed by Pap smear, but this is only a screening test. If the Pap smear returns with low- or high-grade squamous intraepithelial lesion, a punch biopsy is required for diagnosis of CIN or invasive carcinoma. If the biopsy shows CIN I (slight dysplasia), this may resolve and does not require treatment. CIN I is managed with follow-up and a repeat Pap smear in 4-6 months. CIN II and III (moderate and severe dysplasia) are treated with ablative therapy using either laser or cryotherapy. If the punch biopsy returns showing invasive carcinoma, staging is performed, and the patient is treated with hysterectomy and/or radiation therapy. If the biopsy is inconclusive, then cervical conization is indicated.

Which of the following physiologic changes occurs during pregnancy? A. Increased systemic vascular resistance B. Decreased minute ventilation C. Decreased gastric motility D. Decreased fibrinogen E. Increased venous return from lower extremities

23 A and C. Gastric motility and lower extremity venous return are decreased. Systemic vascular resistance, minute ventilation, and fibrinogen are all increased.

A 32-year-old G1 at 39 weeks gestation is admitted in labor at 4 cm dilated and completely effaced; the fetal head is at 0 station. You perform clinical pelvimetry and find the following: the diagonal conjugate is 10 cm, the interischial spine distance is 11 cm with nonconvergent sidewalls, and the intertuberous distance is 9 cm. Those measurements describe which of the following types of pelvis? A. Normal pelvis B. Contracted pelvic inlet C. Contracted midpelvis D. Contracted pelvic outlet E. Generally contracted pelvis

24 B. The pelvic inlet is considered contracted if the anteroposterior diameter is less than 10 cm. The inlet is digitally measured by the diagonal conjugate which is typically 1.5 cm greater than the inlet, therefore a pelvic inlet contraction is defined as a diagonal conjugate less than 11.5 cm. The midpelvis extends from the inferior margin of the symphysis to the ischial spines bilaterally to the sacrum near the junction of the fourth and fifth vertebrae. The average mid-pelvis measurements include: interischial spinous 10.5 cm, anteroposterior from symphysis to sacrum 11.5 cm and posterior sagittal from midpoint of interspinous line to sacrum 5 cm. But there is no precise manual measurement of the midpelvis but contraction is suggested if the spines are prominent, the pelvic sidewalls converge or if the sacrosciatic notch is narrow. The contracted outlet is defined as a intertuberous diameter of 8 cm or less. Outlet contraction without concomitant midpelvis contraction is rare. A generally contracted pelvis is caused by combinations of contractions in the inlet, midpelvis , and outlet.

On routine examination, it is discovered that a 35- year-old woman had been exposed in utero to diethylstilbestrol administered to her mother, who had had a history of recurrent spontaneous abortion. This history suggests that the patient might be at increased risk of (A) adenomyosis (B) clear cell adenocarcinoma (C) lichen sclerosus (D) sarcoma botryoides (E) squamous cell carcinoma

25 B. Diethylstilbestrol = DES

Which step in the steroid hormone synthetic pathway is required for the development of female secondary sex characteristics, but not male secondary sex characteristics? A. Aldosterone synthase B. Aromatase C. Cholesterol desmolase D. 17,20-Lyase E. 5a-Reductase

26 B. Aromatase catalyzes the conversion of testosterone to estradiol in the ovarian granulosa cells. Estradiol is required for the development of female secondary sex characteristics.

A 35-year-old woman is seen 6 months after giving birth to a normal infant. She suffered severe cervical lacerations during delivery, resulting in hemorrhagic shock. Following blood transfusion and surgical repair, postpartum recovery has so far been uneventful. She now complains of continued amenorrhea and loss of weight and muscle strength. Further investigation might be expected to demonstrate which of the following findings? A. Decreased serum cortisol B. Hyperestrinism C. Hyperglycemia D. Increased hair growth in a male distribution pattern E. Increased serum free thyroxine

27 A. The history is strongly suggestive of panhypopituitarism due to ischemic necrosis of the pituitary, occurring as a sequela to childbirth complicated by hemorrhagic shock (Sheehan syndrome). This syndrome is clinically dominated by overt evidence of gonadotropin and corticotropin deficiencies, along with laboratory evidence of these deficiencies and thyrotropin deficiency. Overt secondary hypothyroidism sometimes occurs.

A 20-year-old G1 at 41 weeks has been pushing for 3 hours. The fetal head is at the introitus and beginning to crown. It is necessary to cut an episiotomy. The tear extends through the sphincter of the rectum, but the rectal mucosa is intact. How should you classify this type of laceration? A. First-degree B. Second-degree C. Third-degree D. Fourth-degree E. Fifth-degree

29 C. A first-degree tear involves the vaginal mucosa or perineal skin, but not the underlying tissue. In a second-degree episiotomy, the underlying subcutaneous tissue is also involved, but not the rectal sphincter or rectal mucosa. In a third-degree tear, the rectal sphincter is affected. A fourth-degree episiotomy involves a tear that extends into the rectal mucosa.

A 30 year old female seeks consult for a lump on her vulva with 2 days duration of pain. There was no associated fever, chills, nausea or vomiting. On PE, a 3x3cm fluctuant tender mass is palpated on the 8 o clock position of her vulva. How should she be managed? A. Incision and drainage B. Marsupialization C. Consider broad spectrum antibiotics D. A and B E. All of the above

3 E. This is a case of Bartholin's gland abscess. The bartholin's glands are located in the 4 and 8 o'clock position of the vaginal orifice. They are normally nonpalpable. Consider taking a biopsy in an older patient to rule out adenocarcinoma. Source: Topnotch handout on Obstetrics and Gynecology

A 56-year-old diabetic woman has recently been treated with a 2-week course of antibiotics for a skin infection. She returns to the clinic for follow- up with a new complaint of a cottage cheese-like vaginal discharge with significant vaginal itching. The most likely cause of these symptoms is A. C. granulomatis B. C. albicans C. G. vaginalis D. T. vaginalis E. N. gonorrhea

36 B. C. albicans is a major cause of vulvovaginitis. Many women are carriers of the yeast, and it therefore constitutes normal vaginal flora, although there are many conditions that alter the vaginal microenvironment, including pregnancy, oral contraceptives, and systemic antibiotics, making overgrowth possible. C. granulomatis causes granuloma inguinale.

A 23-year-old G1 at 38 weeks gestation presents in active labor at 6 cm cervical dilatation with ruptured membranes . On cervical examination the fetal nose, eyes, and lips can be palpated. The fetal heart rate tracing is 140 beats per minute with accelerations and no decelerations. The patient's pelvis is adequate. Which of the following is the most appropriate management for this patient? A. Perform immediate CS. B. Allow spontaneous labor with vaginal delivery. C. Perform forceps rotation in the second stage of labor to convert mentum posterior to mentum anterior and to allow vaginal delivery. D. Allow patient to labor spontaneously until complete cervical dilatation is achieved and then perform an internal podalic version with breech extraction. E. Attempt manual conversion of the face to vertex in the second stage of labor.

30 B. In the event of a face presentation, successful vaginal delivery will occur; the majority of the time with an adequate pelvis. Spontaneous internal rotation during labor is required to bring the chin to the anterior position, which allows the neck to pass beneath the pubis . Therefore, the patient is allowed to labor spontaneously; a CS is employed for failure to progress or for fetal distress. Manual conversion to vertex, forceps rotation, and internal version are no longer employed in obstetrics to deliver face presentation because of undue trauma to both the mother and the fetus .

Fetal death at 15 weeks gestation without expulsion of any fetal or maternal tissue for at least 8 weeks is compatible with: A. Complete abortion B. Incomplete abortion C. Threatened abortion D. Missed abortion E. Inevitable abortion

31 D. A threatened abortion takes place when uterine bleeding occurs without any cervical dilatation or effacement. In a patient bleeding during the first half of pregnancy, the diagnosis of inevitable abortion is strengthened if the bleeding is profuse and associated with uterine cramping pains. If cervical dilatation has occurred, with or without rupture of membranes, the abortion is inevitable. If only a portion of the products of conception has been expelled and the cervix remains dilated, a diagnosis of incomplete abortion is made. However, if all fetal and placental tissue has been expelled, the cervix is closed, bleeding from the canal is minimal or decreasing, and uterine cramps have ceased, a diagnosis of complete abortion can be made. The diagnosis of missed abortion is suspected when the uterus fails to continue to enlarge with or without uterine bleeding or spotting. A missed abortion is one in which fetal death occurs before 20 weeks gestation without expulsion of any fetal or maternal tissue for at least 8 weeks thereafter.

215. A 25-year-old G1 at 37 weeks presents with gross rupture of membranes. On cervical examination, she is noted to be 4 cm dilated, 90% effaced with the presenting part at âˆ'3 station. The presenting part is soft and felt to be the fetal buttock. A quick bedside ultrasound reveals a breech presentation with both hips flexed and knees extended. What type of breech presentation is described? A. Frank B. Incomplete C. Complete D. Single footling E. Double footling

32 A. In frank breech presentation, the lower extremities are flexed at the hips and extended at the knees so that the feet lie in close proximity to the head and the fetal buttocks is the presenting part. With a complete breech presentation, one or both knees are flexed. In incomplete breech presentation, single footling, one hip is not flexed and one foot or knee is lowermost in the birth canal.

A 26-year-old primigravida develops gestational diabetes and remains hyperglycemic during the remainder of her pregnancy. Which of the following abnormalities in the newborn child is likely related to the maternal hyperglycemia? A. Ambiguous genitalia B. Cretinism C. Increased birth weight D. Sheehan syndrome E. Thyroglossal duct cyst

33 C. The most common effect of maternal diabetes mellitus and hyperglycemia on the child is increased birth weight. This also increases the likelihood of obstetric complications, including the need for cesarean section and increased likelihood of brachial plexus injuries. Another complication is hyaline membrane disease. Cretinism results from deficiency of thyroid hormone during fetal development and during postnatal life. Ambiguous genitalia can occur in any of the adrenogenital syndromes. Sheehan syndrome occurs in the mother and has no relationship to diabetes. Thyroglossal duct cysts do not usually result in endocrine complications.

A 68-year-old postmenopausal woman presents for evaluation of the recent onset of vaginal bleeding, and a diagnosis of type I endometrial carcinoma is made on endometrial biopsy. Which of the following is a risk factor for this condition? (A) Endometriosis (B) Multiparity (C) Salpingitis (D) Early sexual activity with multiple partners (E) Obesity

34 E. Endometrial cancer is the most common gynecologic malignancy, and Type I cancers (endometrioid) account for the majority (80%) of these cases. A major predisposing factor to Type I endometrial carcinoma is prolonged and unopposed exposure to estrogen. In the case of obesity, adipose tissue converts androgens into estrogens, fueling the proliferation of endometrial tissue. Similarly, diabetes, nulliparity (not multiparity), and hypertension are also predisposing factors. Early sexual activity with multiple partners is associated with cervical cancer. Salpingitis can predispose to ectopic pregnancy. Endometriosis is a benign condition and has no relation to endometrial cancer.

Soon after an uncomplicated delivery at 38 weeks AOG, a mother develops respiratory difficulties and uncontrolled vaginal bleeding progressing to shock, multi-organ failure, and death. At autopsy, masses of debris and epithelial squamous cells are apparent in the pulmonary microcirculation. Which of the following was the likely cause of death? A. Sheehan syndrome B. Chorioamnionitis C. Abruptio placentae D. Amniotic fluid aspiration syndrome E. Amniotic fluid embolism

35 E. The diagnosis is amniotic fluid embolism, which is characterized by sudden peripartal respiratory difficulty, progressing to shock and often to death. Sheehan syndrome is associated with obstetric blood loss, with resultant pituitary ischemia leading to postpartal hypopituitarism. Chorioamnionitis is infection of the placenta, which can have devastating consequences for both the mother and the child. Abruptio placentae is premature separation of the placenta and can lead to antepartal bleeding and fetal death. Amniotic fluid aspiration can occur in the child and can result in difficulties for the infant, particularly if meconium is aspirated. Amniotic fluid embolism and abruptio placentae are well-known causes of DIC, as are retained dead fetus and toxemia.

What is most common cause of perinatal mortality in twin gestation A. Single fetal demise B. Twin to twin transfusion syndrome C. Prematurity D. IUGR E. Two fetal demise

43 C. Prematurity is the most common cause of perinatal mortality in twin gestation.

True of the ischial spine A. Plane of the least pelvic dimension B. Pudendal block is carried out at this level C. Internal rotation occurs when the occiput is at this level D. External OS of the cervix is located normally E. AOTA

44 E.

A 23-year-old G1 at 40 weeks gestation with the complaint of contractions. She states they are occurring every 4 to 8 minutes and each lasts approximately 1 minute. She reports good fetal movement and denies any leakage of fluid or vaginal bleeding. The nurse places an external tocometer and reports that the patient is having contractions every 2 to 10 minutes. You note contractions are mild on palpation. On examination, the cervix is 2 cm dilated, 50% effaced, and the vertex is at -1 station. The patient had the same cervical examination in your office last week. The patient is in what stage of labor? A. Active labor B. Latent labor C. False labor D. Stage 1 of labor E. Stage 2 of labor

37 C. This patient is most likely experiencing false labor, or Braxton-Hicks contractions. False labor is characterized by contractions that are irregular in timing and duration and do not result in any cervical dilation. The intensity of false labor does not change and the discomfort is mainly felt in the lower abdomen and is usually relieved by sedation.

A 23-year-old woman consults an obstetrician because of the onset of vaginal bleeding in what she considers to be the fifth month of pregnancy; however, examination reveals the uterus to be enlarged to the size of a 7-month pregnancy. Intravaginal ultrasound fails to detect a fetal heartbeat and instead shows a snowstorm pattern. HCG is markedly elevated. These findings are strongly suggestive of A. preeclampsia B. eclampsia C. placenta accreta D. ectopic pregnancy E. hydatidiform mole

38 E. Hydatidiform mole should be suspected when the uterus is enlarged beyond the expected size for the time of the pregnancy. HCG is markedly elevated in this gestational trophoblastic disease. Preeclampsia and eclampsia are forms of toxemia of pregnancy marked by severe hypertension, albuminuria, and edema, with the addition of convulsions and DIC in the latter. Placenta accreta occurs when the placenta adheres directly to the myometrium, leading to severe bleeding at the time of delivery. Ectopic pregnancy is usually discovered early in a suspected pregnancy when ultrasound examination reveals the uterus to be empty.

A primipara is in the second stage of labor and an episiotomy is about to be done. Compared with a midline episiotomy, which of the following is an advantage of mediolateral episiotomy? A. Ease of repair B. Fewer breakdowns C. Less blood loss D. Less dyspareunia E. Less extension of the incision

39 E. Midline episiotomies are easier to fix and have a smaller incidence of surgical breakdown, less pain, and lower blood loss. The incidence of dyspareunia is somewhat less. Howwever, the incidence of extensions of the incision to include the rectum is considerably higher than with mediolateral episiotomies. Regardless of technique, attention to hemostasis and anatomic restoration is the key element of a technically appropriate repair.

A 26 year old female complains of amenorrhea for 2 months. Urine pregnancy test was positive. If her LMP was July 20, 2015, when is her EDC? A. March 27, 2016 B. April 13, 2016 C. April 27, 2016 D. May 13, 2016 E. June 13, 2016

4 C. Naegele's rule First day of LMP minus 3 months, plus 7 days, plus 1 year Source: Topnotch handout on Obstetrics and Gynecology

You have just delivered an infant weighing 2.5 kg (5.5 lb) at 38 weeks gestation. Because the uterus still feels large, you do a vaginal examination. A second set of membranes is bulging through a fully dilated cervix, and you feel a small part presenting in the sac. A fetal heart is auscultated at 60 beats per minute. A. External version B. Internal version C. Midforceps rotation D. Low transverse CS E. Classical CS

40 B. According to some studies, 25% of twins are diagnosed at the time of delivery. Although sonography or radiography can diagnose multiple gestations early in pregnancy, these methods are not used routinely in all medical centers. The second twin is probably the only remaining situation where internal version is permissible. Although some obstetricians might perform a CS for a second twin presenting as a footling or shoulder, fetal bradycardia dictates that immediate delivery be done, and internal podalic version is the quickest procedure.

Maximum teratogenecity occurs during A. First two weeks after conception B. 3-8 weeks after conception C. >8 weeks after conception D. Both A and B E. AOTA

41 B.

The smallest circumference of the fetal head A. Suboccipitobregmatic B. Occipitofrontal C. Occipitomental D. Bitemporal E. Biparietal

42 A. BTD (8.0 cm); BPD (9.5 cm); OM (12.5 cm); OF (11.5 cm); SOB (9.5 cm) --> diameter; OF (34.5 cm); SOB (32 cm) --> circumference

A mother came to your clinic for a regular prenatal checkup currently pregnant with twins. Her first pregnancy was an ecotpic pregnancy that was terminated at 8 weeks, her second pregnancy was an H-mole that was terminated by suction curretage at 10 weeks. The third resulted in the live birth of triplet boys at 38 weeks although one died at birth and the other triplet is dying with terminal leukemia. The fourth resulted in the live birth of a daughter at 34 weeks? What is the full OB score? A. G5P4 (3123) B. G5P6 (3122) C G5P4 (3113) D. G5P6 (3123) E. G5P3 (3113)

65 A. In terms of gravidity and parity, mutifetal pregnancy are counted as onebut are counted sepaerately in the TPAL score. H-mole and ectopic pregnancies are counted as gravidities and included as abortions. Expect question regarding OB scoring but not as difficult and colorful as this scenario.

This type of maternal pelvis is narrow anteroposteriorly but wide transversely? A. Gynecoid B. Anthropoid C. Android D. Crystalloid E. Platypelloid

66 E. Gynecoid- round; Android- Triangular/heart shaped; Anthropoid- wide A-P and narrow transversely

At 20 weeks age of gestation, all of the ff developments in the fetus has already occurred EXCEPT? A. Start of production of lung surfactants B. Downy lanugo hair surrounds the skin C. Gender can be determined bu UTZ D. Physiologic herniation of the gut has occured E. Urine poroduction has began

67 A. Lung surfactant production begins at the terminal stage of the lung development at 24-25 weeks AOG and beyond.

While performing a non stress test, you told the mother the test is reactive and reassuring. The mother asked what this means. What is the defintion of a reactive NST? A. 3 or more FHR acelerations that peak atleast 10bpm for 10 secs in a 20 min period B. 3 or more FHR acelerations that peak atleast 10bpm for 10 secs in a 20 min period C. 2 or more FHR acelerations that peak atleast 15bpm for 15 secs in a 20 min period D. 2 or more FHR acelerations that peak atleast 10bpm for 10 secs in a 10 min period E. 2 or more FHR acelerations that peak atleast 15bpm for 15 secs in a 10 min period

68 C. SIMILAR TO PREVIOUS BOARD EXAM CONCEPT/PRINCIPLE

This scenario pertains to question 189-192. A 25 year old G6P4 patient with an LMP of February 2, 2015. Her prior pregancies were unremarkable and she delivered via NSD. She is a non smoker but a heavy alcoholic drinker. Her BP is 140/90 and HR is 78. She came to your clinic because of vaginal bleeding. All of the ff are likely diferentials for the patient EXCEPT? A. Placenta previa B. Threatened Miscarriage C. Abruptio placentae D. Imminent labor E. Abruptio placentae

69 B. All of the choices except threatened miscarriage can explain bleeding in the second half of pregnancy.

At 16 weeks AOG, a 38 year old G2P1 (1001) undergoes a quad screen which revealed increased AFP, normal estriol, HCG and inhibin A. What is the most likely concern for the fetus? A. Nothing. Everything is normal B. Trisomy 21 C. Trisomy 18 D. Ectopic pregnancy E. Neural tube defects

7 E. Quad screen interpretation Trisomy 21 = Increased HCG + inhibin, decreased afp + estriol (My mnemonic is down syndrome = H and I are HI) Trisomy 18 = all are low Causes of increased AFP = NTDs, abdominal wall defects, underestimation of gestational age, fetal death, placental abnormalities, multiple gestation

On further examination,you noted a hard tender globular abdomen with the fetus in a cephalic presentation? What is the primary differential at this time. A. Placenta previa B. Threatened Miscarriage C. Abruptio placentae D. Imminent labor E. Abruptio placentae

70 C. Sudden abdominal pain, vaginal bleeding and a hard tender abdomen is indicative of placental abruption

Accdg to current studies, what is the greatest risk factor for the above condition? A. Prior abruptio B. Preeclampsia/Hypertension C. Short umbilical cord D. Preterm rupture of membranes E. Chorioamnionitis

71 A. All of the choices are risk factors for the development of abruptio but a prior abruptio has a relative risk of 10-50 to develop a subsequent one. Preeclampsia has a relative risk of only 2.1-4.0

All of the ff are complications attributed to the above condition, whoich of the ff is a least likely complication? A. Couvelaire uterus B. Uterine inversion C. Acute Kidney Injury D. Sheehan's syndrome E. Consumptive coagulopathy

72 B. The ff are complications of abruptio placentae: Hypovolemic shock, couvelaire uterus, Acute kidney injury, Sheehan's syndrome and Consumptive coagulopathy. Uterine inversion is assoc more with placenta accreta.

During an internal examination of a 37 weeks AOG primigravid you noted the ff findings: Cervix 4 cm dilated; 60% effaced, medium, midposition, station -2. What is the Bishop's score of the patient? A. 4 B 5 C. 6 D. 7 E. 8

73 D. Pls review the the table for the Bishop scoring.

A 30 year old women comes to your clinic complaining of increasing pelvic pressure, constipation and menorrhagia of about 10 large pads per day. On PE, you palpated a non tender, irregularly enlarged uterus with a lumpy, bumpy firm contour. Her cervix appears normal and she has no evidence of ascites? The patient most likely has what condition? A. Uterine fibroids B. Adenomyosis C. Endometriosis D. Endometrial Malignancy E. Ovarian Malignancy

74 A. Uterine fibroids are ususaly associated with a heavy or prolonged menstrual flow and on physical exam, an irregularly enalrged uterus is palpated versus adenomyosis which has an symmetrical enlarged uterus.

All of the medical therapies can be used to treat menorrhagia in women with the above conditon EXCEPT? A. Combined oral contraceptive pills B. Antifibrinolytic agent C. NSAIDS D. Opiod agonists E. Progestin only pills

75 D. Opiod agonists are narcotic medications used to treat pain. They have no role in the treatment of heavy bleeding in women with fibroids.

A 26 year old G0 patient comes in complaining of intermittent painless mass on her vulva near the introitus. It seems to be aggravated by her intercourse but usually goes away on its own. She's had two lifetime sexual partners and has been with her last partner for 5 years. She never had an STD. You examine her and find a 3 cm non tender mass in the 4 o clock position. What abnormality is present? A. Skene's gland cyst B. Gartner's duct cyst C. Bartholin's duct cyst D. Cystocele E. Epidermal inclusion cyst

76 C. This describes the classic location of thr Bartholin's glands usually at the 4 oclock and 8 o clock position near the introitus providing lubrication of the vagina. The ducts of Bartholin can become blocked resultign in a cyst formation.

An 21 year old nulligravid woman presents to the health clinic with a 4 week history of yellow vaginal discharge. She also reports vulvar itching and irritation. She is sexually active but monogamous.On pelvic exam, there was noted vulvar and vaginal erythema along wit ha yellow frothy malodorous discharge with a pH of 6.5. THe cervix appears to have erythematous punctations? Wet smear was done. What would you expect to see under microscopy? A. Branching hyphae B. Multinucleated giant cells C.Scant WBC D. Flagellated motile organisms E. EPithelial cells covered with bacteria

80 D. The findings and symptoms are consistent with Trichomonas vaginalis infection. The classic cervical finding is a strawberry looking appearance. Motile flagellated protozopans are visible on wet prep microscopic exam.

A 28 year old female presents to the clinic with vaginal pruritus with frothy discharge. Which of the following is the most likely diagnosis? a. bacterial vaginosis b. trichomoniasis c. candidiasis d. herpes simplex e. gonorrhea

81 B. Williams Obstetrics 23rd edition p. 1246

2. In the postpartum ward, PMG 28 year old G2P2002 mother developed pruritic vesicular lesions that crust over in 3 to 7 days spreading from trunk to extremities. Which of the following is true? a. give to newborn VZIG b. give varicella vaccine to pregnant mothers who were exposed at the ward c. vaccine is secreted in breast milk when given postpartum d. vaccine induced immunity is absolute e. Varivax is given 6 months apart to adults with no history of varicella

82 A. VZIG is given to newborns 5 days before and 2 days after maternal clinical evidence of varicella. Give VariZIG to pregnant women exposed to varicella within 96 hours of exposure. Varicella vaccine is not recommended for pregnant women and should not be given to women who may become pregnant during the month following each vaccine dose. The attenuated vaccine is not secreted in milk and postpartum vaccination should not be delayed because of breastfeeding. Vaccine immunity diminishes over time and breakthrough infection occurs 5% in 10 years. Varivax is given 4 - 8 weeks apart. Williams Obstetrics 23rd edition p. 1212

3. FU, 32 year old G3 P1011, comes to the clinic and asks what is she going to do after being exposed to an employee at her office who became AFB sputum positive? a. start with HRZE b. start with isoniazid c. start with HR d. start with HRZES e. start with HRZ

83 B. Isoniazid is considered safe in pregnancy and is given for tuberculin positive patients younger than 35 years old who have no evidence of active disease. INH is given 300mg daily for 1 year. Williams Obstetrics 23rd edition p. 1006

4. Ate Marites, 35 years old G3P1011 24 weeks AOG, diagnosed with medullary sponge kidney comes to the prenatal clinic with complaints of nocturia. Which of the following is true of renal changes in normal pregnancy? a. creatinine increases during normal gestation due to decrease in GFR from compression of uterus b. constriction of ureters which leads to nocturia c. retains same size of kidneys d. increased placental metabolism of AVP which leads to diabetes insipidus e. increased bicarbonate threshold

84 D. GFR and renal plasma flow increases 50% thus decreasing creatinine. There is dilatation of ureters which resembles hydronephrosis on sonogram. Kidney size enlarges 1cm and returns to normal postpartum. Progesterone stimulates respiratory center and decreases bicarbonate threshold. Williams Obstetrics 23rd edition p. 123

5. A 19 year old G0P0 came in for dysfunctional uterine bleeding. Endometrial sampling was done which revealed serrated, dilated glands with intraluminal secretion lined by short columnar cells. Which of the following changes of menstrual cycle is the patient most likely in? a. proliferative b. early secretory c. late secretory d. menstrual e. follicular

85 C. proliferative - straight to slightly coiled tubular glands lined by pseudostratified columnar epithelium early secretory: coiled glands lined by simple columnar epithelium with subnuclear vacuoles menstrual: fragmented endometrium with condensed stroma and glands Williams Obstetrics 23rd edition p. 38

6. A 26 year old female G2 P1001 8 weeks AOG came in for hypogastric pain and vaginal bleeding. TVS showed an adnexal mass. Which of the following is the most likely risk factor for this patient? a. previous PID b. smoking c. prior CS d. documented tubal problem e. previous ectopic pregnancy

86D. Williams Obstetrics 23rd edition p. 239

7. In the emergency room of VSMMC, the intern on duty was able to witness a patient from Oslob bringing a plastic bag containing the pieces which she evacuated in the toilet. Upon inspection of the contents, the intern noticed multiple vesicles with fetal parts. Which of the following is not true in this condition? a. trophoblastic proliferation is widespread b. theca lutein cysts must be removed c. consistent follow up is imperative d. first step in management is chemotherapy e. karyotyping of the contents would reveal 46 XX

87 C. trophoblastic proliferation is only focal as well as villous edema. Theca lutein cysts are just observed and will regress. The first step in management is evacuation of mole and the second is regular follow up to detect persistent trophoblastic disease. Karyotype usually 69XXX or 69XXY for partial moles. Williams Obstetrics 23rd edition p. 258

8. After 3 hours on 8cm cervical dilatation, Mrs MJP 23 years old on 39 weeks AOG, was brought to the operating room for cesarean section. Which of the following will make the anesthesiologist think otherwise in using the epidural route for analgesia? a. use of heparin due to history of APAS b. BP of 80/50 responsive to IV fluid administration c. fetal tachycardia d. preeclampsia e. meconium stained amniotic fluid

88 A. absolute contraindications to regional anesthesia: refractory hypotension, coagulopathy, use of LMWH within 12hours, untreated maternal bacteremia, skin infection at needle placement, and increased ICP from mass lesion. Williams Obstetrics 23rd edition p. 453

9. A 19 year old G1P0 mother came in for watery vaginal discharges. Patient is 38 weeks AOG with regular prenatal check ups at local health center. Upon speculum exam, pooling of fluid is noted and pelvimetry done showed diagonal conjugate of 10cm. WHich of the following is not true in this condition? a. face and shoulder presentations are encountered more frequently b. on internal exam, fetus is at station +3 c. absent pressure of the head on lower uterine segment predisposes to less effective contractions d. cord prolapse are encountered more frequently e. early spontaneous rupture of membranes is more likely

89 B. Williams Obstetrics 23rd edition p. 471

10. A 21 year old G2P1001 mother came in for regular uterine contractions every 2 minutes. Patient had a history of placenta previa during the previous pregnancy in which she underwent cesearean section and delivered a live baby boy 34 weeks. Currently, patient delivered another live baby boy 38 weeks by BS however the intern in charge had difficulty in delivery of placenta. Which of the following is true? a. leave the placenta and give chemotherapy b. histologic diagnosis can be made from the placenta alone c. safest treatment option is still hysterectomy despite young age d. multigravida is the most important risk factor for placenta accreta e. none of the above

90 C. The entire uterus or curettings with myometrium is necessary for histopathologic confirmation. Placenta previa is the most important risk factor next is prior cesarean delivery. Williams Obstetrics 23rd edition p. 777

11. Which of the following is not an etiology of secondary amenorrhea? a. premature ovarian failure b. Turner’s syndrome c. asherman syndrome d. stein - leventhal syndrome e. sheehan’s syndrome

91 B. Berek and Novak’s Gynecology 15th edition p.1051

12. A 12 year old female came in for consult due to amenorrhea. Upon physical examination, pubic and axillary hair was absent and a blind vaginal pouch was seen. Which of the following tests would confirm your impression? a. ultrasound b. hysterosalpingogram c. estrogen and progesterone levels d. karyotyping e. gonadotropin levels

92 D. it is difficult to differentiate a transverse septum or complete absence of the cervix and uterus in a female from a blind vaginal pouch in a male pseudohermaphrodite by examination alone. Androgen insensitivity is likely when pubic and axillary hair is absent. Berek and Novak’s Gynecology 15th edition p.1047

13. Which of the following is the most common and earliest mode of dissemination of ovarian epithelial cancer? a. lymphatic b. hematogeneous c. locally invasive d. transcoelomic e. none of the above

93 D. Berek and Novak’s Gynecology 15th edition p.1367

14. Upon regular PAP smear screening for a 25 year old female, results showed HSIL. Which of the following would be the most appropriate next step in management? a. conization b. repeat PAP smear after 6 months c. HPV DNA testing d. colposcopy e. revert back to routine screening

94 D. any woman with cytologic specimen with HSIL should undergo colposcopy and directed biopsy becuase â..." of patients will have CIN2 or greater. After colposcopically directed biopsy is done, excisional or ablative therapy should be performed. Berek and Novak’s Gynecology 15th edition p.588

15. A 34 year old G0P0 came in for complaints of dysmenorrhea as she did not report to work today in Convergys. She states that she has been having this pain for three years already usually before, during, and after menses usually associated with diarrhea. Which of the following is the confirmatory test for your primary working impression? a. TVS b. CT scan c. laparoscopy d. MRI e. CA 125

95 C. it is the standard technique for visual inspection of pelvis and establishment of definitive diagnosis. Berek and Novak’s Gynecology 15th edition p.516

16. What is the most common cause of anovulation in women? a. Kallman syndrome b. Turner’s syndrome c. asherman syndrome d. stein - leventhal syndrome e. sheehan’s syndrome

96 D. Berek and Novak’s Gynecology 15th edition p.1134

17. A 34 year old female G5P0040 came in for consult for recurrent pregnancy losses. Which of the following workup would you not order? a. karyotyping b. sonohysterogram c. TSH, FT4 d. anticardiolipin antibodies e. none of the above

97 E. Berek and Novak’s Gynecology 15th edition p.1213

19. A 45 year old female came in for complaints of heavy vaginal bleeding. On ultrasound, an adnexal mass 4cm in size was found and subsequently removed. Endometrium was also noted to be thickened. Which of the following would be most likely seen on histopathologic exam of the ovarian mass? a. schiller duval bodies b. flexner rosettes c. call exner bodies d. curschmann spirals e. none of the above

99 C. granulosa cell tumor Berek and Novak’s Gynecology 15th edition p.1407


Conjuntos de estudio relacionados

Ch. 14 Developing and Pricing Goods and Services

View Set

Pack 1, Lesson 2: Practice Exercise 2-1

View Set

Ricci Chapter 3: Anatomy and Physiology of the Reproductive System

View Set

Manufacturing Processes Chapter 5

View Set